Pathophysiology final exam practice

Lakukan tugas rumah & ujian kamu dengan baik sekarang menggunakan Quizwiz!

A health educator is performing a health promotion workshop with the staff of a large, urban homeless shelter, and a component of the teaching centers around tuberculosis. One of the staff members comments, "Anyone who's had contact with tuberculosis in the past can give it to any of the other residents of the shelter, even if they didn't get sick themselves." How could the educator best respond to this comment?

"Actually, people who have the latent form of the disease won't be sick and can't spread it either."

The nurse is performing a health history for a male client who is having a series of diagnostic test to determine the presence of squamous cell carcinoma. What question would be most beneficial for the nurse to ask that would correlate with this suspicion?

"Do you have a history of smoking, and if so, how much?" Explanation: Squamous cell carcinoma is found most commonly in men and is closely correlated with a smoking history.

An adult client with a history of worsening respiratory symptoms has presented for care. Which assessment question will best allow the clinician to address the possibility of chronic bronchitis?

"Do you tend to have a cough even when you don't feel sick?"

A client from Texas is suspected of being infected with coccidioidomycosis and has been admitted to the hospital with fever, cough, pleuritic chest pain, and skin lesions. When performing a health history, which question would be most valuable to assist with diagnosis? a) "Do you smoke?" b) "Do you work in construction and dig soil?" c) "Do you have a history of cancer?" d) "Do you have cats and clean litter boxes?"

"Do you work in construction and dig soil?" Explanation: The disease resembles tuberculosis, and its mechanisms of infection are similar to those of histoplasmosis. It is most prevalent in the deserts in southwestern United States, principally in parts of California, Arizona, Nevada, New Mexico, and Texas. The C. immitis and C. posadasii organism lives in soil and can establish new sites in the soil. Events such as dust storms and digging for construction have been associated with increased incidence of the disease.

A patient has just been diagnosed with acute glomerulonephritis. Which question should the nurse ask this client in attempting to establish a cause? a) "Have you had any type of infection within the last 2 weeks?" b) "Do you have a history of heart failure?" c) "Have you ever been diagnosed with diabetes?" d) "Have you recently had kidney stones?"

"Have you had any type of infection within the last 2 weeks?" Acute post-infectious Glomerulonephritis usually occurs after infection with certain strains of group A β-hemolytic streptococci and is caused by deposition of immune complexes of antibody and bacterial antigens. Other organism can also cause this infection.

A patient has just been diagnosed with acute glomerulonephritis. Which question should the nurse ask this client in attempting to establish a cause? 1."Do you have a history of heart failure?" 2. "have you recently had kidney stones?" 3. "have you ever been diagnosed with diabetes?" 4. "Have you had any types of infection within the last 2 weeks?"

"Have you had any types of infection within the last 2 weeks?"

Which of the following statements by a student demonstrates a sound understanding of the cellular processes of hypertrophy and hyperplasia?

"I know that cells like neurons have little capacity for hyperplastic growth."

A client is diagnosed with Addison's disease. What statement by the client indicates an understanding of the discharge instructions by the nurse?

"I will have to take my medication for the rest of my life."

A client with a pituitary adenoma has had a transsphenoidal removal, successfully. What statement made by the client after education by the nurse indicates the client understands pharmacological treatment?

"I will need to take my medication for about 6 to 12 months, or until my adrenal function returns."

A 51-year-old man has been diagnosed with chronic bronchitis after a long history of recurrent coughing. Which of the man's following statements demonstrates a sound understanding of his new diagnosis?

"If I had quit smoking earlier than I did, I think I could have avoided getting bronchitis."

A client recently diagnosed with bronchiectasis asks what may have caused the condition. Which response by the nurse is accurate?

"It may have developed after you had aspiration pneumonia."

A nurse is changing the wound dressing on the coccyx-region pressure ulcer of an immobilized patient. The existing dressing is saturated with both watery, clear discharge and foul, gray-colored liquid. Which of the following entries in the patient's chart best captures this?

"Large amounts of suppurative and serous exudates noted"

A 24-year-old college student has presented to the campus medical clinic with complaints of frequent, burning urination and has, subsequent to urinalysis, been diagnosed with an acute lower urinary tract infection (UTI) caused by E. coli. What teaching will the clinician most likely provide to the student?

"Many of these bacteria are now resistant to some antibiotics, but I will take that into account when I choose which antibiotic to prescribe."

An elderly patient who was recently diagnosed with emphysema asks the nurse what caused the disease. Which of the following statements is the best response?

"One of the causes of emphysema is a history of cigarette smoking that causes damage to the lungs. Have you ever smoked?"

An elderly patient who was recently diagnosed with emphysema asks the nurse what caused the disease. Which of the following statements is the best response? a) "Emphysema is a permanent dilation of the bronchi and bronchioles caused by destruction of the muscle and elastic supporting tissue as the result of infection and inflammation." b) "This disease is seen most commonly in middle-aged men and is associated with chronic irritation from smoking and recurrent infections." c) "One of the causes of emphysema is a history of cigarette smoking that causes damage to the lungs. Have you ever smoked?" d) "There is no known cause for emphysema; however, it does seem to run in families. Has anyone in your family been diagnosed with this disease?"

"One of the causes of emphysema is a history of cigarette smoking that causes damage to the lungs. Have you ever smoked?" Explanation: The causes of emphysema are smoking, which incites lung injury, and an inherited deficiency of alpha 1-antitrypsin, an antiprotease enzyme that protects the lung from injury.

A frantic mother brings her young child into the emergency department. She states that during the evening bath, she noticed a large mass in her child's abdomen. After diagnostic testing, the pediatrician tells the parents that their child has Wilms tumor, stage IV. After the doctor leaves the room, the parents ask the nurse, "What does this mean?" The nurse will respond, "Your child ('s)

"has cancer in the kidney that has spread most likely to his lungs."

While educating a smoking cessation class, a client asks the nurse, "If I can still get lung cancer, what should I be looking for?" Which response by the nurse is best? a) "When you can no longer get up or have the energy to go to work [fatigue], you should make an appointment with your physician." b) "If you wake up with dried blood on your pillow, call your doctor right away." c) "The earliest symptoms relate to a chronic cough and shortness of breath." d) "If you start losing weight without trying and coughing up thick, yellow sputum, you should seek medical attention."

"The earliest symptoms relate to a chronic cough and shortness of breath." Explanation: The earliest symptoms usually are chronic cough, shortness of breath, and wheezing because of airway irritation and obstruction. Hemoptysis (i.e., blood in the sputum) occurs when the lesion erodes into blood vessels. Later symptoms include coughing up blood, extreme fatigue, and weight loss

As part of a first aid class, a health care instructor is teaching a group of industrial workers about how electrical injuries can cause cell damage. Which of the statements made by one of the workers indicates that further teaching is necessary?

"The greater the skin resistance, the greater the amount of deep and systemic damage a victim is likely to incur."

A child has been removed from a home in which she has experienced severe neglect and emotional abuse, and has been placed in foster care. The child has psychosocial dwarfism and the foster parents ask the nurse what this means for the future of the child. What is the best response by the nurse?

"The prognosis of the child depends on an improvement in behavior and catch-up growth."

A young, male child is born with severe respiratory failure. Over the course of months, the parents note his body looks swollen. They ask, "Is our baby's kidneys not working right? Why is he so swollen?" The nurse bases his or her reply on which of the following physiological principles? a) "The right side of his heart (cor pulmonale) is not pumping effectively. Blood is backlogging in his body, which is why he is so swollen." b) "This happens when he has so many secretions in his lungs. Maybe we should try some expectorant to thin his secretions so he can cough them out." c) "Once we get his oxygenation level back to normal, then maybe his kidneys will receive enough oxygenated blood to filter better." d) "We just need to call the physician and ask him to give you a prescription for more water pills."

"The right side of his heart (cor pulmonale) is not pumping effectively. Blood is backlogging in his body, which is why he is so swollen." Explanation: The term cor pulmonale refers to right heart failure resulting from primary lung disease or pulmonary hypertension. The increased pressures and work result in hypertrophy and eventual failure of the right ventricle. The manifestations of cor pulmonale include signs of right-sided heart failure, which include venous congestion, peripheral edema, shortness of breath, and a productive cough, which becomes worse during periods of heart failure. None of the other statements are applicable to these parents' questions.

A male patient has just been diagnosed with esophageal cancer. He knew that he was losing weight and fatigued most days, but he just attributed it to aging and working. The physician recommends chemotherapy and irradiation. However, the cancer has already metastasized. The patient asks the nurse what he can expect if he agrees to the treatments. The nurse responds, "The therapies may shrink the cancer." "The doctor is prescribing treatment measures to help you swallow better." "These therapies will most likely cure your cancer." "You need to talk with your physician some more. I will page him for you."

"The therapies may shrink the cancer."

When educating a student who lives in a crowded apartment and diagnosed with tuberculosis, the college school nurse will emphasize,

"To destroy this bacterium, you must strictly adhere to a long-term drug regimen."

When educating a student who lives in a crowded apartment and diagnosed with tuberculosis, the college school nurse will emphasize, "Once your fever goes away, you can stop taking the streptomycin injection." "If isoniazid makes you nauseous, we can substitute something milder." "To destroy this bacterium, you must strictly adhere to a long-term drug regimen." "You will have to wear an N95 mask while on campus at all times."

"To destroy this bacterium, you must strictly adhere to a long-term drug regimen."

When educating a student who lives in a crowded apartment and diagnosed with tuberculosis, the college school nurse will emphasize: a) "You will have to wear a N95 mask while on campus at all times." b) "If isoniazid makes you nauseous, we can substitute something milder." c) "Once your fever goes away you can stop taking the streptomycin injection." d) "To destroy this bacterium, you must strictly adhere to a long-term drug regimen."

"To destroy this bacterium, you must strictly adhere to a long-term drug regimen." Explanation: Success of chemotherapy for prophylaxis and treatment of tuberculosis depends on strict adherence to a lengthy drug regimen which includes isoniazid (INH), rifampin, ethambutol, pyrazinamide, and streptomycin (or some combination of these)

An endocrinologist is providing care for a 30-year-old male who has lived with the effects of increased levels of growth hormone (GH). Which of the following teaching points about the client's future health risks is most accurate?

"When your pituitary gland is enlarged, there's a real risk that you'll develop some sight deficiencies."

A client with Graves' disease has opthalmopathy and asks the nurse if the eyes will stay like this forever. What is the best response by the nurse?

"With treatment of the hyperthyroid state, the opthalmopathy usually tends to stabilize."

A 51 year-old female client who is two days post-operative in a surgical unit of a hospital is at risk of developing atelectasis as a result of being largely immobile. Which of the following teaching points by her nurse is most appropriate?

"You should breath deeply and cough to help your lungs expand as much as possible while you're in bed."

A 51 year-old female client who is two days post-operative in a surgical unit of a hospital is at risk of developing atelectasis as a result of being largely immobile. Which of the following teaching points by her nurse is most appropriate? a) "Being in bed increases the risk of fluid accumulating between your lungs and their lining, so it's important for you to change positions often." b) "I'll proscribe bronchodilator medications that will help open up your airways and allow more oxygen in." c) "You should breath deeply and cough to help your lungs expand as much as possible while you're in bed." d) "Make sure that you stay hydrated and walk as soon as possible to avoid our having to insert a chest tube."

"You should breath deeply and cough to help your lungs expand as much as possible while you're in bed." Explanation: Atelectasis is characterized by incomplete lung expansion, and can often be prevented by deep breathing and coughing. Pleural effusion, not atelectasis, is associated with fluid accumulation between the lungs and their lining and neither chest tube insertion nor bronchodilators are common treatments for atelectasis.

A 51-year-old female client who is 2 days postoperative in a surgical unit of a hospital is at risk of developing atelectasis as a result of being largely immobile. Which of the following teaching points by her nurse is most appropriate?

"You should breathe deeply and cough to help your lungs expand as much as possible while you're in bed."

A 34 year-old man has been taking up to 2400 mg of ibuprofen per day following a motor vehicle several months ago and consequent chronic pain. He has recently been diagnosed with chronic analgesic nephritis as a result of his high analgesic intake. The man is surprised at the diagnosis, stating, "I thought that taking too many drugs hurt your liver if anything, not your kidneys." What is the most appropriate response to the man's statement? a) "Your liver does perform most of the detoxifying in your body, but your kidneys can perform this role if the liver is unable to." b) "Your kidneys are vulnerable to damage because of how much blood flows through them and the fact that they break down many drugs." c) "High drug intake can cause your kidneys to be very vulnerable to infections, which is likely what happened in your case." d) "It is very rare for someone as young as yourself to have kidney damage like this; usually only older people are vulnerable to kidney damage from drugs."

"Your kidneys are vulnerable to damage because of how much blood flows through them and the fact that they break down many drugs." High flow and pressure combined with the metabolic transformative of drugs makes the kidneys vulnerable to drug toxicity. They do not play a backup role to the liver in the metabolism of drugs, and while structural and functional damage may occur with drug overuse, infection is a less likely consequence. The elderly are particularly vulnerable to drug-related nephritis and nephropathies, but it would be incorrect to conclude that younger people are thus immune.

A female patient presented to her primary care physician with classic signs/symptoms of Cushing syndrome. Upon testing, it was discovered that the patient had vaginal small cell carcinoma. How can the healthcare providers explain her Cushing syndrome signs/symptoms to this patient?

"Your tumor in your vagina is secreting a hormone called adenocorticotropic hormone (ACTH) which is responsible for these signs/symptoms."

An adult client with a history of worsening respiratory symptoms has presented for care. Which assessment question will best allow the clinician to address the possibility of chronic bronchitis? -"Do you have a family history of lung disease?" -"Have you ever been immunized against pneumococcal pneumonia?" -"Do you tend to have a cough even when you don't feel sick?" -"Do you know if you had respiratory syncytial virus as a child?"

-"Do you tend to have a cough even when you don't feel sick?"

When educating a college student who lives in a crowded apartment and was recently diagnosed with tuberculosis, the school nurse will emphasize which treatment plan? -"Once your fever goes away you can stop taking the streptomycin injection." -"If isoniazid makes you nauseous, we can substitute something milder." -"To destroy this bacterium, you must strictly adhere to a long-term drug regimen of several medications." -"You will have to wear a N95 mask while on campus at all times."

-"To destroy this bacterium, you must strictly adhere to a long-term drug regimen of several medications."

A 51 year-old female client who is two days post-operative in a surgical unit of a hospital is at risk of developing atelectasis as a result of being largely immobile. Which of the following teaching points by her nurse is most appropriate? -"Being in bed increases the risk of fluid accumulating between your lungs and their lining, so it's important for you to change positions often." -"You should breathe deeply and cough to help your lungs expand as much as possible while you're in bed." -"Make sure that you stay hydrated and walk as soon as possible to avoid our having to insert a chest tube." -"I'll proscribe bronchodilator medications that will help open up your airways and allow more oxygen in."

-"You should breathe deeply and cough to help your lungs expand as much as possible while you're in bed."

Upon admission to the ICU, a client with a history of cor pulmonale will likely be exhibiting which clinical manifestations of right-sided heart failure? Select all that apply. -Fine crackles throughout both lung fields. -+4 pitting edema in lower extremities. -Expectorating copious amounts of frothy, pink sputum. -Jugular vein distension. -Altered level of consciousness.

-+4 pitting edema in lower extremities. -Jugular vein distension. -Altered level of consciousness.

Oxygen has been prescribed for a client with chronic obstructive pulmonary disease (COPD). Select the most appropriate treatment for the client. -1 to 2 L/min via nasal cannula -4 to 6 L/min via nonrebreather mask -10 L/min via nasal cannula -5 L/min via venture mask

-1 to 2 L/min via nasal cannula

In hypovolemic shock, renal perfusion and urinary output decline. The nurse will monitor urinary output and knows that output below which of the following levels indicates inadequate renal perfusion? -80—60 mL/hour -60—40 mL/hour -40—20 mL/hour -20 mL/hour

-20 mL/hour

At an influenza vaccination clinic, a nurse is screening clients who are requesting the vaccine. The nurse determines that which of the following clients is unable to receive the vaccine? -A 20-year-old healthy female reporting an allergy to eggs -An 82-year-old male with uncontrolled diabetes -A 6-year-old male with a history of asthma -A 64-year-old female reporting a latex allergy

-A 20-year-old healthy female reporting an allergy to eggs

Which of the following neurological patient's is most likely to have abnormalities in breathing regulation? -A 23-year-old male who has an injury to his frontal lobe following a sports injury -A 45-year-old female with a spinal cord injury at C7 following a motor vehicle accident -A 34-year-old male with damage to his upper and lower pons following a blow to the back of the head. -A 66-year-old male with temporal lobe infarcts secondary to a stroke.

-A 34-year-old male with damage to his upper and lower pons following a blow to the back of the head.

Which client most likely faces the highest risk of developing secondary pulmonary hypertension? -A client with COPD and a 35 pack-year smoking history. -An older adult client who has been hospitalized for the treatment of community-acquired pneumonia. -A client with asthma who uses her inhaled bronchodilator more often than prescribed . -A client who suffered a thermal injury to his lungs in a fire.

-A client with COPD and a 35 pack-year smoking history.

A client recovered from influenza two days ago and informs the nurse that she is feeling better but now has a fever, chills, pain when breathing, and a productive cough. What complication does the nurse anticipate the client will be treated for? -A secondary bacterial pneumonia -A relapse of the flu -Reye syndrome -Tuberculosis

-A secondary bacterial pneumonia

A nurse on a medical unit is providing care for a 37-year-old female patient who has a diagnosis of Graves disease. Which of the following treatments would the nurse most likely anticipate providing for the client? -Adrenergic-blocking medications to reduce sympathetic nervous stimulation Administration of levothyroxine to supplement thyroid function Calcium channel blocking medications to reduce heart rate and cardiac risks Administration of somatostatin analogs to inhibit GH production

-Adrenergic-blocking medications to reduce sympathetic nervous stimulation

Which physiologic dysfunction is the most common cause of hypercapnia? -Alterations in respiratory rate -Alterations in carbon dioxide production -Disturbances in alveolar compliance -Abnormalities in the function of the chest wall

-Alterations in respiratory rate

The nurse is caring for a client who is now 2 days post near-drowning. The focused assessment would involve which of the following areas of the lung involved in gas exchange? -Trachea -Alveoli -Bronchioles -Bronchus

-Alveoli

A client is suffering from severe gastroesophageal reflux disease (GERD) and has been admitted to the hospital with a diagnosis of pneumonia. Which of the following would be the most likely cause for the development of pneumonia? -Recent exposure to TB -Aspiration -Pertussis -Ineffective cough reflex

-Aspiration

An elderly client recently had a cerebrovascular accident that resulted in dysphagia. Which of the following is the nurse's greatest concern while feeding this client? -Aspiration -Accidental biting of cheek -Failure to thrive -Pocketing of food in cheeks

-Aspiration

A client has sudden severe dyspnea, fear, asymmetrical chest movement and decreased lung sounds on the right side. Which intervention is most appropriate? -Administer morphine sulfate (Morphine) to reduce pain and increase contractility. -Assist to high-Fowler's position and prepare for chest tube insertion. -Position HOB at 45 deg. and prepare for intubation to improve respiratory status. -Position limbs above heart for venous return and give dobutamine (Dobutrex) IV.

-Assist to high-Fowler's position and prepare for chest tube insertion.

A patient with a 25-year history of smoking is diagnosed with emphysema. Physical assessment reveals an increased anterior-posterior chest diameter. Which of the following terms should the nurse use to document this finding? -Barrel chest -Pink puffer -Blue bloater -Pneumothorax

-Barrel chest

The nurse determines that the client has clubbing of the fingertips. Which is the best intervention? -Call the health care provider. -Assess peripheral capillary refill. -Check the patient's O2 saturation level. -Monitor the client's heart rate.

-Check the patient's O2 saturation level.

A patient is diagnosed with pulmonary embolism. Which of the following symptoms would most likely be present? -Chest pain and dyspnea -Shallow respirations and wheezing -Left arm pain and diaphoresis -Cough and crackles

-Chest pain and dyspnea

Which condition places the client at risk for hypercapnia if given high-flow oxygen as treatment for a ventilation perfusion mismatch? -Pulmonary edema from heart failure -Closed head injury from a fall -Chronic bronchitis from smoking -Pneumothorax from rib fractures

-Chronic bronchitis from smoking Clients can develop hypoxemia when the rates of ventilation and perfusion do not match. If the cause is reduced cardiac output or pulmonary edema, increasing oxygen concentration can improve the outcome. If the condition is chronic obstructive pulmonary disease (COPD), the high-flow oxygen may reduce the hypoxic drive and cause hypercapnia.

A client has developed chronic hypoxia and has developed pulmonary hypertension (HTN). The nurse recognizes that the most likely cause of pulmonary hypertension would be: -Constant dilation of the pulmonary vessels in response to hypoxia -Constriction of the pulmonary vessels in response to hypoxemia -Hardening of the pulmonary vessels due to increased fat deposits -Decreased vascular resistance in the pulmonary vessels

-Constriction of the pulmonary vessels in response to hypoxemia

A client with primary lung disease has developed right heart failure. The health care provider would document this as: -Adult respiratory distress syndrome -Primary hypertension -Cor pulmonale -Cardiac tamponade

-Cor pulmonale

During the admission interview the client, who is admitted with bacterial pneumonia, reveals a 20 pack per year smoking history. The nurse relates the possible cause of this pneumonia to the decreased defense of the pulmonary system caused by cigarette smoking. Smoking affects the pulmonary defense system by which of these? -Buildup of nicotine on the cilia -Damage or destruction of cilia -Vasodilation of blood vessels -Increased development of atherosclerosis

-Damage or destruction of cilia

There can be many reasons for a client to present with hypoxemia. For a client's PO2 to fall, a respiratory disease is usually involved. Often, clients have involvement from more than one mechanism. Which of the following will result in hypoxemia? (Select all that apply.) -Decreased oxygen in air -Inadequate circulation through pulmonary capillaries -Hyperventilation -Disease in respiratory system -Mismatched pulmonary perfusion -Dysfunction of neurologic system

-Decreased oxygen in air -Disease in respiratory system -Dysfunction of neurologic system

The nurse caring for a client with bilateral rib fractures explains to the client that the pain causes him to take more shallow respirations which will contribute to feeling SOB due to a -Decreased tidal volume. -Increased vital capacity. -Decreased residual volume. -Increased total lung capacity.

-Decreased tidal volume.

A patient arrives in the ED after an automobile accident. Which of the following clinical manifestations leads the nurse to suspect a pneumothorax? Select all that apply. -Diminished breath sounds over painful chest area. -ABG pH level of 7.38 -Asymmetrical chest movements, esp. on inspiration. -Respiratory rate 34. -Pulse oximetry 98%.

-Diminished breath sounds over painful chest area. -Asymmetrical chest movements, esp. on inspiration. -Respiratory rate 34.

The pulmonary rehabilitation specialist is educating medical students on a respiratory disease process that causes a severe compromise in exhalation due to air trapping. Air trapping is caused by the loss of elastic recoil, especially in the alveoli, which occurs after overstretching in which of the following diseases? -Chronic obstructive pulmonary disease (COPD) -Asthma -Emphysema -Chronic bronchitis

-Emphysema

A man with chronic alcoholism is diagnosed is acute respiratory distress syndrome (ARDS) due to alcohol abuse. Assessment findings include: diffuse bilateral infiltrates of the lung tissue and marked hypoxemia. What is the primary physiologic change that results in his ARDS? -Impaired mucus clearance -Air in the pleural space -Epithelial injury with increased permeability of the alveolar--capillary membrane -Narrowed bronchioles with severe mucus accumulation and increased smooth muscle contraction

-Epithelial injury with increased permeability of the alveolar--capillary membrane

The health care provider suspects a newly admitted client may have a hemothorax. The client most likely experienced: -Fractured or dislocated ribs -Congestive heart failure -Pneumonia -Renal failure

-Fractured or dislocated ribs

An asthmatic patient comes to the emergency department very anxious due to increasing shortness of breath. Physical assessment reveals tachypnea, and an arterial blood gas shows decreased carbon dioxide levels and hypoxemia. Which of the following is the most likely cause? -Hyperventilation -Tachycardia -Apnea -Cyanosis

-Hyperventilation

A 20-year-old college student has a pelvic fracture and a severed leg from a motorcycle accident. She lost several units of blood. When the student arrived in the emergency department, her blood pressure was very low, her pulse was high, and her skin was pale. The nurse knows that this patient has developed which of the following types of shock? -Cardiogenic -Hypovolemic -Distributive -Obstructive

-Hypovolemic

The nurse is caring for a client diagnosed with pneumonia. The client's arterial blood gas results identify decreased level oxygen in the arterial blood and a decreased in the partial pressure of oxygen. The nurse would interpret this as: -Hypoxia -Hypercapnia -Hypoxemia -Atelectasis

-Hypoxemia

A patient is brought into the emergency department with severe crushing injuries to the chest wall and signs of respiratory failure following a motor vehicle accident. Which of the following laboratory values would be expected? -Cool moist skin -Increase in PCO2 -Hyperventilation -Metabolic alkalosis

-Increase in PCO2

Ventilation is driven by which alteration in arterial blood? -Increased PCO2 -Decreased PO2 -Increased HCO3 -Increased pH

-Increased PCO2

A child has developed respiratory stridor and is displaying a crowing sound. The parents ask the nurse what is causing this sound. The best response would be: -Pulling in of the soft tissue surrounding the cartilaginous and bony thorax -Nasal obstruction and inhalation occurring through the mouth rather than the nares -Increased turbulence of air moving through the obstructed airways -The collapse of intrathoracic airways because of air trapping

-Increased turbulence of air moving through the obstructed airways

A nurse is instructing a class for people with newly diagnosed asthma to encourage healthy life style choices. The nurse explains that stimulation of certain lung receptors with things such as smoke, cigarette smoke, inhaled dust, or cold air can lead to constriction of the conducting airways resulting in rapid, shallow breathing. How does the nurse identify these receptors? -J receptors -Noxious receptors -Stretch receptors -Irritant receptors

-Irritant receptors

A 3-year-old child has been admitted to the pediatric unit for the treatment of croup. The nurse should include which action in the child's plan of care? -Providing cool, moist air for the child to breathe -Establishing IV access and administering sulfa antibiotics as ordered -Administering antiviral medications such as zanamivir and oseltamivir as ordered -Providing a low-stimulation environment to relieve bronchospasm

-Providing cool, moist air for the child to breathe

Cystic fibrosis (CF) is an autosomal recessive disorder involving the secretion of fluids in specific exocrine glands. The genetic defect in CF inclines a person to chronic respiratory infections from a small group of organisms. Which organisms create chronic infection in a child with cystic fibrosis? -Pseudomonas aeruginosa and Escherichia coli -Staphylococcus aureus and hepatitis C -Haemophilus influenzae and influenza A -Pseudomonas aeruginosa and Staphylococcus aureus

-Pseudomonas aeruginosa and Staphylococcus aureus

Reviewing pathology for an exam on pulmonary vasculature, the nursing student states that blood enters the right side of the heart via vena cava's, then to the right atrium, right ventricle, and then which vessel carries the deoxygenated blood into the pulmonary system? -Pulmonary capillaries. -Pulmonary artery. -Pulmonary vein. -Ductus arteriosus.

-Pulmonary artery.

A child is brought to the emergency department struggling to breathe with a prolonged bronchospasm and severe hypoxemia. Assessment revealed the use of accessory muscles, a weak cough, audible wheezing sound, moist skin, and tachycardia. Which of the following is the most likely diagnosis? -Severe asthma attack -Chronic obstructive pulmonary disease -Pulmonary embolism -Cystic fibrosis

-Severe asthma attack

A client has developed bacterial pneumonia and is admitted to the hospital. The nurse obtains sputum cultures upon admission. What bacteria does the nurse anticipate finding when the results are complete? -Pseudomonas aeruginosa -Staphylococcus aeureus -Streptococcus pneumoniae -Escherichia coli

-Streptococcus pneumoniae

Factors Affecting Alveolar-Capillary Gas Exchange (4)

-Surface area available for diffusion -Thickness of the alveolar-capacity membrane -Partial pressure of alveolar gases -Solubility and molecular weight of the gas

A patient's oxygen-hemoglobin dissociation curve is represented by a shift to the right. The nurse recognizes this as: -The affinity of hemoglobin for oxygen is decreased. -That the affinity of hemoglobin for oxygen is increased. -It occurs in situations associated with a decrease in tissue metabolism. -PO2 available to the tissues at any given level of hemoglobin saturation is decreased.

-The affinity of hemoglobin for oxygen is decreased. A shift to the right indicates that the affinity of hemoglobin for oxygen is decreased and the PO2 that is available to the tissues at any given level of hemoglobin saturation is increased. A shift to the left indicates that the affinity of hemoglobin for oxygen is increased and the PO2 that is available to the tissues at any given level of hemoglobin saturation is decreased. It occurs in situations associated with a decrease in tissue metabolism, such as alkalosis, decreased body temperature, and decreased PCO2 levels.

The nurse is caring for the following clients. Select the client at highest risk for the development of atelectasis. -The client who is mobile within 24 hours after abdominal surgery -The client with a lower leg cast who changes position every 2 hours -The client diagnosed with pneumonia who performs frequent coughing and deep breathing exercises -The client who is postop total knee replacement and receiving client-controlled analgesia

-The client who is postop total knee replacement and receiving client-controlled analgesia

The parents of a child with cystic fibrosis ask the nurse to explain the sweat test performed on their newborn. How should the nurse respond? -The sweat test evaluates the infant's ability to sweat in response to heat. -The sweat test measures the concentration of salt in the infant's sweat. -Abnormal pancreatic enzymes may cause the infant to sweat more. -Cystic fibrosis decreases the amount of salt in the infant's sweat.

-The sweat test measures the concentration of salt in the infant's sweat.

A patient is diagnosed with right-sided heart failure. The nurse knows that a frequent sign of this type of failure is peripheral edema, evidenced by which of the following? -Weight gain -Copious urination -Shortness of breath -Decreased blood pressure

-Weight gain

A patient is admitted with pneumoconiosis. His history most likely reveals which of the following? -Congestive heart failure -Work in a coal mine for 20 years -Tuberculosis -Twenty pack-years of smoking

-Work in a coal mine for 20 years

The parents of a child diagnosed with cystic fibrosis (CF) ask about the risk of any future children having the condition. How should the nurse respond? -CF is autosomal dominant, so you have a 50% risk in another child. -You have a 25% chance that your next child will have CF. -Since the male carries the CF gene, you might consider a sperm donor. -Now that you have one child with CF, the rest will also have it.

-You have a 25% chance that your next child will have CF.

A mother in premature labor asks the nurse why her doctor has prescribed corticosteroids. The nurse's response is based on the fact that: -corticosteroids can perform anti-inflammatory acts that may prevent infections. -cortisol can accelerate maturation of type II cells and stimulate the formation of surfactant. -corticosteroids cause blood glucose levels to rise, thus preventing hypoglycemia in premature infants. -cortisol administration will enhance the secretion of insulin, which is needed for surfactant production.

-cortisol can accelerate maturation of type II cells and stimulate the formation of surfactant.

The nurse is caring for an infant with a large ventricular septal defect, also called a hole in the heart, which is a congenital heart defect causing a right to left shunt. The nurse illustrates for the parents how this compromises their child's ability to deliver oxygenated blood to the tissues causing -cyanosis secondary to an anatomic shunt. -jaundice related to an anatomic shunt. -a physiologic shunt and chest pain. -a physiologic shunt and pallor.

-cyanosis secondary to an anatomic shunt.

The nurse is explaining to the parents of a 23-week premature infant the reason their baby needs to be on mechanical ventilation. The education is successful when the parents state that they understand their baby was born before the type II alveolar cells could mature which has caused a...and what treatment is given -deficiency of alveoli; albuterol -deficiency of type I alveolar cells; epinephrine -deficiency of surfactant; glucocorticoid -deficiency of conducting airways; cortisol

-deficiency of surfactant;glucocorticoid

A distressed, confused client is admitted to the hospital ER with a penetrating right chest stab wound. The nurse assesses the patient and notes a lack of breath sounds in the right lung due. The nurse knows this is likely the result of: -hyperinflation of the lung. -decreased intraalveolar pressure. -loss of intrapleural pressure. -negative intrapleural pressure.

-loss of intrapleural pressure.

When educating a client with possible glucocorticoid dysfunction, the nurse will explain that the CRH controls the release of ACTH. The best time to perform the blood test to measure peak ACTH levels would be:

06:00 to 08:00 AM

A 1-year-old baby boy with renal dysplasia risks end-stage renal disease unless intervention occurs. Which of the following treatment options is his care team most likely to reject?

A) Dietary restriction plus erythropoietin

Oxygen has been prescribed for a client with chronic obstructive pulmonary disease (COPD). Select the most appropriate treatment for the client.

1 to 2 L/min via nasal cannula

Oxygen has been prescribed for a client with chronic obstructive pulmonary disease (COPD). Select the most appropriate treatment for the client. a) 4 to 6 L/min via nonrebreather mask b) 1 to 2 L/min via nasal cannula c) 10 L/min via nasal cannula d) 5 L/min via venture mask

1 to 2 L/min via nasal cannula Explanation: The goal of oxygen delivery for a client with COPD is to keep the PO2 at about 60 mm Hg or less, which can be accomplished with delivery of 1 to 2 L/min of oxygen via nasal cannula, which results in a PO2 at 55 to 65 mm Hg. All the other options would increase the PO2 above 60 mm Hg, which tends to depress the hypoxic stimulus for ventilation and often leads to hypoventilation and carbon dioxide retention.

A patient with small cell lung cancer (SCLC) has developed a paraneoplastic syndrome called Cushing syndrome. Based on this new complication, the nurse will likely assess which of the following clinical manifestations of Cushing syndrome? 1. Weight gain, moon face, buffalo hump, and purple striae on the abdomen 2. bilateral edema in the arms, swollen face, and protruding eyes 3. severe bone/joint pain, nausea/vomiting, and polyuria 4. Tetany, new-onset seizure activity, emotional lability, and extrapyramidal symptoms

1. Weight gain, moon face, buffalo hump, and purple striae on the abdomen

When educating a student who lives in a crowded apartment and diagnosed with TB, the college school nurse will emphasize, 1. "once your fever goes away, you can stop taking the streptomycin injection" 2. " To destroy this bacterium, you must strictly adhere to a long-term drug regimen." 3. "You will have to wear an N95 mask while on campus at all times"

2. " To destroy this bacterium, you must strictly adhere to a long-term drug regimen."

A 22 year old female with a history of intermittent flank pain, repeated UTIs and Hematuria has been diagnosed with an autosomal dom. Polycystic Kidney disease (ADPKD). Which of the following phenomena has most likely contributed to the development of this diagnosis? 1. UTIs coupled with an impaired immune response have cause her ADPKD 2. She has inherited a tenancy for epithelial cells in her tubules to proliferate inappropriately 3. Severe hypertension and portal hypertension are likely precursors 4. She has inherited undersized kidneys that are prone to calculi formation

2. She has inherited a tenancy for epithelial cells in her tubules to proliferate inappropriately

Because they strengthen the pelvic floor muscles, Kegel exercises are most likely to help: 1. Overflow incontinence 2. Stress incontinence 3. Urge incontinence 4. mixed incontinence

2. Stress incontinence

Pneumonia that occurs 48 hours or more after admission to the hospital is considered 1. Community- acquired pneumonia 2. hospital- acquired pneumonia 3. viral pneumonia 4. immunocompromised pneumonia

2. hospital- acquired pneumonia

The neonatal ICU nurse is aware that type II alveolar cells produce surfactant, and they usually develop at how many weeks of gestation? 17 to 18 weeks 19 to 20 weeks 24 to 28 weeks 34 to 38 weeks

24 to 28 weeks

When explaining about the passage of urine to a group of nursing students, the clinical nurse asks them which muscle is primarily responsible for micturition? Their correct reply is the 1. Trigone 2. Urinary vesicle 3. Detrusor 4. External sphincter

3. Detrusor

A short, nonsmoking 44 yo male presents to the emergency room with left-sided chest pain and a cough. He states that the pain started abruptly and worsens with deep breathing and coughing. He denies recent injury. Assessment includes shallow respirations with a rate of 36, normal breath sounds, and no cyanosis. Which condition is most likely? 1. Spontaneous pneumothorax 2.MI 3. Pleuritis related to infection 4. Obstructive atelectasis

3. Pleuritis related to infection

The nurse is working in a pediatric clinic. Which of the following children would the nurse recognize as having isosexual precocious puberty?

A 5-year-old African-American female with developing breasts and pubic hair

A 51 yo female client who is in 2 days post-op in a surgical unit of a hospital is at risk for developing atelectasis as a result of being largely immobile. Which of the following teaching points by her nurse is most appropriate? 1. Being in bed increases the risk of fluid accumulating between your lungs and their lining, so it is important for you to change positions often 2. make sure you stay hydrated and walk as soon as possible to avoid us having to insert a chest tube 3. I'll proscribe bronchodilator medications that will help open up your airways and allow more oxygen in 4. You should breathe deeply and cough to help your lungs expand as much as possible while you're in bed

4. You should breath deeply and cough to help your lungs expand as much as possible while you're in bed

Which of the following patients would have a very poor response related to tissue regeneration of his or her injured area?

A 54-year-old male who had a massive MI 4 days ago and came to the ED today for treatment

The nurse is caring for four clients. Select the client at risk for the development of a pulmonary embolism.

A 62-year-old male postop repair of a fractured femur

Which of the following clients is most likely to have low bone density? A 70-year-old woman with increased numbers of osteoblasts. A 78-year-old female whose osteoclast function is inhibited. A 65-year-old male who is noted to have osteopenia. A 68-year-old male who takes vitamin D supplements.

A 65-year-old male who is noted to have osteopenia.

Which of the following clients of an oncologist is likely to have the poorest prognosis?

A 69-year-old man who has been diagnosed with stage IV malignant melanoma

After receiving change-of-shift report about the following four patients, which patient should the nurse assess first?

70-year-old returning from PACU following partial thyroidectomy who is extremely agitated, has an irregular pulse rate of 134, and an elevated temperature of 103.20 F

The nurse is reviewing the assessment data of four clients. Select the client who would be diagnosed with precocious puberty.

A 7-year-old female with early menarche

4. A 73-year-old man presents to his family physician with complaints of recent urinary hesitation and is eventually diagnosed with benign prostatic hyperplasia (BPH). Which of the following clinical consequences would his care provider expect prior to the resolution of his health problem? A) Hydroureter and pain B) Development of renal calculi and renal cysts C) Unilateral hydronephrosis and pain D) Development of glomerulonephritis or nephrotic syndrome

A

9. A 4-year-old boy who has been deaf since birth and has bilateral cataracts has been brought to the emergency department by his mother because she noticed blood in the toilet after he last voided. Urinalysis confirms heavy microscopic hematuria as well as proteinuria. What will the health care team's initial differential diagnosis most likely be? A) Alport syndrome B) Systemic lupus erythematosus glomerulonephritis C) Henoch-Schönlein purpura nephritis D) Immunoglobulin A nephropathy

A

Which of the following clinical findings among older adults is most unlikely to warrant further investigation and possible treatment?

A 78-year-old female's GFR has been steadily declining over several years.

A farmer is admitted with hypersensitivity pneumonitis. Which factor could have caused his condition?

Mouldy grain dust

Which of the following clients would be considered to have a significant risk of developing the prerenal form of acute renal failure? Select all that apply. A 22-year-old male who has lost large amounts of blood following a workplace injury A 41-year-old female who is admitted for intravenous antibiotic treatment of pyelonephritis A 79-year-old male with diagnoses of poorly controlled diabetes mellitus and heart failure A 20-year-old male who is admitted for treatment of an overdose of a nephrotoxic drug A 68-year-old male with a diagnosis of benign prostatic hyperplasia (BPH) An 80-year-old female who has been admitted for the treatment of dehydration and malnutrition

A 22-year-old male who has lost large amounts of blood following a workplace injury A 79-year-old male with diagnoses of poorly controlled diabetes mellitus and heart failure An 80-year-old female who has been admitted for the treatment of dehydration and malnutrition

A nurse has noted the high incidence of urinary tract obstructions of a variety of etiologies. Which of the following individuals are at risk of developing urinary obstructions? Select all that apply. A 43-year-old male with an acid-base imbalance secondary to malnutrition A 29-year-old female, pregnant for the first time A 69-year-old female with anemia secondary to insufficient erythropoietin production A 70-year-old male with benign prostatic hyperplasia (BPH) A 58-year-old male with renal calculi A 28-year-old male with a neurogenic bladder secondary to spinal cord injury

A 29-year-old female, pregnant for the first time A 70-year-old male with benign prostatic hyperplasia (BPH) A 58-year-old male with renal calculi A 28-year-old male with a neurogenic bladder secondary to spinal cord injury

Which of the following individuals is at the highest risk of developing a urinary tract infection (UTI)?

A 30-year-old woman with poorly controlled diabetes mellitus

Which of the following patients of a primary care physician would not require extra screening for cancer?

A 38-year-old female with Down syndrome and congenital scoliosis

Of the following situations, which one would be an example of a maladaptive cellular change?

A 44-year-old male with a 60-pack-per year smoking history who was diagnosed with a histological grade 3 lung cancer

Which of the following clients is at a high risk for developing dilated cardiomyopathy?

A 44-year-old noncompliant female who forgets to take her hypertensive medications

The nurse is caring for several clients who are taking growth hormone replacement therapy. Which client does the nurse recognize is most at risk for side effects?

A 45-year-old obese client with an elevated serum IGF-1 concentration

Which client most likely faces the highest risk of developing secondary pulmonary hypertension?

A client with COPD and a 35 pack-year smoking history.

A patient is admitted for a relapse for sarcoidosis. Knowing this is usually caused by an inflammatory process, the nurse can anticipate administering: a) An albuterol inhaler. b) A bronchodilator. c) Aspirin. d) A corticosteroid.

A corticosteroid. Explanation: Treatment is directed at interrupting the granulomatous inflammatory process that is characteristic of the disease and managing the associated complications. When treatment is indicated, corticosteroid drugs are used. Bronchodilators may be used if there is wheezing, but this is not a normal medication for this disease. Aspirin is a blood thinner. Albuterol is a short-term bronchodilator for acute asthma

A patient is admitted for a relapse for sarcoidosis. Knowing this is usually caused by an inflammatory process, the nurse can anticipate administering: a) An albuterol inhaler. b) A bronchodilator. c) A corticosteroid. d) Aspirin.

A corticosteroid. Explanation: Treatment is directed at interrupting the granulomatous inflammatory process that is characteristic of the disease and managing the associated complications. When treatment is indicated, corticosteroid drugs are used. Bronchodilators may be used if there is wheezing, but this is not a normal medication for this disease. Aspirin is a blood thinner. Albuterol is a short-term bronchodilator for acute asthma

A client tells the nurse that he is concerned he may be developing chronic bronchitis and asks how the diagnosis is made. The most appropriate information for the nurse to provide would be:

A diagnosis of chronic bronchitis requires a history of a chronic productive cough that has persisted for at least 3 consecutive months in at least 2 consecutive years.

A client is diagnosed with hyperthyroidism and is exhibiting weight loss, diarrhea, and tachycardia. What does the nurse understand that these clinical manifestations are related to?

A hypermetabolic state

Which of the following clients is at risk for developing acute respiratory failure? a) A toddler in daycare who has been sharing toys with peers before the staff could sanitize properly b) An elderly female living in senior housing who has been exposed to a "cold" while her grandchildren visited c) A middle-aged male diagnosed with amyotrophic lateral sclerosis (ALS) who has pneumonia with low O2 saturation d) A teenager in a high school that has had an increase in student absences due to an outbreak of strep throat

A middle-aged male diagnosed with amyotrophic lateral sclerosis (ALS) who has pneumonia with low O2 saturation Explanation: Acute respiratory failure may occur in previously healthy persons as the result of acute disease or trauma involving the respiratory system, or it may develop in the course of a chronic neuromuscular or lung disease. ALS is a neurodegenerative disease with various causes. It is characterized by rapidly progressive weakness due to muscle atrophy and muscle spasticity and difficulty in speaking (dysarthria), swallowing (dysphagia), and breathing (dyspnea). A common cold, strep throat, or sharing toys in daycare will not necessarily place one at risk for respiratory failure

Which of the following changes in an 86-year-old male's skin would necessitate further examination and possible medical treatment?

A new mole-like growth on his forearm

Which of the following clients is at risk for developing a preventable disorder related to prolonged immobility? a) A middle-aged adult male diagnosed with bronchitis related to chronic smoking b) A postsurgical client who is refusing to get out of bed and walk and will not wear those "uncomfortable elastic stocking" c) A sleep apnea client related to a history of smoking who utilizes a C-PAP machine every night at bedtime to maintain airway d) A young adult female diagnosed with sarcoidosis requiring corticosteroids to return her to remission

A postsurgical client who is refusing to get out of bed and walk and will not wear those "uncomfortable elastic stocking" Explanation: A lack of mobility can result in secondary atelectasis (through incomplete lung expansion) and pulmonary embolism (from deep vein thrombosis). This is not the case with the other listed disorders of ventilation and gas exchange.

A young man and his friend started a business sandblasting and refinishing wrought iron. Following a rapid onset of respiratory symptoms, the client was diagnosed with silicosis. What aspect of silicosis should underlie the nurse's care planning?

A quick decline in lung function is likely, and the client is likely to face an early death.

A client is experiencing muscle atrophy following 2 weeks in traction after a motor vehicle accident. Which of the following factors has most likely contributed to the atrophy of the client's muscle cells?

A reduction of skeletal muscle use secondary to the traction treatment

The mother of 6-year-old male and female fraternal twins has brought her son to see a pediatrician because he is nearly 4 inches shorter than his sister. Which of the following phenomena would the physician most likely suspect as contributing factor to the boy's short stature?

A shortage of hypothalamic GHRH production

Of the following list of patients, who would likely benefit the most from hyperbaric oxygen therapy

A trauma patient who developed Clostridium spp., an anaerobic bacterial infection in his femur

A 28-year-old male who is 6.11. tall has a diagnosis of acromegaly. The man is explaining to a curious but sympathetic coworker exactly what accounts for his extraordinary height. Which of the following explanations demonstrates a sound understanding of his health problem? "My pituitary gland produced a much higher than normal amount of growth hormone when I was a child." "A tumor in my brain threw off my hormone levels after I was finished adolescence." "My liver is malfunctioning and produces too many of the hormones that ultimately cause growth." "The high sugar levels that go along with my diabetes made my pituitary gland overproduce the hormones that cause you to grow."

A tumor in my brain threw off my hormone levels after I was finished adolescence."

A 41-year-old female with a family history of breast cancer has had a baseline mammogram. She states that she performs monthly self-breast exams but really has a hard time evaluating her lumps since she has numerous cysts. At her annual mammogram, the technician views a suspicious area and refers her to the radiologist. She asks the nurse in the office, "How can a lump appear so quickly?" The nurse's response is based on which of the following principles?

A tumor is undetectable until it has doubled 30 times and contains at least 1 billion cells.

The nurse should anticipate that a patient diagnosed with spastic bladder dysfunction may be prescribed which of the following medications that will help decrease detrusor muscle tone and increase bladder capacity? Select all that apply. A) Ditropan (Oxybutynin), an antimuscarinic drug B) Detrol LA (tolterodine tartrate), an antimuscarinic drug C) Uroxatral (alfuzosin), an α-adrenergic antagonist D) Flomax (tamsulosin), an α-blocker E) Bactrim (sulfamethoxazole and trimethoprim), antibiotics

A) Ditropan (Oxybutynin), an antimuscarinic drug B) Detrol LA (tolterodine tartrate), an antimuscarinic drug Antimuscarinic drugs, such as oxybutynin, tolterodine, and propantheline, decrease detrusor muscle tone and increase bladder capacity in people with spastic bladder dysfunction. Answer choices C and D are medications prescribed for males with BPH.

A 68-year-old woman with a new onset of vascular dementia has recently begun retaining urine. Which of the following physiological phenomena would her care providers most realistically expect to currently occur as a result of her urinary retention? A) Hypertrophy of the bladder muscle and increased bladder wall thickness B) Decreased urine production and nitrogenous waste excretion by the kidneys C) Decompensation, bladder stretching, and high residual urine volume D) Overflow incontinence and loss of contraction power

A) Hypertrophy of the bladder muscle and increased bladder wall thickness

When educating the patient about possible treatments following surgery for bladder cancer, the nurse might include which of the following chemotherapy options? Select all that apply. A) Intravesical chemotherapy with doxorubicin (Adriamycin) B) Intravenous chemotherapy with at least three agents C) Bacillus Calmette-Guérin (BCG) vaccine D) Endocan, a tumor angiogenesis inhibitor

A) Intravesical chemotherapy with doxorubicin (Adriamycin) C) Bacillus Calmette-Guérin (BCG) vaccine D) Endocan, a tumor angiogenesis inhibitor No chemotherapeutic regimens for bladder cancer have been established. Instillation of chemotherapeutic drugs into the bladder is currently done using thiotepa, mitomycin C, and doxorubicin. BCG vaccine causes a significant reduction in the rate of relapse and prolongs relapse-free intervals in people with cancer in situ. Inhibitors of tumor angiogenesis and inhibitors of EGF drugs are proving effective with bladder cancer.

An 87-year-old male resident of an assisted living facility has been consistently continent of urine until the last several weeks. Which of the following actions by the care providers at the facility is the most likely priority? A) Performing a physical examination and history to determine the exact cause and character of the incontinence B) Providing client education focusing on the fact that occasional incontinence is a normal, age-related change C) Teaching the resident about protective pads, collection devices, and medications that may be effective D) Showing the resident the correct technique for exercises to improve bladder, sphincter, and pelvic floor tone

A) Performing a physical examination and history to determine the exact cause and character of the incontinence The priority in the treatment of incontinence in the elderly is an acknowledgement that it is not an inevitability and that the exact causes should and most often can be identified. This identification by way of history-taking and examination would supersede teaching about protective devices or exercises.

A nurse in a medical unit has noted that a client's potassium level is elevated at 6.1 mEq/L. The nurse has notified the physician, removed the banana from the client's lunch tray, and is performing a focused assessment. When questioned by the client for the rationale for these actions, which of the following explanations is most appropriate?

A) "Your potassium level is high, and so I need you let me know if you feel numbness, tingling, or weakness."

Which of the following clients would be considered to have a significant risk of developing the prerenal form of acute renal failure? Select all that apply.

A) A 22-year-old male who has lost large amounts of blood following a workplace injury c) A 79-year-old male with diagnoses of poorly controlled diabetes mellitus and heart failure f) An 80-year-old female who has been admitted for the treatment of dehydration and malnutrition

Which of the following individuals would be considered to be at risk for the development of edema? Select all that apply.

A) An 81-year-old man with right-sided heart failure and hypothyroidism C) A 34-year-old industrial worker who has suffered extensive burns in a job-related accident E) A 22-year-old female with hypoalbuminemia secondary to malnutrition and anorexia nervosa

6. A client with a diagnosis of chronic renal failure has orders for measurement of her serum electrolyte levels three times per week. Which of the following statements best captures the relationship between renal failure and sodium regulation? A) Clients with advanced renal failure are prone to hyponatremia because of impaired tubular reabsorption. B) Renal clients often require a sodium-restricted diet to minimize the excretion load on remaining nephrons. C) Clients with renal failure often maintain high sodium levels because of decreased excretion. D) Restricting sodium intake helps to preserve nephron function and has the additional benefit of lowering blood pressure.

A) Clients with advanced renal failure are prone to hyponatremia because of impaired tubular reabsorption.

13. A A1-year-old baby boy with renal dysplasia risks end-stage renal disease unless intervention occurs. Which of the following treatment options is his care team most likely to reject? A) Dietary restriction plus erythropoietin B) Continuous cyclic peritoneal dialysis C) Renal transplantation D) Continuous ambulatory peritoneal dialysis

A) Dietary restriction plus erythropoietin

Following the diagnosis of acute renal failure, the nurse knows that one of the earliest manifestations of residual tubular damage is which of the following lab/diagnostic results?

A) Elevated blood urea nitrogen (BUN)

Tumor necrosis factor-a and IL-1 are major cytokines that mediate inflammation. If the patient is developing a systemic response to an infection, the nurse will likely assess which of the following clinical manifestations? Select all that apply.

A) Elevated temperature C) Tachycardia E) Anorexia

A 9-year-old boy has been diagnosed with the nephrotic syndrome. Place the following stages in the development of his health problem in ascending order. Use all the options.

A) Hypoalbuminemia B) Increased glomerular membrane permeability C) Decreased colloidal osmotic pressure D) Proteins escape from the plasma to the glomerular filtrate E) Accumulation of fluid in the interstitial tissue (edema) B, D, A, C, E

A pediatric unit will be receiving an 8-day-old infant with a suspected congenital renal disorder. Which of the following renal abnormalities could be the possible cause? Select all that apply

A) One of the infant's kidneys may have failed to develop normally. B) The kidneys may be misshapen and have cysts present. C) The upper or lower poles of the two kidneys may be fused.

A patient who has had a prolonged period of nasogastric (NG) suctioning following colon surgery is experiencing electrolyte imbalances. The magnesium level is low (1.2 mg/dL). Knowing that magnesium deficiency occurs in conjunction with low calcium levels, the nurse should assess the patient for which of the following clinical manifestations of hypocalcaemia? Select all that apply.

A) Personality changes B) Hyperactive reflexes C) Increase in ventricular arrhythmias

9. Which of the following phenomena contributes to the difficulties with absorption, distribution, and elimination of drugs that are associated with kidney disease? A) Reductions in plasma proteins increase the amount of free drug and decrease the amount of protein-bound drug. B) Acute tubular necrosis is associated with impaired drug reabsorption through the tubular epithelium. C) Decreased retention by the kidneys often renders normal drug dosages ineffective. D) Dialysis removes active metabolites from circulation, minimizing therapeutic effect.

A) Reductions in plasma proteins increase the amount of free drug and decrease the amount of protein-bound drug.

A female client with suspected glomerular disease has been referred to a nephrologist. The nurse knows that which of the following clinical manifestations may be present with the diagnosis of acute nephritic syndrome? Select all that apply.

A) Sudden onset of hematuria B) Proteinuria E) Edema

Blood-borne cancerous cells have recently spread from a woman's primary tumor in her pancreas to her bones. Which of the following components of the woman's immune system are likely to be directly involved in the attempt to eradicate the potential metastasis

A) T lymphocytes B) Macrophages C) Natural killer (NK) cells D) B cells

15. A 56-year-old woman has been diagnosed with CKD. She first went to the doctor due to complications of hypertension. How are hypertension and CKD related? A) The mechanisms that produce hypertension in CKD include an increased vascular volume, elevation of peripheral vascular resistance, decreased levels of renal vasodilator prostaglandins, and increased activity of the renin-angiotensin system. B) The mechanisms that produce hypertension in CKD are due to decreases in vascular volume, the elevation of peripheral vascular resistance, increased levels of renal vasodilator prostaglandins, and decreased activity of the renin-angiotensin system. C) The mechanisms that produce hypertension in CKD are directly related to sporadic increases in the activity of the renin-angiotensin system. D) The mechanisms relate to increases in hydrostatic pressure on the renal vasculature causes inflammation and irreversible damage.

A) The mechanisms that produce hypertension in CKD include an increased vascular volume, elevation of peripheral vascular resistance, decreased levels of renal vasodilator prostaglandins, and increased activity of the renin-angiotensin system.

Which of the following clinical manifestations would lead the nurse to suspect the renal failure patient is developing uremia? Select all that apply.

A) Weakness and fatigue B) Lethargy and confusion C) Extreme itching

1. A pediatric unit will be receiving an 8-day-old infant with a suspected congenital renal disorder. Which of the following renal abnormalities could be the possible cause? Select all that apply. A) One of the infant's kidneys may have failed to develop normally. B) The kidneys may be misshapen and have cysts present. C) The upper or lower poles of the two kidneys may be fused. D) Renal cell carcinoma may be present. E) Urine-filled dilation of renal pelvis associated with atrophy of the kidney may be present.

A,B,C

7. A 9-year-old boy has been diagnosed with the nephrotic syndrome. Place the following stages in the development of his health problem in ascending order. Use all the options. A) Hypoalbuminemia B) Increased glomerular membrane permeability C) Decreased colloidal osmotic pressure D) Proteins escape from the plasma to the glomerular filtrate E) Accumulation of fluid in the interstitial tissue (edema)

A,B,C,D,E

6. A female client with suspected glomerular disease has been referred to a nephrologist. The nurse knows that which of the following clinical manifestations may be present with the diagnosis of acute nephritic syndrome? Select all that apply. A) Sudden onset of hematuria B) Proteinuria C) Flank pain D) Excess urine output E) Edema

A,B,E

5. A client has been diagnosed with having calcium oxalate kidney stones following intravenous pyelography. Which of the following teaching points about the treatment of the health problem are justifiable? Select all that apply. A) "You may need to cut out cocoa, chocolate, and some nuts from your diet." B) "It's important that you avoid high-calcium foods like milk, cheese, and yogurt." C) "We will come up with a plan to safely limit your fluid intake over the next few weeks." D) "Extracorporeal shock-wave lithotripsy treatment may be used to fragment larger stones." E) "Most likely your stones can be dissolved by medications over the next several days."

A,D

Which of the following statements about the use of angiotensin-converting enzyme inhibitor medications and autosomal recessive polycystic kidney disease (ARPKD) is accurate? a) ACE inhibitors should be used strictly in those clients who also have an underlying cardiac history. b) The use of ACE inhibitors will increase the vasopressin levels. c) The ACE inhibitors have been shown to shrink the size of the cysts inside the kidneys. d) ACE inhibitors may interrupt the renin-angiotensin-aldosterone system to reduce renal vasoconstriction.

ACE inhibitors may interrupt the renin-angiotensin-aldosterone system to reduce renal vasoconstriction. In addition to increasing water intake to decrease vasopressin levels, the angiotensin-converting enzyme (ACE) inhibitors or angiotensin II receptor blockers (ARBs) may be used to interrupt the renin-angiotensin-aldosterone system as a means of reducing intraglomerular pressure and renal vasoconstriction. Although not approved by the Food and Drug Administration (FDA), there has been recent interest in the use of vasopressin receptor antagonists (vaptans) to decrease cyst development.

A lung cancer client with small cell carcinoma may secrete an excess of which hormone causing an ectopic form of Cushing syndrome due to a nonpituitary tumor?

ACTH

The health care provider is reviewing diurnal variation pattern in adrenocorticotropic (ACTH) levels. Select the typical diurnal variation pattern in adrenocorticotropic (ACTH) levels.

ACTH peaks in the morning and declines throughout the day.

A 77-year-old lifetime smoker has been diagnosed with a tumor in his lung at the site of an old tubercule scarring site, located in a peripheral area of his bronchiolar tissue. What is this client's most likely diagnosis? a) Large cell carcinoma b) Adenocarcinoma c) Small cell lung cancer d) Squamous cell carcinoma

Adenocarcinoma Explanation: Adenocarcinoma is associated with the periphery of the lungs, often at the site of scarring, and can occur in alveolar or bronchiolar tissue. Squamous cell carcinoma, small cell lung cancer, and large cell carcinoma are less commonly associated with these traits.

Which of the following residents of a long-term care facility is most likely to be exhibiting the signs and symptoms of chronic obstructive pulmonary disease (COPD)?

An 81-year-old male who has a productive cough, dyspnea and hypoxia

17. An 80-year-old Caucasian man has been diagnosed with CKD. His physical and workup show uremia, azotemia, and elevated BUN. What can be done to slow the decline of his kidney function? A) Do a kidney transplant B) Control blood glucose and blood pressure C) Increase intravenous fluids D) Restrict protein in the diet

B) Control blood glucose and blood pressure

The nurse is caring for a 42-year-old male client who is admitted for treatment of heart failure. He has abnormally large hands and feet and a broad face with a protruding jaw. Based on these signs and symptoms, the nurse identifies which of the following endocrine disturbances as the most likely cause for these physical changes?

Acromegaly

At times, it is necessary to give medications that suppress the adrenal glands on a long-term basis. When the suppression of the adrenals becomes chronic, the adrenal glands atrophy. What does the abrupt withdrawal of these suppressive drugs cause?

Acute adrenal insufficiency

A 41 year-old male client has presented to the emergency department with an acute onset of increased respiratory rate and difficulty breathing. STAT chest X-ray indicates diffuse bilateral infiltrates of his lung tissue and ECG displays no cardiac dysfunction. What is this client's most likely diagnosis?

Acute lung injury

A patient is being evaluated for kidney stones. The nurse anticipates the patient will manifest:

Acute onset of colicky or dull and achy intermittent flank pain One of the major manifestations of kidney stones is pain. Depending on the location, there are two types of pain associated with kidney stones—colicky or non colicky. The pain may radiate to the lower abdominal quadrant, bladder area, perineum, or scrotum in men. Stones are not externally visible or palpable. Obstruction by a stone may result in urinary retention and increased fluid volume.

Of the following list of nursing interventions, which would be considered priority when managing a patient with life-threatening myxedematous coma? Select all that apply. Administer 3% sodium IV solution to increase sodium levels. Administer 50% dextrose to raise glucose levels. Place on oxygen therapy to encourage deep breathing. Place on a warming bed to raise body temperature. Administer sedatives frequently to prevent seizures.

Administer 3% sodium IV solution to increase sodium levels. Administer 50% dextrose to raise glucose levels. Place on oxygen therapy to encourage deep breathing.

A client in the emergency department reports sudden onset of coughing with a feeling of chest tightness. Lung sounds are diminished with wheezing in all lung fields. The client is anxious and the skin is moist. What is the first intervention the nurse should implement?

Administer a beta2 agonist.

Which client with a diagnosis of asthma is at greatest risk for developing an asthma attack?

Adult with a respiratory virus and reflux

A prison inmate contracted tuberculosis during a recent outbreak. The nurse caring for these inmates correctly identifies which of the following as the mode of transmit for this disease? a) Airborne droplets b) Direct skin contact c) Body fluid transmission d) Fecal-oral contact

Airborne droplets Explanation: M. tuberculosis is an airborne infection spread by minute, invisible particles, called droplet nuclei that are harbored in the respiratory secretions of people with active tuberculosis. Coughing, sneezing, and talking all create respiratory droplets. These droplets evaporate and leave organisms (droplet nuclei), which remain suspended in the air and are circulated by air currents. Thus, living under crowded and confined conditions increases the risk for spread of the disease.

What description is characteristic of the pathology of chronic bronchitis?

Airways are obstructed by mucus, causing hypoxemia.

What description is characteristic of the pathology of interstitial lung disease?

Airways become stiff and resist expansion, leading to hypoxemia.

A nurse is caring for a patient with diabetic glomerulosclerosis. The analysis is reviewed for the presence of which of the following manifestations? a) Albumin b) Red blood cells c) Sodium d) Potassium

Albumin The clinical manifestations of diabetic glomerulosclerosis are closely linked to those of diabetes. The increased glomerular filtration rate that occurs in people with early alterations in renal function is associated with microalbuminuria, which is defined as urinary albumin excretion of 30 to 300 mg in 24 hours.

A client has been given the diagnosis of diffuse glomerulonephritis. They ask the nurse what diffuse means. The nurse responds: a) All glomeruli and all parts of the glomeruli are involved. b) Only some of the glomeruli are affected. c) That the mesangial cells are being affected. d) Only one segment of each glomerulus is involved.

All glomeruli and all parts of the glomeruli are involved. Glomerular changes can be diffuse, involving all glomeruli and all parts of the glomeruli; focal, meaning only some of the glomeruli are affected; segmental, involving only a certain segment of each glomerulus; and mesangial, affecting only mesangial cells.

The nurse is performing an assessment for a client with Cushing syndrome and observes a "buffalo hump" on the back, a moon face, and a protruding abdomen. What does the nurse understand contributes to the distribution of fat in these areas?

Altered fat metabolism

Many pharmaceutical agents may cause pulmonary damage. The nurse knows the following cardiac medication has been associated with toxic effects in the lungs:

Amiodarone

Which of the following residents of a long-term care facility is most likely to be exhibiting the signs and symptoms of chronic obstructive pulmonary disease (COPD)?

An 81 year-old male who has a productive cough and recurrent respiratory infections.

Which of the following residents of a long-term care facility is most likely to be exhibiting the signs and symptoms of chronic obstructive pulmonary disease (COPD)? A 79-year-old lifetime smoker who is complaining of shortness of breath and pain on deep inspiration An 81-year-old smoker who has increased exercise intolerance, a fever, and increased white blood cells An 81-year-old male who has a productive cough and recurrent respiratory infections An 88-year-old female who experiences acute shortness of breath and airway constriction when exposed to tobacco smoke

An 81-year-old male who has a productive cough and recurrent respiratory infections

Which of the following individuals is experiencing a chronic disorder of ventilation?

An elderly, lifelong smoker who has been admitted to hospital with emphysema

The mother of a 19-week-old infant has brought her baby in for assessment to a pediatrician because of the baby's persistent weight loss and diarrhea. An intestinal biopsy has confirmed a diagnosis of celiac disease, and the child's mother is anxious to know what caused the disease. Which of the following aspects of the etiology of celiac disease would underlie the explanation that the physician provides? Bacterial or chemical invasion of the peritoneum leads to decreased nutrient absorption and transport. An inappropriate T-cell-mediated response results in increased levels of antibodies and an inflammatory response. Neurogenic or muscular inhibition of peristalsis results in inappropriate motility of ingested food in the lower small intestine and the colon. Inability to process or absorb the fat content of breast milk results in malnutrition and deficiency of fat-soluble vitamin

An inappropriate T-cell-mediated response results in increased levels of antibodies and an inflammatory response.

Which additional physical finding would you anticipate seeing in a child suspected of having a Wilms tumor? a) A visible fine red rash on the body b) Presence of a palpable flank mass c) Anomalies of the genitourinary system d) Massive generalized edema

Anomalies of the genitourinary system An important feature of Wilms tumor is its association with other congenital anomalies, aniridia (absence of the iris), hemihypertrophy (enlargement of one side of the face or body), and other congenital anomalies, usually of the genitourinary system. Wilms tumor usually is a solitary mass that occurs in any part of the kidney. The common presenting signs are a large asymptomatic abdominal mass and hypertension.

14. A 28-year-old man presents with complaints of diarrhea, fecal urgency, and weight loss. His stool is light colored and malodorous, and it tends to float and be difficult to flush. He has also noted tender, red bumps on his shins and complains of pain and stiffness in his elbows and knees. Sigmoidoscopy reveals discontinuous, granulomatous lesions; no blood is detected in his stool. Which of the following diagnoses would his care team first suspect? A) Crohn disease B) Ulcerative colitis C) Diverticulitis D) Colon cancer

Ans A Feedback: Crohn disease, like ulcerative colitis, causes diarrhea, fecal urgency, weight loss, and systemic symptoms such as erythema nodosum and arthritis. Unlike ulcerative colitis, it also causes steatorrhea but is not as likely to cause blood in the stool. The granulomatous "skip" lesions confirm the diagnosis of Crohn disease. Neither diverticulitis nor colon cancer would cause this combination of symptoms and signs.

3. A female neonate has been in respiratory distress since delivery and is unresponsive to oxygen therapy. Endoscopy has confirmed a diagnosis of esophageal atresia and tracheoesophageal fistulae (EA/TEF). Which of the following explanations should the care team provide to the infant's parents? A) "We will have to perform surgery to correct the hole in her throat to make sure that she is able to swallow and breathe normally." B) "This problem will require respiratory therapy and supplementary feeding, but it will likely resolve itself over time." C) "The biggest risk that your daughter will face until this is fixed is the danger of malnutrition and dehydration." D) "The priority in our immediate treatment prior to her surgery will be pain management, as the contents of her stomach can burn her lungs."

Ans A Feedback: EA/TFE necessitate surgery and preclude both normal respiration and swallowing. Aspiration is the primary immediate risk and the priority for treatment. Although the infant will require respiratory therapy to assist with proper tube placement and ventilator maintenance, the only effective treatment is surgery. Aspiration of feeding (aspiration pneumonia) is a major complication that can occur immediately and can be life threatening. Maintaining an open airway and adequate gas exchange are the priority nursing diagnoses for this infant.

16. A number of clients on a geriatric subacute medical floor of a hospital have developed foul-smelling diarrhea over the last several days, and subsequent culture of stool samples has confirmed the presence of Clostridium difficile in each case. The care team in the unit would recognize that which of the following factors likely contributed to the health problem and would anticipate which of the following treatments? A) The use of broad-spectrum antibiotics likely played a role in the development of infections, and most clients would likely receive metronidazole as a treatment. B) Genetic predisposition and the presence of the bacterium in clients' normal flora likely contributed, and treatment would consist of broad-spectrum antibiotics. C) Poor hand washing practice on the part of care providers led to the outbreak, and treatment will consist of hydration and nutritional support. D) Ingestion of contaminated food probably contributed to the infections, and corticosteroids will be needed to treat them.

Ans A Feedback: Elimination of the normal intestinal flora by broad-spectrum antibiotics commonly precedes infection by C. difficile, and metronidazole is the normal treatment. Neither genetic predisposition nor ingestion of contaminated food is a likely factor. While poor hand hygiene can spread the bacteria, treatment for C. difficile necessitates antibiotics and not simply hydration and nutritional support. The treatment of C. difficile is with antibiotics. Metronidazole is the drug of choice with vancomycin, being reserved for people who cannot tolerate metronidazole. Corticosteroids will not kill the bacterium.

4. A stroke patient is having difficulty swallowing food and beverages. The patient complains that he feels like "the food is sticking to the back of his throat." Given this information, the priority nursing interventions would be to A) make the patient "nothing per os" (NPO) and call the physician. B) feed the patient while he is sitting in an upright position. C) add a thickening agent to all of the patient's beverages. D) warrant no action since this is a normal occurrence after a stroke.

Ans A Feedback: People with dysphagia usually complain of choking, coughing, or an abnormal sensation of food sticking in the back of the throat or upper chest when they swallow. A neuromuscular cause involves lesions of the CNS, such as a stroke, which often involve the cranial nerves that control swallowing. Feeding in upright position is good once it is determined by swallowing evaluation that the patient can swallow food without it going into the lungs. Likewise, thickening agents help dysphagia patients after a swallow evaluation has been performed. No action could put the patient at risk for aspiration pneumonia.

9. A male patient has just been diagnosed with esophageal cancer. He knew that he was losing weight and fatigued most days, but he just attributed it to aging and working. The physician recommends chemotherapy and irradiation. However, the cancer has already metastasized. The patient asks the nurse what he can expect if he agrees to the treatments. The nurse responds, A) "The therapies may shrink the cancer." B) "The doctor is prescribing treatment measures to help you swallow better." C) "These therapies will most likely cure your cancer." D) "You need to talk with your physician some more. I will page him for you."

Ans A Feedback: The prognosis for people with cancer of the esophagus, although poor, has improved. Even with modern forms of therapy, the long-term survival is limited because, in many cases, the disease has already metastasized by the time the diagnosis is made. These therapies may help with food consumption, but that is not their primary purpose. Nor, will these therapies cure the cancer.

17. Parents have brought their 7-year-old child into the emergency room with abdominal pain. Which of the following clinical manifestations would lead the health care team to suspect the child has appendicitis? Select all that apply. A) Tenderness in right lower quadrant with palpation B) Rebound tenderness in inguinal areas with palpation C) Redness and warmth over right lower quadrant D) Bloating and flatulence noticeable E) Urine has the smell of stool with brown coloring

Ans A, B Feedback: Appendicitis usually has an abrupt onset. Palpation of the abdomen usually reveals a deep tenderness in the LRQ, which is confined to a small area approximately the size of the fingertip. It usually is located at approximately the site of the inflamed appendix. Rebound tenderness, which is pain that occurs when pressure is applied to the area and then released, and spasm of the overlying abdominal muscles are common. An abscess may display redness and warmth. Bloating and flatulence are usually seen with diverticular disease as is the development of fistula (vesicosigmoid), where stool is seen in the urine.

10. A 43-year-old male client has presented to the emergency department with vomiting that he claims is of a sudden onset. The client also states that the emesis has often contained frank blood in the hours prior to admission. His vital signs are stable with temperature 98.3°F, pulse 88, BP 140/87, and respiratory rate 18. Which of the following potential contributing factors would the health care team suspect first? A) Overuse of antacids B) Alcohol consumption C) Staphylococcal enterotoxins D) Effects of Helicobacter pylori

Ans B Feedback: Acute gastritis associated with alcohol use is characterized by intermittent vomiting and the possibility of hematemesis. Aspirin and H. pylori do not normally cause such an acute symptom onset, and infectious organisms do not normally cause bleeding of the stomach lining. A combination of calcium carbonate and magnesium is commonly found in antacids. Overdose of antacids can result in irregular heartbeat, poor balance, shallow, rapid breathing and stupor (lack of alertness).

19. The mother of a 19-week-old infant has brought her baby in for assessment to a pediatrician because of the baby's persistent weight loss and diarrhea. An intestinal biopsy has confirmed a diagnosis of celiac disease, and the child's mother is anxious to know what caused the disease. Which of the following aspects of the etiology of celiac disease would underlie the explanation that the physician provides? A) Bacterial or chemical invasion of the peritoneum leads to decreased nutrient absorption and transport. B) An inappropriate T-cell-mediated response results in increased levels of antibodies and an inflammatory response. C) Neurogenic or muscular inhibition of peristalsis results in inappropriate motility of ingested food in the lower small intestine and the colon. D) Inability to process or absorb the fat content of breast milk results in malnutrition and deficiency of fat-soluble vitamins.

Ans B Feedback: Celiac disease is rooted in an inappropriate immune response that initiates an inflammatory response, resulting in loss of absorptive villi. Bacterial or chemical invasion of the peritoneum is associated with peritonitis, while inhibition of peristalsis is associated with obstructions. An inability to process or absorb fat is associated with malabsorption syndrome.

11. A patient has recently been diagnosed with H. pylori gastritis. The nurse knows that this form of gastritis is usually treated with a combination of an antibiotic and A) antianxiety medications. B) proton pump inhibitors. C) lactulose, to reduce the blood ammonia levels. D) calcium carbonate, an antacid.

Ans B Feedback: H. pylori is associated with an increased risk of gastric adenocarcinoma, gastric atrophy, and peptic ulcer. It is less likely to contribute to IBD, esophagitis, or diverticular disease. Eradication of H. pylori is difficult. Treatment requires a combination therapy that includes the use of antibiotics and a proton pump inhibitor. The proton pump inhibitors have direct antimicrobial properties against H. pylori. Antianxiety medications will not kill the bacteria. H. pylori is not associated with elevated blood ammonia levels. Calcium carbonate is usually given to relieve heartburn caused by GERD.

2. One of the most reliable predictors for worsening autosomal dominant polycystic kidney disease is A) serum creatinine levels. B) blood urea nitrogen (BUN) level. C) urine albumin excretion (UAE). D) urine specific gravity.

C

2. A middle-aged male walks into the emergency department complaining of chest pain radiating to the neck, shortness of breath, and nausea. His heart rate is 120 and BP is 94/60. The ED physician recognizes the patient is having an acute MI with decreased cardiac output. The nurse identifies the nausea to be in response to A) the patient not having a very high pain tolerance. B) hypoxia exerting a direct effect on the chemoreceptor trigger zone. C) the patient not having digested his meal completely. D) fear of having to make major lifestyle changes.

Ans B Feedback: Hypoxia exerts a direct effect on the vomiting center (chemoreceptor trigger zone), producing nausea and vomiting. This direct effect probably accounts for the vomiting that occurs during periods of decreased cardiac output, shock, and environmental hypoxia. We are given no information about the patient's pain tolerance, when he last had a meal, or his routine lifestyle. This patient is going into shock (rapid pulse, low BP) that can result in shunting of blood away from the gut and other organs. During shock, the priority organs for oxygenation include the heart, brain, lungs, and kidneys.

13. Which of the following clients is most clearly displaying the signs and symptoms of irritable bowel disease (IBD)? A) A 32-year-old mother who complains of intermittent abdominal pain that is worse during her menstrual period B) A 51-year-old male who states that his stomach pain is in his lower abdomen, "comes and goes," and "feels more like a cramp than a dull ache" C) A 44-year-old man who works the evening shift at a factory and who states that his lower abdominal pain is much worse at night than during the day D) A 24-year-old man who has a stressful job but whose diarrhea and cramping do not worsen during periods of high stress

Ans B Feedback: IBD is commonly manifested as intermittent lower abdominal pain that feels like cramping. Defecation normally relieves the pain, and symptoms are normally not present at night or during sleep. Stress commonly exacerbates symptoms.

18. A 22-year-old student has developed a fever and diarrhea while on a backpacking trip in Southeast Asia. His oral temperature is 101.4°F. The diarrhea is bloody, frequent, and small in volume. These clinical manifestations are sufficiently distressing that he is visiting a local medical clinic in the area. Which of the following diagnoses best characterizes this health problem? A) Noninflammatory diarrhea B) Inflammatory diarrhea C) Factitious diarrhea D) Secretory diarrhea

Ans B Feedback: Inflammatory diarrhea is often characterized by small-volume diarrhea that is bloody and accompanied by a fever. Noninflammatory diarrhea is normally larger in volume and not bloody. Factitious diarrhea is normally attributable to laxative use, and secretory diarrhea is associated with increased secretory processes of the bowel; neither is likely to produce bloody stool.

8. A 68-year-old African American man who has smoked for at least 50 years reports that lately he feels as though food is getting stuck in his throat. At first, this was a problem just with dry food, but now his morning oatmeal is getting "stuck." On questioning, he reports drinking at least three alcoholic beverages nearly every day. His problem is most likely A) achalasia. B) squamous cell carcinoma of the esophagus. C) dysphagia secondary to scleroderma. D) gastrointestinal reflux disease.

Ans B Feedback: Squamous cell carcinoma of the esophagus is the seventh leading cause of cancer death among men, particularly black men; mean age at diagnosis is 67 years. Alcohol and tobacco use are the main risk factors for this cancer, and dysphagia is a common presenting complaint. An esophageal motility disorder involves the smooth muscle layer of the esophagus and the lower esophageal splincter (LES). Achalasia is characterized by difficulty swallowing and regurgitation. GERD (gastroesophageal reflux disease) is a condition that causes the esophagus to become irritated and inflamed. Clients with GERD usually feel a burning in the chest or throat called heartburn. Sometimes, they taste stomach fluid in the back of the mouth.

5. A nurse practitioner is providing care for a male client with a long-standing hiatal hernia. Which of the following statements most accurately captures an aspect of the pathophysiology of hiatal hernias? A) Paraesophageal hiatal hernias are common and are normally not treated if the client is asymptomatic. B) The root causes of hiatal hernias are normally treatable with medication. C) If esophageal acid clearance is impaired, esophagitis can result. D) An incompetent pyloric sphincter and high-fat diet are commonly implicated in the development of hiatal hernias.

Ans C Feedback: Erosive esophagitis can be a complication of hiatal hernias if esophageal acid clearance is significantly impaired. Paraesophageal hiatal hernias are more serious than the sliding variety and require treatment. The root cause of hiatal hernias, herniation of the stomach through the diaphragm, is not normally amenable to treatment with medication. The pyloric sphincter is not associated with hiatal hernias.

7. Parents of a 20-month-old infant report that he refuses food or eats poorly and that he grimaces when he swallows. He also is irritable and cries a lot. The mother is worried that he ate something inappropriate this morning, because he vomited something that looked like coffee grounds. Which of the following health problems would the care team first suspect? A) Rotavirus infection B) Appendicitis C) Esophagitis from gastrointestinal reflux D) Hirschsprung disease

Ans C Feedback: Esophagitis secondary to reflux can cause feeding problems, early satiety, and hematemesis. Infants may demonstrate signs of pain when swallowing and may be irritable and cry frequently. Rotavirus causes diarrhea and vomiting, but not the other symptoms. Appendicitis is inflammation of the appendix. Appendicitis usually starts with the main symptom of pain around the navel that moves to the lower right abdomen. Hirschsprung disease is a blockage of the large intestine due to improper muscle movement in the bowel. It is a congenital condition, which means it is present from birth. In Hirschsprung disease, the nerves are missing from a part of the bowel. One primary s/s is a failure to pass meconium shortly after birth.

12. Following a history of gastric pain and an endoscopy, a client has been diagnosed with a duodenal peptic ulcer. Which of the following teaching points should his caregiver provide? A) "While your diet most certainly contributed to this problem, the good news is that changing your diet can help solve it." B) "Ulcers like yours do not penetrate all layers of the stomach or duodenum, so you don't have to worry about losing too much blood." C) "Your family history, your smoking history, and NSAID use may all have contributed to this problem." D) "While there aren't really any effective medications for these ulcers, changes in lifestyle can keep them well controlled."

Ans C Feedback: Family history, NSAID use, and smoking have all been identified as contributing factors in the development of peptic ulcers. Diet therapy has not been shown to be effective, and duodenal peptic ulcers are more common than the gastric variant. Perforation occurs when an ulcer erodes through all layers of the stomach or duodenum wall. When perforation occurs in older adults, their mortality is significantly increased. Effective medication regimens are available with antacids, H2-receptor antagonists or proton pump inhibitors being the most common medications used.

20. A 71-year-old male has been recently diagnosed with a stage III tumor of colorectal cancer and is attempting to increase his knowledge base of his diagnosis. Which of the following statements about colorectal cancer demonstrates a sound understanding of the disease? A) "If accurate screening test for this type of cancer existed, it could likely have been caught earlier." B) "The NSAIDs and aspirin that I've been taking for many years probably contributing to me getting cancer." C) "While diet is thought to play a role in the development of colorectal cancer, the ultimate causes are largely unknown." D) "A large majority of patients who have my type of colon cancer survive to live many more years."

Ans C Feedback: The etiology of cancer of the colon and rectum remains largely unidentified, though dietary factors are thought to exist. The prognosis, especially with stage III tumors, is poor. Simple and accurate screening tests do exist for colorectal cancer, while drugs are not implicated in the etiology.

1. The nurse walks into a room and finds the patient forcefully expelling stomach contents into a wash basin. When documenting this occurrence, the nurse will use the term A) nauseous. B) retching. C) vomiting. D) expatriate.

Ans C Feedback: Vomiting or emesis is the sudden and forceful oral expulsion of the contents of the stomach. It is usually preceded by nausea. Nausea is a subjective and unpleasant sensation. Retching consists of rhythmic spasmodic movements of the diaphragm, chest wall, and abdominal muscles. It usually precedes or alternates with periods of vomiting. Expatriate means to banish or withdraw.

6. After several months of persistent heartburn, a 57-year-old female client has been diagnosed with gastroesophageal reflux disease (GERD). Which of the following treatment regimens is likely to best address the woman's health problem? A) Surgical correction of the incompetent pylorus B) Antacids; avoiding positions that exacerbate reflux; a soft-textured diet C) Weight loss and administration of calcium channel blocking medications D) Proton pump inhibitors; avoiding large meals; remaining upright after meals

Ans D Feedback: Proton pump inhibitors block the final stage of gastric acid production, effectively controlling the root cause of the esophageal damage associated with GERD. The pylorus is not involved, and a soft diet is not indicated. Calcium channel-blocking drugs would not address the problem. Calcium channel blockers are primarily heart disease drugs that relax blood vessels and increase the supply of blood and oxygen to the heart while also reducing the heart's workload.

15. A 20-year-old woman has visited her family physician due to occasional bouts of bloody diarrhea over the past several weeks, a phenomenon that she experienced 2 years prior as well. Her physician has diagnosed her with ulcerative colitis based on her history and visualization of the affected region by colonoscopy and sigmoidoscopy. Which of the following pathophysiological phenomena is most likely to underlie the client's health problem? A) Fissures and crevices developing in the mucosa that are seen as a characteristic "cobblestone" appearance B) Erosion of the endothelial lining of the distal small intestine by a combination of genetic, autoimmune, and environmental factors C) Compromise of the mucosal layer of the large intestinal surface by the effects of H. pylori D) Mucosal hemorrhages that have developed into crypt abscesses, which have in turn necrotized and ulcerated

Ans D Feedback: The etiology and course of ulcerative colitis involves mucosal hemorrhages developing into crypt abscesses, with consequent necrosis and ulceration. "Cobblestone" appearance of intestinal mucosa is associated with Crohn disease. Ulcerative colitis is confined to the colon and rectum, and H. pylori is not commonly implicated in the etiology.

A client has sudden severe dyspnea, fear, asymmetrical chest movement and decreased lung sounds on the right side. Which intervention is most appropriate?

Assist to high-Fowler's position and prepare for chest tube insertion.

A nurse is teaching a group of older adults about the value of including foods containing antioxidants in their diet. Which of the following statements best captures the rationale underlying the nurse's advice?

Antioxidants inhibit the actions of reactive oxygen species

The nurse is providing care for a client with a diagnosis of amyotrophic lateral sclerosis (ALS). The nurse recognizes which of the following mechanisms is suspected to play a role in the cellular death associated with ALS?

Apoptosis

A car accident client is admitted with a chest tube following pneumothorax. He also has an elevated blood alcohol level. When the nurse enters his room, she notes the client is dyspneic, short of breath, and holding his chest tube in his hand. When the nurse pulls the linens back, she finds a "sucking" chest wound. After calling a "code blue," the next priority intervention would be to: a) Observe and wait for the code blue team to bring equipment b) Apply a Vaseline gauze (airtight) dressing over the insertion site c) Try to calm the patient down by maintaining therapeutic communication d) Place the client's meal napkin over the wound

Apply a Vaseline gauze (airtight) dressing over the insertion site Explanation: The client has a medical emergency. Sucking chest wounds, which allow air to pass in and out of the chest cavity, should be treated by promptly covering the area with an airtight covering. Chest tubes are inserted as soon as possible. The other interventions will not help minimize the amount of air entering the pleural space

Which medication order should the nurse question for a client with hyperthyroidism who is experiencing signs of impending thyroid storm?

Aspirin

In the early morning, an African-American woman brings her 5-year-old son to the emergency room. The boy is wheezing, short of breath, and has a dry cough. The mother states that he has always been very healthy. He went to bed with only a slight cold and a runny nose but woke her with his coughing shortly after 4 a.m. His symptoms worsened so dramatically that she brought him to the hospital. The care team would most likely suspect that he has:

Asthma

The parents of an infant bring the child to the emergency room three hours after they put the child to bed in apparent good health. The child is restless, the pulse is 140 beats/ minute, and lung sounds are diminished in all lung fields with intercostal retractions. What is the most likely cause of these symptoms?

Asthma

A patient arrives in the ED after an automobile accident. Which of the following clinical manifestations leads the nurse to suspect a pneumothorax? Select all that apply.

Asymmetrical chest movements, esp. on inspiration. • Diminished breath sounds over painful chest area. • Respiratory rate 34.

Prolonged immobility is implicated in the development of which of the following disorders?

Atelectasis and pulmonary embolism

The nurse suspects that a newborn infant who presents with bilateral flank masses, impaired lung development, and oliguria may be suffering from which of the following disorders? a) Autosomal recessive polycystic kidney disease (ARPD) b) Simple renal cysts c) Horseshoe kidney abnormality d) Autosomal dominant polycystic kidney disease

Autosomal recessive polycystic kidney disease (ARPD) The typical infant with ARPD presents with bilateral flank masses, accompanied by severe renal failure, signs of impaired lung development, and variable degrees of liver fibrosis and portal hypertension.

Atelectasis is the term used to designate an incomplete expansion of a portion of the lung. Depending on the size of the collapsed area and the type of atelectasis occurring, the nurse may see a shift of the mediastinum and trachea. Which way does the mediastinum and trachea shift in compression atelectasis?

Away from the affected lung

Atelectasis is the term used to designate an incomplete expansion of a portion of the lung. Depending on the size of the collapsed area and the type of atelectasis occurring, the nurse may see a shift of the mediastinum and trachea. Which way does the mediastinum and trachea shift in compression atelectasis? a) Toward the affected lung b) Away from the trachea c) Away from the affected lung d) Toward the mediastinum

Away from the affected lung Explanation: If the collapsed area is large, the mediastinum and trachea shift to the affected side. In compression atelectasis, the mediastinum shifts away from the affected lung. None of the other answers are correct.

10. A frantic mother brings her young child into the emergency department. She states that during the evening bath, she noticed a large mass in her child's abdomen. After diagnostic testing, the pediatrician tells the parents that their child has Wilms tumor, stage IV. After the doctor leaves the room, the parents ask the nurse, "What does this mean?" The nurse will respond, "Your child ('s) A) "has cancer in his stomach." B) "has cancer in the kidney that has spread most likely to his lungs." C) "will need to undergo surgery to remove both kidneys and then go on dialysis." D) "tumor can be easily treated with chemotherapy. We will start this soon."

B

Which of the following individuals are likely to display identified risk factors for the development of lower urinary tract obstruction? Select all that apply. A) A 32-year-old woman who had a healthy delivery of her third child 4 months ago B) A 68-year-old man who has been diagnosed with benign prostatic hyperplasia (BPH) C) A 55-year-old man with diabetes who is receiving diuretic medications for the treatment of hypertension D) A 30-year-old woman who has been diagnosed with gonorrhea E) A 74-year-old woman who has developed a lower bowel obstruction following several weeks of chronic constipation F) A 20-year-old man who has spina bifida and consequent impaired mobility.

B) A 68-year-old man who has been diagnosed with benign prostatic hyperplasia (BPH) D) A 30-year-old woman who has been diagnosed with gonorrhea E) A 74-year-old woman who has developed a lower bowel obstruction following several weeks of chronic constipation F) A 20-year-old man who has spina bifida and consequent impaired mobility. BPH frequently obstructs the urethra, while sexually transmitted diseases, bowel obstructions, and spina bifida are also associated with physical blockages of the lower urinary tract. Postpartum women and individuals receiving diuretics would be more likely to be at risk for incontinence rather than urinary retention.

Which of the following medications would the nurse anticipate being prescribed for the renal failure patient who has hyperphosphatemia?

B) Calcium carbonate

A 55-year-old man has made an appointment to see his family physician because he has been awakening three to four times nightly to void and often has a sudden need to void with little warning during the day. What is the man's most likely diagnosis and possible underlying pathophysiological problem? A) Stress incontinence due to damage to CNS inhibitory pathways B) Overactive bladder that may result from both neurogenic and myogenic sources C) Overactive bladder due to intravesical pressure exceeding urethral pressure D) Overflow incontinence that can result from displacement of the angle between the bladder and the posterior proximal urethra

B) Overactive bladder that may result from both neurogenic and myogenic sources The man's complaints are typical of overactive bladder, a condition that can result from the interaction of both the nervous control of bladder emptying and the muscles of the bladder itself. His symptoms are not characteristic of stress incontinence, and when intravesical pressure exceeds, urethral pressure overflow incontinence results. The angle between the bladder and the posterior proximal urethra is more commonly a factor in the continence of females.

A 51-year-old woman diagnosed with a cerebrovascular accident (CVA) 5 months prior is distressed that she has had several recent episodes of urinary incontinence. She has asked her nurse practitioner why this is the case. Which of the following statements best captures the fact that would underlie the nurse's response to the client? A) Neurological diseases like MS often result in flaccid bladder dysfunction. B) She may be unable to sense her bladder filling as a result of her MS. C) Lesions to the basal ganglia or extrapyramidal tract associated with MS inhibit detrusor contraction. D) Pathological reductions in bladder volume brought on my MS necessitate frequent micturition.

B) She may be unable to sense her bladder filling as a result of her MS.

During male ejaculation, which of the following statements addresses why sperm is not normally seen inside the bladder? A) The parasympathetic nervous system keeps the seminal fluid inside the urethra. B) The musculature of the trigone area, bladder neck, and prostatic urethra contract at the same time. C) With ejaculation, the male expels some urine along with the seminal fluid to wash any extra sperm out of the bladder. D) The detrusor muscle relaxes allowing for the closing of the sphincter at the base of the bladder.

B) The musculature of the trigone area, bladder neck, and prostatic urethra contract at the same time. During male ejaculation, which is mediated by the SNS, the musculature of the trigone area and that of the bladder neck and prostatic urethra contracts and prevents the backflow of seminal fluid into the bladder.

An 82-year-old resident of a long-term care facility with a recent history of repeated urinary tract infections and restlessness is suspected of having urinary retention. Which of the following actions by the care team is most appropriate? A) Uroflowmetry to determine the rate of the client's urine flow B) Ultrasound bladder scanning to determine the residual volume of urine after voiding C) Renal ultrasound aimed at identifying acute or chronic kidney disease D) Urinalysis focusing on the presence of or absence of microorganisms, blood, or white cells in the man's urine

B) Ultrasound bladder scanning to determine the residual volume of urine after voiding Ultrasound bladder scanning yields a fast and noninvasive indication of whether or not an individual is adequately emptying his or her bladder with each void. Uroflowmetry would be less indicative of whether the man is retaining, and renal ultrasound would address deficits in urine production rather than bladder emptying. Urinalysis would be useful in the diagnosis of infections and/or renal issues more than deficiencies in bladder emptying.

5. A nurse is collecting a urine specimen prior to measuring the albumin level in a client's urine. A colleague questions the rationale for the test, stating, "I thought albumin was related to liver function, not kidney function." How can the nurse best respond to this statement? A) "Urine should normally be free of any proteins, and albumin is one of the more common proteins to be excreted in chronic renal failure." B) "Albumin levels are useful for diagnosing chronic kidney disease in clients with diabetes." C) "A urine dipstick test will tell us exactly how much albumin is being spilled by the client's kidneys." D) "A urine test for albumin allows us to estimate the client's GFR quite accurately."

B) "Albumin levels are useful for diagnosing chronic kidney disease in clients with diabetes."

Which of the following patients scheduled for an interventional radiology procedure requiring administration of radiocontrast dye would be considered at high risk for nephrotoxicity? Select all that apply.

B) A 25-year-old with a history of glomerular nephritis who is complaining of severe flank pain C) A 67-year-old diabetic undergoing diagnostic testing for new-onset proteinuria

A nurse has noted the high incidence of urinary tract obstructions of a variety of etiologies. Which of the following individuals are at risk of developing urinary obstructions? Select all that apply

B) A 29-year-old female, pregnant for the first time D) A 70-year-old male with benign prostatic hyperplasia (BPH) E) A 58-year-old male with renal calculi F) A 28-year-old male with a neurogenic bladder secondary to spinal cord injury

11. Which of the following clinical findings among older adults is most unlikely to warrant further investigation and possible treatment? A) An 81-year-old man's serum creatinine level has increased sharply since his last blood work. B) A 78-year-old woman's GFR has been steadily declining over several years. C) A 90-year-old woman's blood urea nitrogen (BUN) is rising. D) A dipstick of an 80-year-old man's urine reveals protein is present.

B) A 78-year-old woman's GFR has been steadily declining over several years.

7. Which of the following clients' diagnostic blood work is most suggestive of chronic kidney disease (CKD)? A) A client with high pH, low levels of calcium, and low levels of phosphate B) A client with low vitamin D levels, low calcitrol levels, and elevated parathyroid hormone (PTH) levels C) A client with low bone density, low levels of calcium, and low levels of phosphate D) A client with low potassium levels, low calcitrol levels, and increased PTH levels

B) A client with low vitamin D levels, low calcitrol levels, and elevated parathyroid hormone (PTH) levels

Which of the following clients' diagnostic blood work is most suggestive of chronic kidney disease (CKD)?

B) A client with low vitamin D levels; low calcitriol levels; and elevated parathyroid hormone (PTH) levels

Following an automobile accident where the patient had a traumatic amputation of his lower leg and lost greater than 40% of his blood volume, he is currently not producing any urine output. The nurse bases this phenomena on which of the following humoral substances responsible for causing severe vasoconstriction of the renal vessels

B) Angiotensin II and ADH

16. A patient is being treated for chronic kidney disease (CKD). One of the nurse's responsibilities is to explain to the patient the need to keep her blood pressure under control. Why is blood pressure control so important in CKD patients? A) Elevated blood pressure will result in greater amounts of urine formation and will over tax renal function. B) Elevated blood pressure will decrease nephron loss and accelerate renal failure C) Elevated blood pressure will slow the excretion of protein (proteinuria) and lead to a hypertrophic kidney D) Elevated blood pressure will decrease pressure on the nephron with a corresponding decrease in GFR leading to renal failure.

B) Elevated blood pressure will decrease nephron loss and accelerate renal failure

Which of the following lab results would be associated with abnormalities in kidney function? Select all that apply.

B) Increased creatinine levels e) Detectable levels of glucose in a urine sample F) Elevated cystatin-C level.

Which of the following aspects of a patient's site of inflammation would help the care provider rule out chronic inflammation?

B) Increased neutrophils

1. Which of the following data would a clinician consider to be most indicative of acute renal failure? A) Alterations in blood pH; peripheral edema B) Increased nitrogenous waste levels; decreased glomerular filtration rate (GFR) C) Decreased serum creatinine and blood urea nitrogen (BUN); decreased potassium and calcium levels D) Decreased urine output; hematuria; increased GFR

B) Increased nitrogenous waste levels; decreased glomerular filtration rate (GFR)

Which of the following processes would most likely be considered an anomaly during the cellular phase of inflammation?

B) Vasoconstriction

A 24-year-old woman presents with fever and painful, swollen cervical lymph nodes. Her blood work indicates neutrophilia with a shift to the left. She most likely has

B) a severe bacterial infection.

3. A nurse has noted the high incidence of urinary tract obstructions of a variety of etiologies. Which of the following individuals are at risk of developing urinary obstructions? Select all that apply. A) A 43-year-old male with an acid-base imbalance secondary to malnutrition B) A 29-year-old female, pregnant for the first time C) A 69-year-old female with anemia secondary to insufficient erythropoietin production D) A 70-year-old male with benign prostatic hyperplasia (BPH) E) A 58-year-old male with renal calculi F) A 28-year-old male with a neurogenic bladder secondary to spinal cord injury

B,D,E,F

While speaking to a senior citizen club about osteoarthritis (OA), which of the following facts are accurate to share? Select all that apply. By the time you are in your 70s, about 85% of adults will have some form of OA Men usually get OA in their hands, whereas women get OA primarily in their hips Obesity in women has been correlated to having OA in the knees Heredity does not play a significant role in the development of OA

By the time you are in your 70s, about 85% of adults will have some form of OA Obesity in women has been correlated to having OA in the knees

A patient with a 25-year history of smoking is diagnosed with emphysema. Physical assessment reveals an increased anterior-posterior chest diameter. Which of the following terms should the nurse use to document this finding?

Barrel chest

A patient with a 25-year history of smoking is diagnosed with emphysema. Physical assessment reveals an increased anterior-posterior chest diameter. Which of the following terms should the nurse use to document this finding? a) Blue bloater b) Pneumothorax c) Pink puffer d) Barrel chest

Barrel chest Explanation: An increased anterior-posterior chest diameter is referred to as a barrel chest.

Which of the following conditions is a frequent cause of urinary obstruction? Prostatic cancer Benign prostatic hyperplasia Prostatitis Urethritis

Benign prostatic hyperplasia

Which clinical manifestations would you expect to see in an infant diagnosed with autosomal recessive polycystic kidney disease (ARPKD)? a) Gross hematuria and massive generalized edema b) Elevated systemic blood pressure and severe pain c) Bilateral flank masses and impaired lung development d) Ascending urinary tract infection and vomiting

Bilateral flank masses and impaired lung development Clinical manifestations of the typical infant with ARPKD include bilateral flank masses, accompanied by severe renal failure, signs of impaired lung development, and variable degrees of liver fibrosis and portal hypertension. Potter facies and other defects associated with oligohydramnios may be present. Hypertension is usually noted within the first few weeks of life and is often severe.

A moderate hemothorax diagnosis would be confirmed by the presence of which of the following?

Blood filling approximately one third of the pleural space

A nurse is caring for a patient with systemic lupus erythematosis (SLE). The patient asks why a urinalysis is necessary. The best answer is that a urinalysis would determine whether which of the following factors was present in the urine? a) Sugar b) Blood or protein c) Cancer cells d) Potassium

Blood or protein Because of the high risk of kidney disease, all people with SLE should undergo routine urinalysis to monitor for the presence of hematuria or proteinuria.

A client with acromegaly comes to the clinic and informs the nurse that she is having a productive cough and a low grade fever. This is the client's fourth visit in one year for the same problem. What condition does the nurse understand results from this client's enlarged cartilaginous structures?

Bronchitis

8. A 25-year-old Asian American man arrives in the emergency room in a panic. Except for a bout with bronchitis a week earlier, he has been healthy his entire life; today he has blood in his urine. What disease has likely caused of his hematuria and how should it be treated? A) Goodpasture syndrome and will be treated with plasmapheresis and immunosuppressive therapy B) Membranous glomerulonephritis and should be treated with corticosteroids C) Immunoglobulin A nephropathy and may be advised to use omega-3 fatty acids to delay progression of disease D) Kimmelstiel-Wilson syndrome and should be treated with medication to control high blood pressure

C

A 24-year-old man is currently in a rehabilitation facility following a spinal cord injury at level T2. He is discussing his long-term options for continence management. Which of the following statements by the client demonstrates he has a clear understanding of the issue? A) "Self-catheterization can limit the recovery of my neural pathways that control my voiding if I do it too often." B) "It's critical that intermittent catheterization be performed using sterile technique." C) "An indwelling catheter certainly would work well, but it comes with a number of risks and possible complications." D) "An indwelling urethral catheter is the option that best minimizes my chance of a urinary tract infection."

C) "An indwelling catheter certainly would work well, but it comes with a number of risks and possible complications." Indwelling catheters carry a risk of infections and kidney stones. Catheterization does not influence the activity of the neural pathways, and intermittent catheterization can be performed using clean technique. Indwelling urethral catheters carry a high risk of urinary tract infections.

A 63-year-old woman has visited a physician because she has been intermittently passing blood-tinged urine over the last several weeks, and cytology has confirmed a diagnosis of invasive bladder cancer. Which of the following statements by the physician is most accurate? A) "There are new and highly effective chemotherapy regimens that we will investigate." B) "Fortunately, bladder cancer has a very low mortality rate, and successful treatment is nearly always possible." C) "It's likely that you'll need surgery, possibly a procedure called a cystectomy." D) "Unfortunately, there are nearly no treatment options for this type of cancer, but we will

C) "It's likely that you'll need surgery, possibly a procedure called a cystectomy." Surgical interventions are common in the treatment of bladder cancer. Effective chemotherapeutic regimens are not yet available, though there are certainly treatment options. The mortality rate of bladder cancer is high, at around 25%.

When teaching a community education class about the seven warning signs of cancer, the nurse will note that the most common sign of bladder cancer is A) inability to empty the bladder fully. B) colic spasms of the ureters. C) painless bloody urine. D) passage of large clots after voiding.

C) painless bloody urine. The most common sign of bladder cancer is painless hematuria. Gross hematuria is a presenting sign in the majority of people with the disease, and microscopic hematuria is present in most others. Answer choice A refers to flaccid bladder; answer choice B refers to kidney stones; answer choice D refers to clots that are usually seen after surgery such as TURP where bladder irrigation is called for to prevent the clots from blocking urine output.

A patient who has suffered a spinal cord injury at C4 is experiencing a sudden change in condition. His BP is 186/101; heart rate is 45; and he is profusely sweating and complaining of "not feeling right." The nurse should A) call a "Code Blue." B) page physician stat. and ask for an antihypertensive medication. C) palpate his bladder for overdistention. D) place his bed flat and elevate the foot of the bed.

C) palpate his bladder for overdistention. The most common causes of spastic bladder dysfunction are spinal cord lesions such as spinal cord injury, herniated intervertebral disk, vascular lesions, tumors, and myelitis. Because the injury interrupts CNS control of sympathetic reflexes in the spinal cord, severe hypertension, bradycardia, and sweating can be triggered by insertion of a catheter or mild overdistention of the bladder. The patient does not qualify for a Code Blue since he still has a pulse and is breathing. Antihypertensive medication is not necessary if the bladder is emptied. Placing him flat with the foot of the bed elevated will not help this situation.

Because they strengthen the pelvic floor muscles, Kegel exercises are most likely to help A) overflow incontinence. B) urge incontinence. C) stress incontinence. D) mixed incontinence.

C) stress incontinence. Stress incontinence is commonly caused by weak pelvic floor muscles, which allow the angle between the bladder and the posterior proximal urethra to change so that the bladder and urethra are positioned for voiding when some activity increases intra-abdominal pressure. Overflow incontinence results when the bladder becomes distended and detrusor activity is absent. Urge incontinence is probably related to CNS control of bladder sensation and emptying or to the smooth muscle of the bladder. Mixed incontinence, a combination of stress and urge incontinence, probably has more than one cause.

8. A nurse educator is performing client education with a 51-year-old man who has been recently diagnosed with chronic kidney disease. Which of the following statements by the client would the nurse most likely want to correct or clarify? A) "I'll be prone to anemia, since I'm not producing as much of the hormone that causes my bones to produce red blood cells." B) "My heart rate might go up because of my kidney disease, and my blood might be a lot thinner than it should be." C) "My kidney problems increase my chance of developing high blood pressure or diabetes." D) "I'll have a risk of either bleeding too easily or possibly clotting too quickly, though dialysis can help minimize these effects."

C) "My kidney problems increase my chance of developing high blood pressure or diabetes."

A nurse educator is performing client education with a 51-year-old man who has been recently diagnosed with chronic kidney disease. Which of the following statements by the client would the nurse most likely want to correct or clarify?

C) "My kidney problems increase my chance of developing high blood pressure or diabetes."

3. The clinical nurse educator on a nephrology unit of a large, urban hospital is orienting recent nursing graduates to the unit. Which of the following teaching points about acute tubular necrosis (ATN) should the educator include in the orientation session? A) "The cardinal signs of ATN are oliguria and retention of potassium, creatinine, and sulfates." B) "Ureteral and bladder outlet obstructions are often contributors to ATN." C) "Trauma, burns, and major surgery are common precursors to ATN." D) "Tubular epithelial cells are sensitive to ischemia and toxins, and damage is irreversible."

C) "Trauma, burns, and major surgery are common precursors to ATN."

The clinical nurse educator on a nephrology unit of a large, urban hospital is orientating recent nursing graduates to the unit. Which of the following teaching points about acute tubular necrosis (ATN) should the educator include in the orientation session?

C) "Trauma, burns, and major surgery are common precursors to ATN."

4. A family physician is providing care for a 61-year-old obese man who has a history of diabetes and hypertension. Blood work has indicated that the man has a GFR of 51 ml/min and elevated serum creatinine levels. Which of the following statements will the physician most likely provide to the client in light of these results? A) "We will regularly monitor your kidney function, but most likely your kidneys will be able to compensate on their own, and intervention is not required." B) "You likely have chronic kidney disease, and there may be urine in your blood until it is controlled." C) "Your chronic kidney disease has likely been caused by your diabetes and high blood pressure." D) "You're in kidney failure, and I'll be starting dialysis treatment immediately."

C) "Your chronic kidney disease has likely been caused by your diabetes and high blood pressure."

A family physician is providing care for a 61-year-old obese male who has a history of diabetes and hypertension. Blood work has indicated that the man has a GFR of 51 mL/minute with elevated serum creatinine levels. Which of the following statements will the physician most likely provide the client in light of these results?

C) "Your chronic kidney disease has likely been caused by your diabetes and high blood pressure."

During a period of extreme excess fluid volume, a renal dialysis patient may be administered which type of IV solution to shrink the swollen cells by pulling water out of the cell?

C) 3% sodium chloride

2. Which of the following clients would be considered to have a significant risk of developing the prerenal form of acute renal failure? A) A 68-year-old man with a diagnosis of benign prostatic hyperplasia (BPH) B) A 41-year-old woman who is admitted for intravenous antibiotic treatment of pyelonephritis C) A 79-year-old man with diagnoses of poorly controlled diabetes mellitus and congestive heart failure D) A 20-year-old man who is admitted for treatment of an overdose of a nephrotoxic drug

C) A 79-year-old man with diagnoses of poorly controlled diabetes mellitus and congestive heart failure

14. A patient presents with elevated BUN, systemic edema, a BP of 140/90, recurrent infections, and a GFR less than 1.00-1.48 ml/sec per 1.73 m2. What is the most likely diagnosis? A) Glomerulonephritis B) Acute renal failure C) Chronic kidney disease D) Pyelonephritis

C) Chronic kidney disease

A 63-year-old woman has begun a diet that purports to minimize the quantity and effects of free radicals in her body. What physiological processes could best underlie her care provider's teaching about her new diet?

C) Free radicals increase cytokine expression and adhesion molecule levels, resulting in increased inflammation.

12. A 35-year-old female ultramarathon runner is admitted to hospital following a day- long, 80-kilometer race because her urinary volume is drastically decreased and her urine is dark red. Tests indicate that she is in the initiating phase of acute tubular necrosis. Why is her urine red? A) Hematuria B) Hemoglobinuria C) Myoglobinuria D) Kidney bleeding

C) Myoglobinuria

A physician who is providing care for a 71-year-old male client with a recent diagnosis of renal failure and an acid-base imbalance is explaining some of the underlying etiology of the man's diagnoses to him and his family. Which of the following phenomena would most accurately underlie the teaching that the physician provides?

C) The kidneys have the primary responsibility for eliminating excess hydrogen ions from the body.

When explaining about the passage of urine to a group of nursing students, the clinic nurse asks them which muscle is primarily responsible for micturition? Their correct reply is the A) urinary vesicle. B) trigone. C) detrusor. D) external sphincter.

C) detrusor. The detrusor muscle is the muscle of micturition. When it contracts, urine is expelled from the bladder. External sphincter is a circular muscle that surrounds the urethra distal to the base of the bladder and can stop micturition when it is occurring. Trigone is a smooth triangular area that is bounded by the ureters and the urethra. Urinary vesicle is another name for the bladder.

A home health nurse is making a visit to a family with an 8-month-old infant with severe motor deterioration. The physician has diagnosed the infant with Tay-Sachs disease. The parents are asking the nurse why this happened. The nurse will base her answer knowing the root cause of Tay-Sachs is

C) hypoxia caused by placing the infant on their abdomen during sleep.

One of the most reliable predictors for worsening autosomal dominant polycystic kidney disease is

C) urine albumin excretion (UAE).

The nurse is caring for an adult client with growth hormone deficiency. When performing an assessment of this client, which system should the nurse be sure to assess for complications related to this disorder?

Cardiovascular system

An adult client with growth-hormone deficiency related to hypopituitarism has been taking replacement therapy for several months. The client informs the nurse that she is having pain in the hand and wrist almost constantly. What does the nurse understand is a common side effect of this therapy that seems to have affected this client?

Carpal tunnel syndrome

A patient who has had an intestinal bypass has developed a kidney stone. Which of the following types of kidney stone does the nurse recognize that this patient will most likely be treated for? a) Cystine b) Uric acid c) Calcium d) Magnesium ammonium

Calcium Most kidney stones are calcium stones-calcium oxalate, calcium phosphate, or a combination of the two materials. Patients who have had intestinal bypass surgery are at a higher risk for developing calcium kidney stones.

A community health care nurse is teaching a group of female high school students about the importance of regular Papanicolaou (Pap) smears. The nurse recognizes that what fact underlies the rationale for this teaching?

Cancer of the uterine cervix develops incrementally at a cellular level.

For which client condition will a pulse oximeter reading be inaccurate?

Carbon monoxide poisoning

Which complication of acromegaly can be life threatening?

Cardiac structures increase in size

A patient is admitted to the hospital in Addisonian crisis one month after a diagnosis of Addison's disease. The nurse knows which of the following clinical manifestations would support this diagnosis?

Change in level of consciousness and profound hypotension

The nurse determines that the client has clubbing of the fingertips. Which is the best intervention?

Check the patient's O2 saturation level.

The nurse determines that the client has clubbing of the fingertips. Which is the best intervention? a) Assess peripheral capillary refill. b) Check the patient's O2 saturation level. c) Call the health care provider. d) Monitor the client's heart rate.

Check the patient's O2 saturation level. Explanation: Clubbing of the nails indicates long-term hypoxia and should be evaluated.

The nurse determines that the client has clubbing of the fingertips. Which is the best intervention? a) Call the health care provider. b) Monitor the client's heart rate. c) Assess peripheral capillary refill. d) Check the patient's O2 saturation level.

Check the patient's O2 saturation level. Explanation: Clubbing of the nails indicates long-term hypoxia and should be evaluated.

Which of the following statements best captures the etiology of asthma?

Chemical mediators are released from presensitized mast cells as a consequence of exposure to an allergen.

Which of the following statements best captures the etiology of the acute response phase of extrinsic (atopic) asthma?

Chemical mediators are released from presensitized mast cells.

What etiologic determinants are important in the development of the pneumoconioses? (Select all that apply.)

Chemical nature of dust particle Size of dust particle Ability of particle to incite lung destruction

A patient is diagnosed with pulmonary embolism. Which of the following symptoms would most likely be present?

Chest pain and dyspnea

A patient is diagnosed with pulmonary embolism. Which of the following symptoms would most likely be present? a) Shallow respirations and wheezing b) Chest pain and dyspnea c) Cough and crackles d) Left arm pain and diaphoresis

Chest pain and dyspnea Explanation: The manifestations of pulmonary embolism depend on the size and location of the obstruction. Chest pain, dyspnea, and increased respiratory rate are the most frequent signs and symptoms of pulmonary embolism

A client with hypercapnia needs to increase calorie intake without increasing work of breathing. What menu selection is the best choice?

Chicken salad, refried beans, and guacamole

A client with hypercapnia needs to increase calorie intake without increasing work of breathing. What menu selection is the best choice? a) Chicken salad, refried beans, and guacamole b) Grilled salmon, baked potato, and bread pudding c) Mixed green salad, corn muffin, and iced tea d) Pasta with meat sauce, garlic bread, and carrots

Chicken salad, refried beans, and guacamole Explanation: Dietary fat metabolism produces the least amount of carbon dioxide (CO2) in relation to the amount of oxygen used, and supplies 9 calories per gram. Proteins produce more CO2 and supply 4 calories per gram. Carbohydrate metabolism produces the most CO2 and supplies 4 calories per gram. So the client should eat more fats and proteins while reducing carbohydrate intake

Which condition places the client at risk for hypercapnia if given high-flow oxygen as treatment for a ventilation perfusion mismatch?

Chronic bronchitis from smoking

Chronic obstructive pulmonary disease (COPD) is a combination of disease processes. What disease processes have been identified as being part of COPD?

Chronic obstructive bronchitis and emphysema

For a client with which of the following is the use of oxygen at a rate above 2 L/minute inappropriate?

Chronic obstructive pulmonary disease (COPD), sitting in bed with cyanosis and pulse oximeter reading 57%

A middle-aged client with a 30-year history of smoking was diagnosed with lung cancer. A health history revealed previous exposure to air pollution, asbestos, and radiation. Which of the following most likely had the greatest impact on development of the lung cancer? a) Asbestos b) Radiation c) Cigarette smoke d) Air pollution

Cigarette smoke Explanation: Cigarette smoking causes more than 80% of cases of lung cancer. The risk for lung cancer among cigarette smokers increases with duration of smoking and the number of cigarettes smoked per day. Cigarette smokers can benefit at any age from smoking cessation. Industrial hazards also contribute to the incidence of lung cancer. A commonly recognized hazard is exposure to asbestos. The mean risk for lung cancer is significantly greater in asbestos workers compared to the general population. In addition, tobacco smoke contributes heavily to the development of lung cancer in people exposed to asbestos.

The parents of an active 9-year-old are leaving the emergency department (ED) following cast placement for their son's lower extremity as a result of a fall in soccer. The nurse should emphasize that assessment of the cast is important and that they should bring their son back to the ED if they note which of the following signs and symptoms? Select all that apply. Cold, pale toes on the side with the cast. Patient complaining of tingling and numbness in the casted leg. Swelling in the lower leg has remained the same since they left the ED 12 hours ago. Pulses are palpable below the level of the cast. Bruising noted in the lower foot and toes.

Cold, pale toes on the side with the cast. Patient complaining of tingling and numbness in the casted leg.

A patient has been diagnosed with a brain tumor that cannot be removed surgically. During each office visit, the nurse will be assessing the patient for syndrome of inappropriate antidiuretic hormone (SIADH). Which of the following assessments would alert the clinic nurse that the patient may be developing this complication?

Complaints that his urine output is decreased, no edema noted in ankles, and increasing headache

A patient has been diagnosed with a brain tumor that cannot be removed surgically. During each office visit, the nurse will be assessing the patient for syndrome of inappropriate antidiuretic hormone (SIADH). Which of the following assessments would alert the clinic nurse that the patient may be developing this complication? Complaints that his urine output is decreased, no edema noted in ankles, and increasing headache Elevated blood glucose levels, dry mucous membranes, and severe projectile vomiting Fever, diarrhea, and nausea Muscle cramps, pins and needle sensation around the mouth/lips, and unexplained bruising

Complaints that his urine output is decreased, no edema noted in ankles, and increasing headache

A client has developed chronic hypoxia and has developed pulmonary hypertension (HTN). The nurse recognizes that the most likely cause of pulmonary hypertension would be:

Constriction of the pulmonary vessels in response to hypoxemia

A client has developed chronic hypoxia and has developed pulmonary hypertension (HTN). The nurse recognizes that the most likely cause of pulmonary hypertension would be: a) Constant dilation of the pulmonary vessels in response to hypoxia b) Hardening of the pulmonary vessels due to increased fat deposits c) Decreased vascular resistance in the pulmonary vessels d) Constriction of the pulmonary vessels in response to hypoxemia

Constriction of the pulmonary vessels in response to hypoxemia Explanation: Pulmonary HTN occurs as a result of chronic hypoxia. In response to hypoxia, the pulmonary vessels constrict. The pulmonary vessels differ from the systemic circulation vessels, which dilate in response to hypoxia and hypercapnia. Smooth muscle hypertrophy and proliferation of the vessel intima occur in pulmonary HTN.

The nurse is assessing a client whose recent health history is suggestive of sarcoidosis. What assessment should the nurse prioritize?

Constructing a genogram from the client's report of her family history

A client with a history of chronic pyelonephritis has been admitted several times with recurrent bacterial infection of the urinary tract. The nurse should anticipate educating this client with regard to which common treatment regimen? a) Increase intake of cranberry juice to 2 L/day. b) Continue taking antibiotics for full 10 to 14 days even if symptoms of infection disappear. c) Take prescribed diuretics early in the day to avoid having to get up during the night. d) Force micturition every 2 hours while awake.

Continue taking antibiotics for full 10 to 14 days even if symptoms of infection disappear. Chronic pyelonephritis involves a recurrent or persistent bacterial infection superimposed on urinary tract obstruction, urine reflux, or both. Chronic obstructive pyelonephritis can be bilateral, caused by conditions that obstruct bladder outflow; or unilateral, such as occurs with ureteral obstruction. Cranberry juice, forced micturition, and diuretics are not standard treatments for chronic pyelonephritis.

A client with primary lung disease has developed right heart failure. The health care provider would document this as:

Cor pulmonale

The nurse is preparing a client for a test that will measure negative feedback suppression of ACTH. Which medication will the nurse administer in conjunction for this test?

Dexamethasone

A client with primary lung disease has developed right heart failure. The health care provider would document this as: a) Cor pulmonale b) Adult respiratory distress syndrome c) Primary hypertension d) Cardiac tamponade

Cor pulmonale Explanation: The term cor pulmonale refers to right heart failure resulting from primary lung disease or pulmonary hypertension. The increased pressures and work result in hypertrophy and eventual failure of the right ventricle. The manifestations of cor pulmonale include the signs and symptoms of the primary lung disease and the signs of right-sided heart failure. Primary hypertension is elevated blood pressure of unknown cause. Cardiac tamponade is pressure on the heart that occurs when blood or fluid builds up in the space.

What intervention is appropriate for a client with sarcoidosis?

Corticosteroids

What intervention is appropriate for a client with sarcoidosis? a) Antifungal agents b) Corticosteroids c) Antihistamines d) Lymph node radiation

Corticosteroids Explanation: Sarcoidosis treatment is directed at interrupting the inflammatory process with corticosteroid medications. Sarcoidosis is overgrowth of granulomas and does not have a fungal or allergy component

A client is diagnosed with adrenocorticotropic hormone deficiency (ACTH) and is to begin replacement therapy. Regarding which type of replacement will the nurse educate the client?

Cortisol replacement therapy.

The newborn-nursery nurse is obtaining a blood sample to determine if a newborn has congenital hypothyroidism. What long-term complication is the nurse aware can occur if this test is not performed and the infant has congenital hypothyroidism?

Cretinism

A 28-year-old man presents with complaints of diarrhea, fecal urgency, and weight loss. His stool is light colored and malodorous, and it tends to float and be difficult to flush. He has also noted tender, red bumps on his shins and complains of pain and stiffness in his elbows and knees. Sigmoidoscopy reveals discontinuous, granulomatous lesions; no blood is detected in his stool. Which of the following diagnoses would his care team first suspect? Crohn disease Ulcerative colitis Diverticulitis Colon cancer

Crohn disease

Which disorder is a result of excess cortisol?

Cushing syndrome

A patient with chronic obstructive pulmonary disease presents to the emergency department with difficulty breathing. Physical examination reveals bluish skin, nail beds, and mucous membranes. Which of the following terms correctly describes this condition?

Cyanosis

What type of diabetes results from inadequate or defective insulin receptors? Diabetes mellitus 1A Diabetes mellitus 1B Diabetes mellitus 2 Diabetes insipidus

Diabetes mellitus 2

A patient with chronic obstructive pulmonary disease presents to the emergency department with difficulty breathing. Physical examination reveals bluish skin, nail beds, and mucous membranes. Which of the following terms correctly describes this condition? a) Ischemia b) Cyanosis c) Hemoptysis d) Gematemesis

Cyanosis Explanation: Cyanosis refers to the bluish discoloration of the skin and mucous membranes that result from an excessive concentration of reduced or deoxygenated hemoglobin in the small blood vessels. It usually is most marked in the lips, nail beds, ears, and cheeks. The degree of cyanosis is modified by the amount of cutaneous pigment, skin thickness, and state of the cutaneous capillaries.

Which of the following best describes nephronophthisis-medullary cystic kidney disease? a) Small elongated cysts form in the collecting ducts and maintain contact with the nephron of origin. b) Cysts are restricted to the corticomedullary border. c) Cysts develop in the kidney as a consequence of aging, dialysis, or other conditions that affect tubular function. d) The tubule wall, which is lined by a single layer of tubular cells, expands and then rapidly closes the cyst off from the tubule of origin.

Cysts are restricted to the corticomedullary border. Nephronophthisis and adult-onset medullary cystic kidney disease both produce progressive medullary tubulointerstitial cystic disease. The other conditions described are elsewhere in the renal space.

A diabetes education nurse is teaching a group of recently diagnosed diabetics about the potential genitourinary complications of diabetes and the consequent importance of vigilant blood glucose control. Which of the following teaching points best conveys an aspect of bladder dysfunction and diabetes mellitus? A) "People with diabetes are highly susceptible to urethral obstructions, and these can heal more slowly and cause more damage than in people without diabetes." B) "High blood sugar results in a high glucose level in your urine, and this can make your bladder muscle less able to fully empty the bladder." C) "Many people with diabetes find it necessary to live with an indwelling catheter to ensure their bladders do not become too full." D) "It's important for you to empty your bladder frequently because diabetes carries risks of kidney damage that can be exacerbated by incomplete bladder emptying."

D) "It's important for you to empty your bladder frequently because diabetes carries risks of kidney damage that can be exacerbated by incomplete bladder emptying." Diabetics are vulnerable to peripheral neuropathies that can be somewhat mitigated by regular voiding; they are also especially vulnerable to renal damage from high blood sugars, a situation that is worsened when accompanied by incomplete bladder emptying. Urethral obstructions are not a noted complication of diabetes, and indwelling catheter placement is not normally necessary. High blood sugars do not necessarily yield high-glucose urine, and the bladder deficits associated with diabetes are neurological in nature rather than a result of particular urine chemistry.

When explaining a cystometry test to measure bladder pressure during filling and voiding in a normal adult, the nurse informs the nursing students that the normal capacity when adults have a desire to void is A) 100 to 150 mL. B) 200 to 250 mL. C) 300 to 399 mL. D) 400 to 500 mL.

D) 400 to 500 mL. The desire to void occurs when the bladder is full (normal capacity is approximately 400 to 500 mL). At this point, a definite sensation of fullness occurs; the pressure rises sharply to 40 to 100 cm H2O; and voiding occurs around the catheter.

A middle-aged man with diabetes reports that he must strain to urinate and that his urine stream is weak and dribbling. He also reports feeling that his bladder never really empties. The nurse knows that all of his complaints are likely caused by which of the following medical diagnoses? A) Detrusor muscle areflexia B) Detrusor-sphincter dyssynergia C) Uninhibited neurogenic bladder D) Bladder atony with dysfunction

D) Bladder atony with dysfunction Diabetes causes peripheral neuropathy, which can affect the sensory axons of the urinary bladder. Bladder atony with dysfunction is a frequent complication of diabetes mellitus.

A patient asks the nurse what it means when the doctor said that he had adenocarcinoma of the bladder. Reviewing the pathophysiologic principles behind this type of cancer, the nurse knows A) it is a low-grade tumor that is readily cured with bladder surgery. B) after resection of the cancer, the prognosis is excellent with this type of cancer cell. C) that these types of cancer cells are very invasive to the tissue; therefore, the entire bladder must be removed. D) this is a rare but highly metastatic tumor that has a very poor prognosis.

D) this is a rare but highly metastatic tumor that has a very poor prognosis. Adenocarcinoma is rare and highly metastatic. Answer choices A and B relate to urothelial carcinoma; answer choice C relates to squamous cell carcinoma.

10. A 42-year-old man has been diagnosed with renal failure secondary to diabetes mellitus and is scheduled to begin dialysis soon. Which of the following statements by the client reflects a sound understanding of the process of dialysis? A) "It's stressful knowing that committing to dialysis means I can't qualify for a kidney transplant." B) "I know I'll have to go to a hospital or dialysis centre for treatment." C) "Changing my schedule to accommodate 3 or 4 hours of hemodialysis each day will be difficult." D) "If I choose hemodialysis, I won't be able to go about my normal routine during treatment like I will with peritoneal dialysis."

D) "If I choose hemodialysis, I won't be able to go about my normal routine during treatment like I will with peritoneal dialysis."

18. A 45-year-old Caucasian woman is being treated for ovarian cancer. Her treatment involves the chemotherapy agent, cisplatin. What should the oncologist be wary of with this treatment? A) Glomerulonephritis B) Chronic renal failure C) Anemia D) Nephrotoxic acute tubular necrosis

D) Nephrotoxic acute tubular necrosis

A 79-year-old female resident of an assisted living facility receives care from a community nurse on a regular basis for treatment of a chronic venous leg ulcer. Which of the following factors would the nurse be most justified in ruling out as a contributing factor to the client's impaired wound healing?

Decreased antibody levels

A nurse in the emergency department admits a male client who has experienced severe frostbite to his hands and toes after becoming lost on a ski hill. The nurse recognizes that which of the following phenomena has contributed to his tissue damage?

Decreased blood flow has induced hypoxia.

A 22-year-old woman has sustained a lateral blow to her right knee during a game, and subsequent imaging has confirmed a severe meniscus injury. Which of the following consequences should the woman expect over the course of recovery? Select all that apply. Decreased lubrication in the knee joint Decreased stability of the knee Reduced shock absorption capacity Presence of infectious microorganisms in the synovial capsule Rapid restoration of the cartilage cells in the meniscus

Decreased lubrication in the knee joint Decreased stability of the knee Reduced shock absorption capacity

Which of the following diagnoses will present the highest risk for hypercapnic/hypoxemic respiratory failure?

Drug overdose

When developing the plan of care for a child with respiratory distress, what nursing intervention can the nurse provide that will facilitate respiratory movements and avoid airway compression? a) Handle the child frequently to keep her stimulated to breathe. b) Administer antibiotics. c) Elevate the head of the bed. d) Administer intravenous fluids.

Elevate the head of the bed. Explanation: Elevation of the head facilitates respiratory movements and avoids airway compression. Handling is kept at a minimum to avoid tiring. Administration of IV fluids will prevent dehydration from insensible loss from rapid respirations but will not facilitate respiratory movements or prevent airway compression. Because the infection is viral, antibiotics are not effective and are given only for a secondary bacterial infection.

Which laboratory findings should you expect to see in a patient diagnosed with nephritic syndrome? a) Decreased high-density lipoproteins and increased iron levels b) Low triglycerides and elevated sodium levels c) Elevated urine protein and hypoalbuminemia d) Abnormal blood clotting and elevated blood pressure

Elevated urine protein and hypoalbuminemia The nephritic syndrome is due to glomerular disease that is usually of acute onset and is accompanied by grossly visible hematuria, mild to moderate proteinuria, and hypertension. The nephrotic syndrome, also due to glomerular disease, is characterized by heavy proteinuria, hypoalbuminemia, and severe edema.

Congenital adrenal hyperplasia is a congenital disorder in which a deficiency exists in any of the enzymes necessary for the synthesis of cortisol. Infants of both sexes are affected, although boys are not diagnosed at birth unless of enlarged genitalia. Female infants often have ambiguous genitalia because of the oversecretion of adrenal androgens. What are the manifestations of the ambiguous genitalia caused by congenital adrenal hyperplasia?

Enlarged clitoris, fused labia, and urogenital sinus

What nursing strategy implemented after a patient is admitted to the CCU will decrease the risk that he or she will develop hospital-acquired pneumonia?

Ensure that all members of the health care team follow hand hygiene protocols strictly.

Which diagnosis places a child at the greatest risk for airway obstruction? a) Bronchiolitis b) Coup c) Syncytial virus (RSV) d) Epiglottitis

Epiglottitis Explanation: The child with epiglottitis is at risk for airway obstruction. Epiglottitis is a life-threatening supraglottic infection that may cause airway obstruction and asphyxia. The child with bronchiolitis is at risk for respiratory failure resulting from impaired gas exchange. Acute bronchiolitis is a viral infection of the lower airways, most commonly caused by the respiratory syncytial virus (RSV). The symptoms of coup usually subside when the child is exposed to moist air.

A man with chronic alcoholism is diagnosed is acute respiratory distress syndrome (ARDS) due to alcohol abuse. Assessment findings include: diffuse bilateral infiltrates of the lung tissue and marked hypoxemia. What is the primary physiologic change that results in his ARDS?

Epithelial injury with increased permeability of the alveolar--capillary membrane

Parents of a 20-month-old infant report that he refuses food or eats poorly and that he grimaces when he swallows. He also is irritable and cries a lot. The mother is worried that he ate something inappropriate this morning, because he vomited something that looked like coffee grounds. Which of the following health problems would the care team first suspect? Rotavirus infection Appendicitis Esophagitis from gastrointestinal reflux Hirschsprung disease

Esophagitis from gastrointestinal reflux

Which of the following pain descriptions would lead the nurse to suspect the client is experiencing ureteral colic?

Excruciating pain in the flank and upper outer quadrant of the abdomen that radiates to the bladder area

Which of the following pain descriptions would lead the nurse to suspect the client is experiencing ureteral colic? Right upper quadrant pain that worsens with deep breaths and palpation Excruciating pain in the flank and upper outer quadrant of the abdomen that radiates to the bladder area Pain described as "fire poking in their side," pulsating with every heart beat but decreases when in fetal position Perineal pain that increases when urinating and then lessens until the time to urinate again

Excruciating pain in the flank and upper outer quadrant of the abdomen that radiates to the bladder area

Which of the following phenomena best accounts for the increased presence of leukocytes at the site of inflammation?

Existing leukocytes stick to the epithelial cells and move along blood vessel walls.

Which test can the nurse prepare the client for to determine the differentiation between a benign and a malignant thyroid disease?

Fine-needle aspiration biopsy

A nurse in an acute medical unit of a hospital has admitted a 62-year-old female from the emergency department who has been diagnosed with acute pyelonephritis. Which of the following statements most accurately conveys an aspect of the knowledge base that the nurse needs to perform adequate care and teaching?

Flank pain, dysuria, and nausea and vomiting are likely assessment findings

The nurse is caring for a child with congenital adrenal hyperplasia and has depletion of sodium levels. What pharmacological therapy does the nurse anticipate administering to this child?

Fludrocortisone acetate

A client is brought to the emergency department with complaints of shortness of breath. Assessment reveals a full, bounding pulse, severe edema, and audible crackles in lower lung fields bilaterally. What is the client's most likely diagnosis?

Fluid volume excess

A client is seeking treatment for infertility. What decrease in hormone secretion from the anterior lobe of the pituitary gland that regulates fertility would the nurse recognize may cause this issue?

Follicle stimulating hormone (FSH)

The health care provider suspects a newly admitted client may have a hemothorax. The client most likely experienced:

Fractured or dislocated ribs

The health care provider suspects a newly admitted client may have a hemothorax. The client most likely experienced: a) Pneumonia b) Congestive heart failure c) Renal failure d) Fractured or dislocated ribs

Fractured or dislocated ribs Explanation: Hemothorax is a collection of blood in the thoracic cavity. Bleeding may arise from chest injury, a complication of chest surgery, malignancies, or rupture of a great vessel such as an aortic aneurysm. Fractured and dislocated ribs alert the nurse to the presence of a possible chest injury. Pleural effusion refers to an abnormal collection of fluid in the pleural cavity and is associated with diagnoses of congestive heart failure, pneumonia, and renal failure

Advances in technology have made it possible to assess hypothalamic-pituitary function by newly developed imaging and radioimmunoassay methods. When baseline tests are not sufficient, what suppression test gives information about combined hypothalamic-pituitary function?

GH suppression test

You are volunteering in the medical tent of a road race on a hot, humid day. A runner who has collapsed on the road is brought in with the following symptoms: sunken eyes, a body temperature of 100°F, and a complaint of dizziness while sitting to have his blood pressure taken (which subsides upon his lying down). These are signs of a fluid volume deficit. Which of the following treatments should be carried out first?

Give him an electrolyte solution by mouth

A client with Graves' disease has had radioiodine treatment with worsening of ophthalmopathy. What medication does the nurse prepare to administer that the client will use for several weeks to decrease these symptoms?

Glucocorticoids

A patient has passed a kidney stone composed of uric acid (urate). Which of the following pathological conditions is a contributing factor for the development of this type of kidney stone? a) Arteriosclerosis b) Atherosclerosis c) Gout d) Sickle-cell disease

Gout Uric acid stones develop in conditions of gout and high concentrations of uric acid in the urine.

A 25-year-old female client exhibits exophthalmos of both eyes. The health care provider recognizes this as a manifestation of:

Graves disease

Acute postinfectious glomerulonephritis, as its name implies, follows an acute infection somewhere else in the body. What is the most common cause of acute postinfectious glomerulonephritis? a) E. coli b) S. aureus c) Group A Β-hemolytic streptococci d) P. aeruginosa

Group A Β-hemolytic streptococci Group A Β-hemolytic streptococci have the ability to seed from one area of the body to another. One area it seeds to is the kidney, where it causes acute postinfectious glomerulonephritis. Other organisms can cause acute postinfectious glomerulonephritis, but they are not the most common cause of the disease.

A client is having radiation prior to having a pituitary tumor removed. Which loss of pituitary hormone is typically the first to be lost?

Growth hormone

When the assessment of thyroid autoantibodies is performed, what is the suspected diagnosis?

Hashimoto thyroiditis

The nurse is performing an assessment for a client who has hyperthyroidism that is untreated. When obtaining vital signs, what is the expected finding?

Heart rate 110 and bounding

A 7-year-old boy is admitted to the hospital with a suspected diagnosis of lead toxicity. Which of the following assessment findings is most congruent with the client's diagnosis?

Hemoglobin 9.9 g/dL

A 77-year-old female diagnosed with chronic obstructive pulmonary disease (COPD) is experiencing impaired gas exchange and CO2 retention, despite a rapid respiratory rate. Which of the following pathophysiological principles would her health care team expect if her compensatory mechanisms are working?

Her kidneys will adapt with an increase in plasma HCO3- and her pH will decrease.

A 55-year-old woman has presented to the emergency department following a panic attack. Her blood pressure, respiratory rate, and heart rate are all highly elevated, while her temperature and oxygen saturation are within normal ranges. What is the woman's body most likely doing to address the changes in pH associated with her situation?

Her kidneys will limit the amount of bicarbonate that they reabsorb.

A deficiency in which of the following would result in an inhibition of the inflammatory response?

Histamine

Which factors increase the risk that a client will develop chronic obstructive pulmonary disease (COPD)? Select all that apply.

History of tobacco use History of asthma

A 9-year-old boy has been diagnosed with bronchial asthma. Which factor is most likely contributing to his condition?

House dust

Following a visit to her campus medical clinic motivated by persistent abdominal pain and dyspareunia, a 20-year-old female college student has been referred for a diagnostic workup to rule out pelvic inflammatory disease. Her elevated white cell and C-reactive protein levels lead her care provider to suspect pelvic inflammatory disease (PID). What follow-up question is most likely to help with the differential diagnosis? Are you using tampons during your period or do you normally use pads?" "What does your daily hygiene routine usually consist of?" "How many sexual partners have you had?" "Have you ever had a therapeutic abortion in the past?"

How many sexual partners have you had?

A nurse is caring for a patient who has a recent history of passing calcium urinary stones. Which of the following is a priority nursing consideration for this patient? a) Bowel elimination b) Skin care c) Bed rest d) Hydration

Hydration A major goal of treatment in people who have passed kidney stones or have had them removed is to prevent their recurrence. Adequate fluid intake reduces the concentration of stone-forming crystals in the urine and needs to be encouraged.

The accumulation of fluid inside of the scrotum is called: Spermatocele Varicocele Hydrocele Enterocele

Hydrocele

The nurse is teaching a client diagnosed with Addison disease about the importance of lifetime oral replacement therapy. Select the pharmacologic agent that would be prescribed.

Hydrocortisone

A 73-year-old man presents to his family physician with complaints of recent urinary hesitation and is eventually diagnosed with Benign Prostatic hyperplasia (BPH). Which of the following clinical consequences would his care provider expect prior to the resolution of his health problem? 1.Hydroureter and pain 2. development of renal calculi and renal cysts 3. Unilateral hydronephrosis and pain 4. Development of glomerulonephritis or nephrotic syndrome

Hydroureter and pain

A 73-year-old man presents to his family physician with complaints of recent urinary hesitation and is eventually diagnosed with benign prostatic hyperplasia (BPH). Which of the following clinical consequences would his care provider expect prior to the resolution of his health problem?

Hydroureter and pain

A 73-year-old man presents to his family physician with complaints of recent urinary hesitation and is eventually diagnosed with benign prostatic hyperplasia (BPH). Which of the following clinical consequences would his care provider expect prior to the resolution of his health problem? Hydroureter and pain Development of renal calculi and renal cysts Unilateral hydronephrosis and pain Development of glomerulonephritis or nephrotic syndrome

Hydroureter and pain

A 68-year-old woman with a new onset of vascular dementia has recently begun retaining urine. Which of the following physiological phenomena would her care providers most realistically expect to currently occur as a result of her urinary retention? Hypertrophy of the bladder muscle and increased bladder wall thickness Decreased urine production and nitrogenous waste excretion by the kidneys Decompensation, bladder stretching, and high residual urine volume Overflow incontinence and loss of contraction power

Hypertrophy of the bladder muscle and increased bladder wall thickness

An asthmatic patient comes to the emergency department very anxious due to increasing shortness of breath. Physical assessment reveals tachypnea, and an arterial blood gas shows decreased carbon dioxide levels and hypoxemia. Which of the following is the most likely cause?

Hyperventilation

A major manifestation of Cushing syndrome includes which of the following?

Hypokalemia

While rock climbing, a 22 year-old male has endured a severe head injury. Which of the following statements best captures expected clinical manifestations and treatments for his immediate condition?

Hypoventilation may exist, resulting in increased PCO2 and hypoxemia that may require mechanical ventilation.

While rock climbing, a 22 year-old male has endured a severe head injury. Which of the following statements best captures expected clinical manifestations and treatments for his immediate condition? a) Oxygen therapy is likely to decrease his respiratory drive and produce an increase in PCO2. b) Hypoventilation may exist, resulting in increased PCO2 and hypoxemia that may require mechanical ventilation. c) Cheyne-Stokes breathing is likely but will respond to bronchodilators. d) The client is unlikely to respond to supplementary oxygen therapy due to impaired diffusion.

Hypoventilation may exist, resulting in increased PCO2 and hypoxemia that may require mechanical ventilation. Explanation: Brain injuries and accompanying hypoventilation are often associated with increased PCO2 and by hypoxemia that responds to oxygen therapy. Persons with COPD are more vulnerable to diminished respiratory drive secondary to oxygen therapy, while Cheyne-Stokes breathing is not identified as a likely consequence of brain injury. Impaired alveolar diffusion is not an aspect of the client's injury.

A patient diagnosed with congestive heart failure has an arterial blood oxygen level (PaO2) of 89%. Which of the following terms should the nurse use to document a reduced oxygenation of arterial blood?

Hypoxemia

The nurse is caring for a client diagnosed with pneumonia. The client's arterial blood gas results identify decreased level oxygen in the arterial blood and a decreased in the partial pressure of oxygen. The nurse would interpret this as:

Hypoxemia

The nurse is caring for a client diagnosed with pneumonia. The client's arterial blood gas results identify decreased level oxygen in the arterial blood and a decreased in the partial pressure of oxygen. The nurse would interpret this as: a) Hypoxemia b) Hypoxia c) Atelectasis d) Hypercapnia

Hypoxemia Explanation: Hypoxemia refers to a reduction in PO2 of the arterial blood. Hypoxemia can result from an inadequate amount of O2 in the air, disease of the respiratory system, dysfunction of the neurologic system, or alterations in circulatory function. Hypercapnia is an increase in the carbon dioxide content of the arterial blood. Atelectasis is collapse of a segment of the lung due to airway obstruction

A patient diagnosed with congestive heart failure has an arterial blood oxygen level (PaO2) of 89%. Which of the following terms should the nurse use to document a reduced oxygenation of arterial blood? a) Ischemia b) Hypoxemia c) Hypocapnia d) Ventilation

Hypoxemia Explanation: Hypoxemia refers to a reduction in arterial blood O2 levels, which is considered a PaO2 less than 95 mm Hg. Hypoxemia can result from an inadequate amount of O2 in the air, a disorder of the respiratory system, dysfunction of the neurologic system, or alterations in circulatory function.

Which of the following clients are displaying known risk factors for the development of pulmonary emboli? A client who is: (Select all that apply)

Immobilized following orthopedic surgery A smoker and who takes oral contraceptives

What is the underlying cause of respiratory failure in a child with bronchiolitis? a) Metabolic acidosis b) Impaired gas exchange c) Ischemia d) Obstructive process

Impaired gas exchange Explanation: The child with bronchiolitis is at risk for respiratory failure resulting from impaired gas exchange. The other answers are not applicable.

Following the identification of low blood levels of cortisol and low 24-hour urinary free cortisol, a 51-year-old female client has been diagnosed with a primary adrenal cortical insufficiency. Which of the following health consequences would be attributable to her low levels of cortisol?

Impaired immunological and inflammatory response

A patient is brought into the emergency department with severe crushing injuries to the chest wall and signs of respiratory failure following a motor vehicle accident. Which of the following laboratory values would be expected?

Increase in PCO2

A patient is brought into the emergency department with severe crushing injuries to the chest wall and signs of respiratory failure following a motor vehicle accident. Which of the following laboratory values would be expected? a) Metabolic alkalosis b) Cool moist skin c) Increase in PCO2 d) Hyperventilation

Increase in PCO2 Explanation: Hypoventilation or ventilatory failure occurs when the volume of "fresh" air moving into and out of the lung is significantly reduced. It is commonly caused by conditions outside the lung such as depression of the respiratory center, diseases of the nerves supplying the respiratory muscles, or thoracic cage disorders such as a crushed chest. Hypoventilation almost always causes an increase in PCO2. The PCO2 level is a good diagnostic measure for hypoventilation. Additional indicators of hypercapnia are warm and flushed skin and hyperemic conjunctivae

When the nurse is performing a health history for a client who is being admitted for hyperthyroidism, what symptoms does the client report that the nurse would find associated with this disorder?

Increase in appetite

The nurse is assessing a patient who has a unilateral obstruction of the urinary tract. The nurse anticipates the patient may develop: a) Excretion of dilute urine b) Increase in blood pressure c) Inability to control urination d) Increased urine output

Increase in blood pressure Hypertension is an occasional complication of urinary tract obstruction. It is more common in cases of unilateral obstruction in which renin secretion is enhanced, probably secondary to impaired renal blood flow. In these circumstances, removal of the obstruction often leads to a reduction in blood pressure. The urine output would be decreased and not diluted

When educating a patient with a wound that is not healing, the nurse should stress which of the following dietary modifications to ward off some of the negative manifestations that can occur with inflammation?

Increase your intake of oily fish and fish oil so that you will increase absorption of omega-3 polyunsaturated fatty acids.

Which alteration in acid-base balance could be expected as a compensatory response for a client with severe chronic respiratory impairment?

Increased arterial blood bicarbonate

Which alteration in acid-base balance could be expected as a compensatory response for a client with severe chronic respiratory impairment? a) Decreased arterial blood carbon dioxide b) Decreased arterial blood pH c) Increased arterial blood oxygen d) Increased arterial blood bicarbonate

Increased arterial blood bicarbonate Explanation: Hypercapnia (elevated CO2 levels) develops as a result of chronic respiratory impairment. This produces a state of respiratory acidosis. The compensatory mechanism is for the kidneys to retain bicarbonate ions to balance the increased acidity

What is a predominant effect of a prolonged excessive growth hormone level?

Increased blood glucose levels

As a consequence of a long-standing lung disease, a client is in a chronic state of hypoxia. Which of the following phenomena would the client's care team be most justified in anticipating?

Increased erythropoietin production

A client comes to the clinic with fatigue and muscle weakness. The client also states she has been having diarrhea. The nurse observes the skin of the client has a bronze tone and when asked, the client says she has not had any sun exposure. The mucous membranes of the gums are bluish-black. When reviewing laboratory results from this client, what does the nurse anticipate seeing?

Increased levels of ACTH

Which of the following data would a clinician consider as most indicative of acute renal failure?

Increased nitrogenous waste levels; decreased glomerular filtration rate (GFR)

A child has developed respiratory stridor and is displaying a crowing sound. The parents ask the nurse what is causing this sound. The best response would be: a) Nasal obstruction and inhalation occurring through the mouth rather than the nares b) Increased turbulence of air moving through the obstructed airways c) The collapse of intrathoracic airways because of air trapping d) Pulling in of the soft tissue surrounding the cartilaginous and bony thorax

Increased turbulence of air moving through the obstructed airways Explanation: The crowing sound of stridor occurs with the increased turbulence of air moving through the obstructed airways. Pulling in of the soft tissue surrounding the cartilaginous and bony thorax describes the mechanisms that produce inspiratory retractions. The collapse of intrathoracic airways because of air trapping describes the mechanism for the production of wheezing.

Following the identification of low levels of T3 and T4 coupled with the presence of a goiter, a 28-year-old female has been diagnosed with Hashimoto's thyroiditis. In light of this diagnosis, which of the following assessment results would constitute an unexpected finding?

Increased white cell count and audible crackles on chest auscultation.

A patient is to receive a radiocontrast media as part of a diagnostic scan. Which of the following is intended to reduce the nephrotoxic effects of the radiocontrast media? a) Administering ibuprophen 600 mg prior to the procedure b) Administering one unit of packed red blood cells c) Having the patient take nothing by mouth d) Increasing the normal saline intravenous infusion rate to 125 mL/hour

Increasing the normal saline intravenous infusion rate to 125 mL/hour Some drugs, such as high-molecular-weight radiocontrast media, the immunosuppressive drugs cyclosporine and tacrolimus and nonsteroidal anti-inflammatory drugs can cause acute prerenal failure by decreasing renal blood flow. Administering intravenous saline can improve hydration and renal perfusion to decrease the toxic effects of the radiocontrast media.

Pleuritis, an inflammatory process of the pleura, is common in infectious processes that spread to the pleura. Which are the drugs of choice for treating pleural pain?

Indomethacin

Pleuritis, an inflammatory process of the pleura, is common in infectious processes that spread to the pleura. Which are the drugs of choice for treating pleural pain? a) Indomethacin b) Warfarin c) Inderal d) Acetaminophen

Indomethacin Explanation: Treatment of pleuritis consists of treating the underlying disease and inflammation. Analgesics and nonsteroidal anti-inflammatory drugs (e.g., indomethacin) may be used for pleural pain. Although these agents reduce inflammation, they may not entirely relieve the discomfort associated with deep breathing and coughing. The other answers are not used to treat pleural pain

Which one of the following is the usual cause of acute pyelonephritis? a) Autoimmunity b) Obstruction c) Infection d) Reflux

Infection Gram-negative bacteria, including Escherichia coli and Proteus, Klebsiella, Enterobacter, and Pseudomonas species, are the most common causative agents for acute pyelonephritis.

A 23-year-old man has received a recent diagnosis of appendicitis following 24 hours of acute abdominal pain. The nurse providing care for the man is explaining that while unpleasant, the inflammation of his appendix is playing a role in his body's fight against the underlying infectious process. Which of the following teaching points should the nurse eliminate from his teaching to the patient?

Inflammation helps your body to produce the right antibodies to fight the infection."

A 22-year-old student has developed a fever and diarrhea while on a backpacking trip in Southeast Asia. His oral temperature is 101.4°F. The diarrhea is bloody, frequent, and small in volume. These clinical manifestations are sufficiently distressing that he is visiting a local medical clinic in the area. Which of the following diagnoses best characterizes this health problem? Noninflammatory diarrhea Inflammatory diarrhea Factitious diarrhea Secretory diarrhea

Inflammatory diarrhea

At an influenza vaccination clinic, a nurse is screening clients who are requesting the vaccine. The nurse determines that which of the following clients is unable to receive the vaccine? a) A 64-year-old female reporting a latex allergy b) An 82-year-old male with uncontrolled diabetes c) A 20-year-old healthy female reporting an allergy to eggs d) A 6-year-old male with a history of asthma

Influenza vaccines are contraindicated in persons with allergy to eggs or to other components of the vaccine, persons with a history of a Guillain-Barré syndrome, and persons with acute febrile illness.

Which of the following statements most accurately conveys an aspect of cell injury due to impaired calcium homeostasis?

Injured cells tend to accumulate calcium.

A client is brought to the emergency department and immediately diagnosed with a tension pneumothorax. The priority intervention would be:

Insertion of a large-bore needle or chest tube

A client is brought to the emergency department and immediately diagnosed with a tension pneumothorax. The priority intervention would be: a) Arterial blood gas analysis b) Chest x-ray c) Insertion of a large-bore needle or chest tube d) Administration of oxygen by face mask

Insertion of a large-bore needle or chest tube Explanation: Emergency treatment of tension pneumothorax involves the prompt insertion of a large-bore needle or chest tube into the affected side of the chest along with one-way valve drainage or continuous chest suction to aid in lung reexpansion. Other listed options may be implemented after the emergency measure.

Testing for short stature growth hormone (GH) problems can be done by pharmacologic means. Which of the following medications can be utilized to test for a rise in GH? Select all that apply Insulin Levodopa Persantine Dobutamine Sestamibi

Insulin Levodopa

Which is not a cause of icterus (jaundice)? Hemolytic anemia Hepatitis Pancreatic cancer Insulinoma

Insulinoma

Talc lung can occur from injected or inhaled talc powder that has been mixed with heroin, methamphetamine, or codeine as a filler. What are people with talc lung very susceptible to?

Interstitial lung disease

The management of cor pulmonale is directed at the underlying lung disease and heart failure. Why is low-flow oxygen therapy a part of the management of cor pulmonale?

It reduces pulmonary hypertension and polycythemia associated with chronic lung disease

The nurse teaches a client with a new diagnosis of asthma about measures to prevent asthma attacks. What should the nurse include in their teaching plan? Select all that apply.

Limit exposure to dust. Schedule the influenza vaccine. Use an inhaled cromolyn

A client with bipolar disorder has developed hypothyroidism with a goiter. When the nurse obtains a medication history, which medication taken by the client does the nurse understand may cause this disorder?

Lithium carbonate

A 16-year-old girl has broken her arm while snowboarding. When she arrives at hospital, she is shocked at the amount of swelling at the injury site. Which of the following statements best explains the physiological rationale for her swelling?

Loss of plasma proteins causes an increase in interstitial osmotic pressure.

Staghorn kidney stones, or struvite stones, are usually located in the renal pelvis. These stones are made from what? a) Uric acid b) Calcium oxalate c) Magnesium ammonium phosphate d) Cystine

Magnesium ammonium phosphate Phosphate levels are increased in alkaline urine and magnesium, always present in the urine, and combine to form struvite stones. These stones can increase in size until they fill an entire renal pelvis. Because of their shape, they often are called staghorn stones. The other minerals can form stones, but not staghorn stones.

A 61-year-old woman who has had an upper respiratory infection for several weeks has presented to her family physician with complaints of a recent onset of urinary retention. She reveals to her physician that she has been taking nonprescription cold medications over and above the suggested dose for the past 2 weeks. Which of the following phenomena will her physician most likely suspect is contributing to her urinary retention? A) Cholinergic actions of the cold medicine are triggering internal and external sphincter contraction. B) Antihistamine effects inhibit communication between the pons and the thoracolumbar cord. C) The anticholinergic effects of the medication are impairing normal bladder function. D) Over-the-counter medications such as cold medicine stimulate the parasympathetic nervous system and inhibit bladder emptying.

Many over-the-counter cold medications have an anticholinergic effect that interferes with normal bladder emptying. These effects on micturition are not a result of cholinergic actions or miscommunication between the pontine micturition center and the spinal cord. Stimulation of the parasympathetic nervous system would tend to increase rather than decrease bladder emptying.

Elderly people are very susceptible to pneumonia in all its varieties. The symptoms the elderly exhibit can be very different than those of other age groups who have pneumonia. What signs and symptoms are elderly people with pneumonia less likely to experience than people with pneumonia in other age groups? a) Deterioration in mental status b) Loss of appetite c) Pleuritic pain d) Marked elevation in temperature

Marked elevation in temperature Explanation: Pleuritic pain, a sharp pain that is more severe with respiratory movements, is common. With antibiotic therapy, fever usually subsides in approximately 48 to 72 hours, and recovery is uneventful. Elderly persons are less likely to experience marked elevations in temperature; in these persons, the only sign of pneumonia may be a loss of appetite and deterioration in mental status.

When the color of the feces is black (tarry) due to an upper digestive bleeding, it is named? Hematemesis Melena Hematochezia Hemoptysis

Melena

You are an emergency-room nurse caring for an elderly trauma patient. Your patient has the following arterial blood gas results: pH 7.26, PaCO2 28, HCO3 11 mEq/L. How would you interpret these results?

Metabolic acidosis with a compensatory respiratory alkalosis

A hospital patient's arterial blood gases indicate normal levels of oxygen and increased carbon dioxide. The patient's respiratory rate is 12 breaths/minute (normal 14 to 20 breaths/minute) with all other vital signs within normal range. While not evident from assessment and diagnostics, the patient's kidneys are minimizing both H+ excretion and HCO3- reabsorption. What is this client's most likely diagnosis? Respiratory alkalosis Metabolic acidosis Respiratory acidosis Metabolic alkalosis

Metabolic alkalosis

A hospital patient's arterial blood gases indicate normal levels of oxygen and increased carbon dioxide. The patient's respiratory rate is 12 breaths/minute (normal 14 to 20 breaths/minute) with all other vital signs within normal range. While not evident from assessment and diagnostics, the patient's kidneys are minimizing both H+ excretion and HCO3- reabsorption. What is this client's most likely diagnosis?

Metabolic alkalosis

The ICU nurse is caring for a patient who experienced trauma in a workplace accident. The patient is complaining of having trouble breathing with abdominal pain. An ABG reveals the following results: pH 7.28, PaCO2 50 mm Hg, HCO3- 23 mEq/L. The nurse should recognize the likelihood of what acid-base disorder?

Mixed acid-base disorder

A 20-year-old woman has visited her family physician due to occasional bouts of bloody diarrhea over the past several weeks, a phenomenon that she experienced 2 years prior as well. Her physician has diagnosed her with ulcerative colitis based on her history and visualization of the affected region by colonoscopy and sigmoidoscopy. Which of the following pathophysiological phenomena is most likely to underlie the client's health problem? Fissures and crevices developing in the mucosa that are seen as a characteristic "cobblestone" appearance Erosion of the endothelial lining of the distal small intestine by a combination of genetic, autoimmune, and environmental factors Compromise of the mucosal layer of the large intestinal surface by the effects of H. pylori Mucosal hemorrhages that have developed into crypt abscesses, which have in turn necrotized and ulcerated

Mucosal hemorrhages that have developed into crypt abscesses, which have in turn necrotized and ulcerated

A 26-year-old male patient with a diagnosis of schizophrenia has been admitted with suspected hyponatremia after consuming copious quantities of tap water. Given this diagnosis, what clinical manifestations and lab results should the nurse anticipate the patient will exhibit?

Muscle weakness, lethargy, and headaches

An 81-year-old female has a long-standing diagnosis of hypocalcemia secondary to kidney disease. She will be moving into an assisted living facility shortly. Which of the following clinical manifestations would the nursing staff at the facility likely observe in this patient?

Muscular spasms and complaints of tingling in the hands/feet

An elderly woman is brought to the emergency department by her family. They relate to the nurse that the client has had mental status changes and cannot remember her grandchildren's names. They go on to say that she is intolerant of cold and is lethargic. On physical examination, the nurse notes that the client has a husky voice, her face is puffy around the eyes, and her tongue appears to be enlarged. What diagnosis would the nurse suspect?

Myxedema

A nurse is caring for a patient who is diagnosed with acute poststreptococcal glomerulonephritis. Which of the following is the most common clinical presentation of this condition? a) Occult hematuria b) Nephrotic syndrome c) Nephritic syndrome d) Renal colic

Nephritic syndrome The acute nephritic syndrome is the clinical correlate of acute glomerular inflammation. The nephritic syndromes produce a proliferative inflammatory response, whereas the nephrotic syndrome produces increased permeability of the glomerulus. The acute nephritic syndrome may occur in such a systemic disease as systemic lupus erythematosus. Typically, however, it is associated with acute proliferative glomerulonephritis such as postinfectious glomerulonephritis. Acute postinfectious glomerulonephritis usually occurs after infection with certain strains of A beta-hemolytic streptococci and is caused by deposition of immune complexes of antibody and bacterial antigens

A patient presented to the emergency department of the hospital with a swollen, reddened, painful leg wound and has been diagnosed with methicillin-resistant Staphylococcus aureus (MRSA) cellulitis. The patient's physician has ordered a complete blood count and white cell differential. Which of the following blood components would the physician most likely anticipate to be elevated?

Neutrophils

A 34-year-old male client has diagnoses of liver failure, ascites, and hepatic encephalopathy secondary to alcohol abuse. The client's family is questioning the care team as to why his abdomen is so large even though he is undernourished and emaciated. Which of the following statements most accurately underlies the explanation that a member of the care team would provide the family?

Normally small transcellular fluid compartment, or third space, is becoming enlarged.

Which of the following inhibits growth hormone (GH)?

Obesity

Surgical stone removal is indicated for which of the following instances? a) Stone too small to pass b) Mild, intermittent pain for one day c) Obstruction of urine flow d) Small stone without evidence of size increase

Obstruction of urine flow Open stone surgery may be required to remove large calculi or those that are resistant to other forms of removal. Kidney stones are a major cause of upper urinary tract obstruction.

Recognizing the prevalence and incidence of dehydration among older adults, a care aide at a long-term care facility is in the habit of encouraging residents to drink even though they may not feel thirsty at the time. Which of the following facts underlies the care aide's advice?

Older adults often experience a decrease in the sensation of thirst, even when serum sodium levels are high.

A pediatric unit will be receiving an 8-day-old infant with a suspected congenital renal disorder. Which of the following renal abnormalities could be the possible cause? Select all that apply. One of the infant's kidneys may have failed to develop normally. The kidneys may be misshapen and have cysts present. The upper or lower poles of the two kidneys may be fused. Renal cell carcinoma may be present. Urine-filled dilation of renal pelvis associated with atrophy of the kidney may be present.

One of the infant's kidneys may have failed to develop normally. The kidneys may be misshapen and have cysts present. The upper or lower poles of the two kidneys may be fused.

A 35-year-old woman who has been in recovery from alcoholism for 2 years presents at her primary care physician's office with chronic hip pain. She reports that as part of her commitment to her recovery, she began exercising regularly about a year earlier. After a month or two, her hip began to hurt when she ran on the treadmill. She stretches, has had a physical trainer check her form to ensure that it is correct, and rests adequately between each workout. Six months ago, the pain began waking her at night, and now it is constant. She is not aware of any injury to her hip and has no other outward symptoms. Which of the following is most likely to be the cause of her pain? Hematogenous osteomyelitis Osteomyelitis with vascular insufficiency Tuberculosis of the bone Osteonecrosis

Osteonecrosis

A patient who has been on a high-protein diet comes to the emergency department with respiratory symptoms. Upon analysis of arterial blood gases (ABGs), the patient is diagnosed with hypercapnia. The nurse will note the ABG results that confirm this diagnosis include:

PCO2 of 51 mm Hg

A nurse caring for a client with asthma monitors respiratory function. Which data indicate the client has severe persistent asthma?

PEF 350 ml in AM, 200 ml at noon, symptoms occur continuously

A nurse caring for a client with asthma monitors respiratory function. Which data indicate the client has severe persistent asthma? a) PEF 450 ml in AM, 300 ml at noon, symptoms occur daily b) PEF 350 ml in AM, 200 ml at noon, symptoms occur continuously c) PEF 400 ml in AM, 390 ml at noon, symptoms occur twice a week d) PEF 500 ml in AM, 427 ml at noon, symptoms occur 2-3 times a week

PEF 350 ml in AM, 200 ml at noon, symptoms occur continuously Explanation: Asthma can be rated at four levels: mild intermittent, mild persistent, moderate persistent, and severe persistent. In mild intermittent asthma the symptoms occur 2 or fewer times per week and the client is asymptomatic between attacks with a normal peak expiratory flow (PEF). In mild persistent asthma the symptoms occur more than 2 times per week, but not daily. Exacerbations may affect activity. The PEF has 20-30% variability. Moderate persistent asthma has daily symptoms, exacerbations affect activity and the PEF variability is greater than 30 percent. In severe persistent asthma the symptoms are continual with frequent exacerbations and the PEF is less than 60 percent of what is predicted with variability more than 30 percent

A nurse caring for a client with asthma monitors respiratory function. Which data indicate the client has mild intermittent asthma?

PEF 400 ml in AM, 390 ml at noon, symptoms occur twice a week

A nurse caring for a client with asthma monitors respiratory function. Which data indicate the client has mild intermittent asthma? a) PEF 500 ml in AM, 427 ml at noon, symptoms occur 2-3 times a week b) PEF 450 ml in AM, 300 ml at noon, symptoms occur daily c) PEF 350 ml in AM, 200 ml at noon, symptoms occur continuously d) PEF 400 ml in AM, 390 ml at noon, symptoms occur twice a week

PEF 400 ml in AM, 390 ml at noon, symptoms occur twice a week Explanation: Asthma can be rated at four levels: mild intermediate, mild persistent, moderate persistent, and severe persistent. In mild intermittent asthma the symptoms occur 2 or fewer times per week and the client is asymptomatic between attacks with a normal peak expiratory flow (PEF). In mild persistent asthma the symptoms occur more than 2 times per week, but not daily. Exacerbations may affect activity. The PEF has 20-30% variability. Moderate persistent asthma has daily symptoms, exacerbations affect activity and the PEF variability is greater than 30%. In severe persistent asthma the symptoms are continual with frequent exacerbations and the PEF is less than 60% of what is predicted with variability more than 30%

A 15-year-old female has presented to her family physician complaining of frequent discomfort around the time of her period. She has subsequently been diagnosed with primary dysmenorrhea. Which of the following treatments is most likely to be effective? Investigation and resolution of her hypothalamic-pituitary-ovarian disorder Hormone therapy aimed at resolving her estrogen deprivation Pain control with prostaglandin synthetase inhibitors Dilation and curettage

Pain control with prostaglandin synthetase inhibitors

When the foreskin is too tight and constricted that it cannot cover the glans of the penis is called: Phimosis Paraphimosis Hypospadia Epispadia

Paraphimosis

A 52-year-old patient has just passed a kidney stone and has high levels of calcium in her urine. Blood test show high levels of calcium in her blood as well. What subsequent lab results would be most likely to distinguish between primary hyperthyroidism and hypercalcemia? Parathyroid hormone level Bone Scan Plasma phosphate levels serum magnesium levels

Parathyroid hormone level

A 52-year-old patient has just passed a kidney stone and has high levels of calcium in her urine. Blood tests show high levels of calcium in her blood as well. What subsequent lab results would be most likely to distinguish between primary hyperparathyroidism and hypercalcemia of malignancy?

Parathyroid hormone level

A 52-year-old patient has just passed a kidney stone and has high levels of calcium in her urine. Blood tests show high levels of calcium in her blood as well. What subsequent lab results would be most likely to distinguish between primary hyperparathyroidism and hypercalcemia of malignancy? Parathyroid hormone level Bone scan Plasma phosphate levels Serum magnesium level

Parathyroid hormone level

The newborn-nursery nurse is preparing to perform a required neonatal screening for congenital hypothyroidism. What should the nurse do to obtain the necessary sample?

Perform a heel stick to obtain a drop of blood for a T4 and TSH.

A 70-year-old woman who delivered four children during her reproductive years has weakened pelvic floor muscles. Which of her following anatomical structures is least susceptible to inappropriate herniation into her vagina? Peritoneum Uterus Bladder Rectum

Peritoneum

Asthma is considered a hypersensitive reaction that can be triggered by:

Pet Dander

Which of the following is most likely to precipitate an asthmatic attack in a child with a diagnosis of extrinsic, or atopic, asthma?

Pet dander

A patient who has been on a high-protein diet comes to the emergency department with respiratory symptoms. Upon analysis of arterial blood gases (ABGs), the patient is diagnosed with hypercapnia. The nurse will note the ABG results that confirm this diagnosis include: Select all that apply Ph 7.31 (normal 7.35 to 7.45) PO2 of 97% PCO2 of 58 mm Hg (normal 38 to 42) Serum HCO3 of 33mEq/L (normal 22 to 28) Serum K+ (potassium) of 3.6 mmol/L (normal 3.5 ot 5.0)

Ph 7.31 (normal 7.35 to 7.45) PCO2 of 58 mm Hg (normal 38 to 42) Serum HCO3 of 33 mEq/L (normal 22 to 28)

A patient diagnosed with Goodpasture's syndrome would require which of the therapies to remove proteins and autoantibodies from the system? a) Intervenous calcium b) Renal transplant c) Kidney removal d) Plasmapheresis

Plasmapheresis Plasmapheresis is used to filter the blood for removal of proteins and the circulating anti-GBM (glioblastoma multiforme) antibodies. Renal transplantation would return the kidneys to normal function, but are the extreme of treatment. The other options would not produce the necessary treatment outcomes

A diagnosis of exudative pleural effusion would be based on which of the following?

Pleural fluid protein/serum protein ratio >0.5

A pulmonary embolism occurs when there is an obstruction in the pulmonary artery blood flow. Classic signs and symptoms of a pulmonary embolism include dyspnea, chest pain, and increased respiratory rate. What is a classic sign of pulmonary infarction?

Pleuritic pain

A young adult is brought to the urgent care by a parent. The client states that he just does not feel good. Upon further assessment, the health care provider notes the following: • Unilateral chest pain when the client is asked to cough or move • Rapid respiratory rate • Splinting of the chest • Decreased tidal volume The provider would interpret these manifestations as:

Pleuritis

A short, nonsmoking 44-year-old male presents to the emergency room with left-sided chest pain and a cough. He states that the pain started abruptly and worsens with deep breathing and coughing. He denies recent injury. Assessment includes shallow respirations with a rate of 36, normal breath sounds, and no cyanosis. Which condition is most likely causing his symptoms? Myocardial infarction Spontaneous pneumothorax Pleuritis related to infection Obstructive atelectasis

Pleuritis related to infection

A short, nonsmoking middle-aged man presents to the emergency room with left-sided chest pain and a cough. He says that the pain started abruptly, just after lunch, and that breathing and coughing make it worse. He denies recent injury. He is breathing shallowly and rapidly and expresses fear that he may be having a "heart attack." Breath sounds are normal, and he is not cyanotic. Which condition is most likely causing his symptoms?

Pleuritis related to infection

A short, nonsmoking 44 year old male presents to the emergency room with left-sided chest pain and a cough. He states the pain started abruptly and worsens with deep breathing and coughing. He denies recent injury. Assessment includes shallow respirations with a rate of 36, normal breath sounds, no cyanosis. Which condition is most likely causing his symptoms? a) Obstructive atelectasis b) Spontaneous pneumothorax c) Myocardial infarction d) Pleuritis related to infection

Pleuritis related to infection Explanation: Pleuritis, which frequently accompanies infections that cause cough, is unilateral, starts abruptly, and is worsened by coughing or deep breathing. The client's shallow, rapid breathing may be due to anxiety but also is a way of maintaining adequate air intake while avoiding deep breathing, which exacerbates the pain of pleuritis. His cough may be an indication of infection, especially as he is not a smoker. The pain of myocardial infarction is not worsened by deep breathing or coughing. Spontaneous pneumothorax would be very unlikely in a short, nonsmoking middle-aged man. Tachypnea might indicate obstructive atelectasis, but normal breath sounds and lack of cyanosis argue against it

Which one of the following would the nurse see as being liable to cause the most serious long-term problems? a) Unilateral renal agenesis b) Polycystic kidney disease c) Simple renal cyst d) Horseshoe kidney

Polycystic kidney disease Agenesis refers to failure of an organ to develop at all. The other kidney usually undergoes compensatory hypertrophy and performs the function of the missing kidney. Most simple cysts do not produce signs or symptoms or compromise renal function. A horseshoe kidney occurs when the upper and lower poles of the two kidneys are fused, producing a horeshoe-shaped structure. The condition usually does not cause problems. Polycystic kidneys may be associated with aneurysm, and subarachnoid hemorrhage is a frequent cause of death.

A 51-year-old woman has been diagnosed with Cushing syndrome after a diagnostic workup that reveals cortisol hypersecretion. The nurse knows which of the following assessment findings would be inconsistent with her diagnosis? Increased blood pressure and decreased potassium levels A protruding abdomen and a "buffalo hump" on the back Poor stress management and hyperpigmentation A "moon face" and muscle weakness

Poor stress management and hyperpigmentation

A 40-year-old male presents to the emergency department reporting chest pain and shortness of breath. The health care provider suspects a pulmonary embolism and orders several diagnostic tests. Select the test that would require further follow-up.

Positive D-dimer

A 56-year-old female with a diagnosis of breast cancer has developed bone metastases, which her oncologist would like to treat with radiation therapy. What is her physician's most likely primary goal in the treatment plan of the metastases? Thorough elimination of neoplastic cells in the bone structure Preservation of normal weight-bearing and range of motion Ensuring vascular supply to the bone is not affected Prevention of pathologic fractures

Prevention of pathologic fractures

A patient recently had surgery for a hip fracture. Which of the following nursing interventions would be most effective for preventing pulmonary emboli in this patient?

Prevention of the development of a deep vein thrombosis

One of the first signs that indicates an infant may have congenital hypothyroidism is:

Prolonged period of physiologic jaundice

The nurse is caring for a client with hyperthyroidism and is preparing to administer the morning medications. Which medication will the nurse administer in order to block the conversion of T4 to T3 in the tissues?

Propylthiouracil (PTU)

Which of the following substances would not be found in glomerular filtrate? a) Water b) Protein c) Potassium d) Sodium

Protein The glomerular filtrate has a chemical composition similar to plasma which contains sodium, potassium and water, but it contains no proteins because large molecules do not readily cross the glomerular wall.

The nurse is reviewing the lab results of a patient with suspected nephrotic syndrome. The nurse anticipates that the results to include: a) Abnormal blood clotting factors b) Serum hyperalbuminemia c) Protein in the urine d) Decreased tryglycerides e) Decreased low-density lipoproteins

Protein in the urine In a person with nephrotic syndrome there is massive proteinuria (protein in the urine), serum hypoalbuminemia, generalized edema and hyperlipidemia.

A nurse who works on a urology-gynecology ward of a hospital is coming on shift and will be caring for a 34-year-old woman who has been admitted overnight for the treatment of a large endometriosis. What interventions should the nurse most realistically anticipate providing over the course of the shift and the next several days? Providing pain control; preparing the client for a laparoscopic procedure or hysterectomy. Administration of packed red blood cells to compensate for low hemoglobin; administering hormone therapy. Assisting with a Pap smear; administration of high-dose corticosteroids. Administration of male androgens to minimize endometrial hyperplasia; facilitating a dilation and curettage.

Providing pain control; preparing the client for a laparoscopic procedure or hysterectomy.

Cystic fibrosis (CF) is an autosomal recessive disorder involving the secretion of fluids in specific exocrine glands. The genetic defect in CF inclines a person to chronic respiratory infections from a small group of organisms. Which organisms create chronic infection in a child with cystic fibrosis?

Pseudomonas aeruginosa and Staphylococcus aureus

Cystic fibrosis (CF) is an autosomal recessive disorder involving the secretion of fluids in specific exocrine glands. The genetic defect in CF inclines a person to chronic respiratory infections from a small group of organisms. Which organisms create chronic infection in a child with cystic fibrosis? a) Pseudomonas aeruginosa and Escherichia coli b) Staphylococcus aureus and hepatitis C c) Pseudomonas aeruginosa and Staphylococcus aureus d) Haemophilus influenzae and influenza A

Pseudomonas aeruginosa and Staphylococcus aureus Explanation: In addition to airway obstruction, the basic genetic defect that occurs with CF predisposes to chronic infection with a surprising small number of organisms, the most common being Pseudomonas aeruginosa, Burkholderia cepacia, Staphylococcus aureus, and Haemophilus influenzae. The other disease causing organisms are not linked to CF.

Pain is an expected assessment finding in patients who have which of the following lung diseases?

Pulmonary Embolism

Three days following surgical repair of a hip fracture a client becomes anxious and complains of sudden shortness of breath. What disorder is the client most likely experiencing?

Pulmonary embolism

Pulmonary hypertension is usually caused by long-term exposure to hypoxemia. When pulmonary vessels are exposed to hypoxemia, what is their response?

Pulmonary vessels constrict

The nurse needs to assess the oxygen status of a client who is suddenly experiencing shortness of breath. The most appropriate noninvasive measurement techniques would be:

Pulse Oximetry

A patient arrives in the ED after an automobile accident. Which of the following clinical manifestations lead the nurse to suspect a pneumothorax? Select all that apply. Respiratory rate 34 Asymmetrical chest movements, especially on inspiration Diminished breath sounds over the painful chest area Pulse oximetry 98% ABG pH level of 7.38

Respiratory rate 34 Asymmetrical chest movements, especially on inspiration Diminished breath sounds over the painful chest area

The nurse needs to assess the oxygen status of a client who is suddenly experiencing shortness of breath. The most appropriate noninvasive measurement techniques would be: a) Pulmonary function tests b) Arterial blood gas analysis c) Assessment of the rate and depth of respiration d) Pulse oximetry

Pulse oximetry Explanation: Noninvasive measurements of arterial O2 saturation of hemoglobin can be obtained using an instrument called the pulse oximeter. Sensors that can be placed on the finger, toe, ear, or forehead are available. Although pulse oximetry is not as accurate as arterial blood gas measurements, it provides the means for noninvasive and continuous monitoring of O2 saturation, which is a useful indicator of respiratory and circulatory status.

A nurse in the emergency department is preparing a presentation about acute respiratory distress syndrome (ARDS) for new nurses. What are common causes of ARDS that should be included? Select all that apply.

Radiation Smoke inhalation Fat embolism

Premature infants who are treated with mechanical ventilation, mostly for respiratory distress syndrome, are at risk for developing bronchopulmonary dysplasia (BPD), a chronic lung disease. What are the signs and symptoms of BPD? a) Weight loss and a barrel chest b) A barrel chest and rapid weight gain c) Tachycardia and slow shallow breathing d) Rapid and shallow breathing and chest retractions

Rapid and shallow breathing and chest retractions Explanation: The infant with BPD often demonstrates a barrel chest, tachycardia, rapid and shallow breathing, chest retractions, cough, and poor weight gain. Other signs and symptoms listed are not those of BPD.

A nurse is providing care for a client who has been admitted to a medical unit with a diagnosis of bronchiectasis. Which of the following signs and symptoms should the nurse expect to find during physical assessment of the client and the review of the client's history? Select all that apply. Recurrent chest infections Production of purulent sputum A barrel chest Low hemoglobin levels Recent surgery

Recurrent chest infections Production of purulent sputum Low hemoglobin levels

The nurse reviews the lab results for a patient who has advanced autosomal dominant polycystic kidney disease (ADPKD). The patient 's hemoglobin is 8.8 g/dL. The nurse suspects this lab value is related to which of the following causes? a) Poor dietary intake of iron b) Low calcium levels c) Reduced production of erythropoietin d) Hemorrhage

Reduced production of erythropoietin As ADPKD progresses, the nephrons reduce the production of erythropoietin (EPO). EPO is necessary for red blood cell production by bone marrow, so EPO deficiency causes anemia.

Which of the following phenomena contributes to the difficulties with absorption, distribution, and elimination of drugs that are associated with kidney disease?

Reductions in plasma proteins increase the amount of free drug and decrease the amount of protein-bound drug.

Which of the following is the most common cancer of the kidney? a) Lymphoma b) Renal cell carcinoma c) Wilms tumor d) Transitional cell carcinoma

Renal cell carcinoma Renal cell carcinoma accounts for 80% to 90% of all kidney tumors.

Which of the following diagnoses puts a patient at risk for developing an immunologic form of interstitial lung disease?

Rheumatoid arthritis

Following a motorcycle accident that resulted in bilateral femoral fractures, a 42-year-old male has been receiving skeletal traction for the past 8 days. His care providers would recognize that which of the following risks is paramount? Risk of thromboemboli Risk of compartment syndrome Risk of permanent muscle atrophy Risk of decreased bone density and increased future fracture risk

Risk of thromboemboli

Following destruction of the pituitary gland, ACTH stimulation stops. Without ACTH to stimulate the adrenal glands, the adrenals' production of cortisol drops. This is an example of which type of endocrine disorder?

Secondary

A child is brought to the emergency department struggling to breathe with a prolonged bronchospasm and severe hypoxemia. Assessment revealed the use of accessory muscles, a weak cough, audible wheezing sound, moist skin, and tachycardia. Which of the following is the most likely diagnosis? a) Severe asthma attack b) Pulmonary embolism c) Chronic obstructive pulmonary disease d) Cystic fibrosis

Severe asthma attack Severe asthma attacks are accompanied by use of the accessory muscles, distant breath sounds due to air trapping, and loud wheezing. As the condition progresses, fatigue develops, the skin becomes moist, and anxiety and apprehension are obvious. Sensations of shortness of breath may be severe, and often the person is able to speak only one or two words before taking a breath. At the point at which airflow is markedly decreased, breath sounds become inaudible with diminished wheezing, and the cough becomes ineffective despite being repetitive and hacking. This point often marks the onset of respiratory failure.

A 22 year-old female with a history of intermittent flank pain, repeated UTIs and hematuria has been diagnosed with autosomal dominant polycystic kidney disease (ADPKD). Which of the following phenomena has most likely contributed to the development of this diagnosis? a) She has inherited undersized kidneys that are prone to calculi formation. b) UTIs coupled with an impaired immune response have caused her ADPKD. c) She has inherited a tendency for epithelial cell in her tubules to proliferate inappropriately. d) Severe hypertension and portal hypertension are likely precursors.

She has inherited a tendency for epithelial cell in her tubules to proliferate inappropriately. ADPKD is an inherited condition, and the etiology is thought to involve cysts arising in segments of the renal tubules from a few epithelial cells that proliferate abnormally. UTIs are consequent, not causative, of the condition. Severe hypertension and portal hypertension are more commonly associated with ARPKD (autosomal recessive polycystic kidney disease) than ADPKD. Kidneys are typically oversized in ADPKD and renal calculi are not noted sequelae

A 22-year-old female with a history of intermittent flank pain, repeated UTIs, and hematuria has been diagnosed with autosomal dominant polycystic kidney disease (ADPKD). Which of the following phenomena has most likely contributed to the development of this diagnosis?

She has inherited a tendency for epithelial cells in her tubules to proliferate inappropriately.

A 77-year-old female hospital patient has contracted Clostridium difficle during her stay and is experiencing severe diarrhea. Which of the following statements best conveys a risk that this woman faces? 1.She is susceptible to isotonic fluid volume deficit 2.She is prone to isotonic fluid volume loss 3.She could develop third-spacing edema as a result of plasma protein losses 4.She is at risk of compensatory fluid volume overload secondary to gastrointestinal water and electrolyte losses.

She is susceptible to isotonic fluid volume deficit

A 77-year-old female hospital patient has contracted Clostridium difficile during her stay and is experiencing severe diarrhea. Which of the following statements best conveys a risk that this woman faces?

She is susceptible to isotonic fluid volume deficit.

Precocious puberty is a disorder that occurs in both boys and girls. What does precocious puberty cause in adults?

Short stature in adults

A child is brought to the emergency department with an asthma attack. Assessment revealed the use of accessory muscles, a weak cough, audible wheezing sound, moist skin, and tachycardia. Which of the following drugs will the nurse anticipate administering first?

Short-acting beta 2-adrenergic agonists such as albuterol (SABA)

A child is brought to the emergency department with an asthma attack. Assessment revealed the use of accessory muscles, a weak cough, audible wheezing sound, moist skin, and tachycardia. Which of the following drugs will the nurse anticipate administering first? a) Anti-inflammatory agents such as sodium cromolyn b) Short-acting beta 2-adrenergic agonists such as albuterol (SABA) c) Oral corticosteroids d) Anticholinergic medications such as ipratropium

Short-acting beta 2-adrenergic agonists such as albuterol (SABA) Explanation: The quick-relief medications such as SABA (e.g., albuterol, levalbuterol, pirbuterol) relax bronchial smooth muscle and provide prompt relief of symptoms, usually within 30 minutes. They are administered by inhalation (i.e., metered-dose inhaler or nebulizer), and their recommended use is in alleviating acute attacks of asthma.

Which description of symptoms would the nurse expect to obtain from a client with exacerbation of sarcoidosis?

Shortness of breath, non-productive cough, and chest pain

A parent arrives in the endocrinology clinic with her 8-year-old son, concerned about his rapid development and tall stature. What significant assessment finding does the nurse recognize is important to report to the physician related to the development of precocious puberty?

Significant genital enlargement

A client with severe hypothyroidism is presently experiencing hypothermia. What nursing intervention is a priority in the care of this client?

Slow rewarming of the client to prevent vasodilation and vascular collapse

A 33-year-old female client who presents to the doctor's office with pronounced enlargement of her hands and facial features is diagnosed with acromegaly. The acromegaly is most likely the result of:

Somatotrope adenoma

A client who has had a diagnosis of lung cancer is scheduled to begin radiation treatment. The nurse knows that which of the following statements listed below about potential risks of radiation is most accurate?

Some clients experience longer-term irritation of skin adjacent to the treatment site.

The nurse is providing care for a 21-year-old female client with gas gangrene secondary to her compound fracture in her arm. Which of the following assessment findings would the nurse most reasonably expect to find when caring for a client with a diagnosis of gas gangrene?

Spreading edema

An immunocompromised host is open to pneumonia from all types of organisms. There is, however, a correlation between specific types of immunologic deficits and specific invading organisms. What organism is most likely to cause pneumonia in an immunocompromised host with neutropenia and impaired granulocyte function? a) Β-Hemolytic Streptococcus b) Staphylococcus aureus c) Eosinophilic bacillus subtilis d) Gram-positive bacilli e) Haemophilus influenza

Staphylococcus aureus Explanation: Neutropenia and impaired granulocyte function, as occurs in persons with leukemia, chemotherapy, and bone marrow depression, predispose to infections caused by S. aureus, Aspergillus, Gram-negative bacilli, and Candida. All the other organisms can cause pneumonia, but they are not usually seen in people with neutropenia and impaired granulocyte function.

An adolescent boy asks the nurse what increases the secretion of growth hormone (GH). The best response would be:

Starvation

Drug-related nephropathies occur all too often. They involve functional and/or structural changes to the kidney after exposure to a drug. What does the tolerance to drugs depend on? a) Glomerular filtration rate b) State of hydration c) Proteinuria d) Vesicoureteral reflux

State of hydration The tolerance to drugs varies with age and depends on renal function, state of hydration, blood pressure, and the pH of the urine. None of the other answers are correct.

A nurse is caring for a patient who has been diagnosed with kidney colic and has not yet passed the stone. Which of the following interventions should the nurse be sure to include when planning the care for this patient? a) Have the client take cool baths. b) Administer acetominophen (Tylenol) every 4 hours for pain. c) Restrict fluid intake. d) Strain the urine.

Strain the urine. All urine should be strained during an attack in the hope of retrieving the stone for chemical analysis and determination of type. This information, along with a careful history and laboratory tests, provides the basis for long-term preventative measures. The patient will require an increase in fluid intake. The patient will also require an opioid analgesic to control the pain

A client reports chest pain to the nurse. Which characteristics of the pain indicate pleuritis?

Sudden sharp pain in one side made worse by deep breathing

The nurse develops a plan to prevent atelectasis in a postsurgical client. Which intervention will be effective?

Supervision of hourly deep breathing exercises

The nurse develops a plan to prevent atelectasis in a postsurgical client. Which intervention will be effective? a) Supervision of hourly deep breathing exercises b) Encouraging calf pumping exercises c) Administering IV antibiotics d) Teaching the client to wear an abdominal binder

Supervision of hourly deep breathing exercises Explanation: Ambulation, deep breathing exercises and changing body positions help to prevent postoperative atelectasis. An abdominal binder may restrict full diaphragmatic movement. Calf pumping exercises and elastic stockings are designed to prevent blood clots, but do not assist in lung expansion. IV antibiotics prevent and treat infection, but do not aid lung expansion.

An 8-year-old child has just been diagnosed with systemic lupus erythematosus (SLE). The parents wonder what the child's prognosis is going to be. Which of the following findings would be considered a good prognostic indicator of the extent/seriousness of the disease? Complaints of arthralgias and arthritis in joints with movement Ligaments and tendons hurt during passive ROM Has a rash on the nose and cheeks Swelling in the face and eyes and rust/blood-colored urine

Swelling in the face and eyes and rust/blood-colored urine

Which of the following medications helps treat the inflammatory reaction of asthma clients diagnosed with late-phase asthma response? a) Phosphodiesterase inhibitors b) Anticholinergic agents c) Systemic corticosteroids d) Long-acting β2-agonists

Systemic corticosteroids Explanation: A short course of systemic corticosteroids, administered orally or parenterally, may be used for treating the inflammatory reaction associated with the late-phase response. The anticholinergic agents block cholinergic receptors and reduce intrinsic vagal tone that causes bronchoconstriction. The long-acting β2 agonists, available for administration by the inhaled or oral routes, act by relaxing bronchial smooth muscle. Theophylline, a phosphodiesterase inhibitor, is a bronchodilator that acts by relaxing bronchial smooth muscle.

A client arrives in the clinic and states to the nurse, "I am tired all the time and have gained weight. My hair is so dry it is breaking." The nurse assesses that the client's face is puffy with edematous eyelids and the outer third of the eyebrows are thinning. What lab test will the nurse prepare the client for that is characteristic of this disorder?

T4 and TSH

Which of the following characteristics could apply to healthy somatic cells rather than cancerous cells?

The cells are unable to proliferate except by mitotic division.

A parent brings his child to the clinic, concerned about her short stature and asking the nurse if there is a problem with her height. What indication is the nurse aware of when performing the assessment that determines whether this child has short stature?

The child's height is well below the third percentile on several clinic visits.

A nurse is providing care for a patient who has been admitted with a newly diagnosed bilateral pleural effusion. Which of the following findings from the nurse's initial assessment of the patient is incongruent with the patient's diagnosis and would require further investigation?

The client complains of sharp pain exacerbated by deep inspiration.

A 23-year-old woman goes to the drugstore to buy a medication to ease the symptoms of her cold. Her friends have told her to buy a medication with an antihistamine in it to help dry up her runny nose and make it easier to breathe. The woman talks with the pharmacist, who has known her many years. The pharmacist recommends that this young woman not buy a cold medication with a decongestant in it. Why would he do that? a) The client has a history of type I diabetes mellitus. b) The client has a history of juvenile rheumatoid arthritis. c) The client has a history of hypotension. d) The client has a history of hyperthyroidism

The client has a history of hyperthyroidism. Explanation: Decongestant drugs (i.e., sympathomimetic agents) are available in over-the-counter nasal sprays, drops, and oral cold medications. These drugs constrict the blood vessels in the swollen nasal mucosa and reduce nasal swelling. Rebound nasal swelling can occur with indiscriminate use of nasal drops and sprays. Oral preparations containing decongestants may cause systemic vasoconstriction and elevation of blood pressure when given in doses large enough to relieve nasal congestion. They should be avoided by persons with hypertension, heart disease, hyperthyroidism, diabetes mellitus, or other health problems

A 68-year-old male client with aortic stenosis secondary to calcification of the aortic valve is receiving care. Which of the following statements best captures an aspect of this client's condition?

The client has possibly undergone damage as a result of calcification following cellular injury.

A 56-year-old female hospital patient with a history of alcohol abuse is receiving intravenous (IV) phosphate replacement. Which of the following health problems will this IV therapy most likely resolve?

The client is acidotic and has impaired platelet function.

The nurse is educating a newly diagnosed client with Hashimoto thyroiditis who is to be discharged from the acute care facility. What should the nurse be sure to include in the education to prevent complications?

The client should be informed about the signs and symptoms of severe hypothyroidism and the need for early intervention.

The nurse is providing discharge instructions for a client with Graves' disease who has ophthalmopathy. What should the nurse be sure to include in the instructions to decrease exacerbation of this clinical manifestation?

The client should be strongly urged not to smoke.

The nurse is providing education to a client with Addison's disease who has been treated for hyponatremia and hypoglycemia related to the disease. What should the nurse inform the client should be done to ensure control of these conditions?

The client should eat and exercise on a regular schedule.

The nurse is caring for the following clients. Select the client at highest risk for the development of atelectasis.

The client who is postop total knee replacement and receiving client-controlled analgesia

The nurse is caring for the following clients. Select the client at highest risk for the development of atelectasis. a) The client who is postop total knee replacement and receiving client-controlled analgesia b) The client with a lower leg cast who changes position every 2 hours c) The client who is mobile within 24 hours after abdominal surgery d) The client diagnosed with pneumonia who performs frequent coughing and deep breathing exercises

The client who is postop total knee replacement and receiving client-controlled analgesia Explanation: The danger of obstructive atelectasis increases after surgery as anesthesia, pain, administration of narcotics, and immobility tend to promote retention of viscid bronchial secretions and hence airway obstruction. The encouragement of coughing and deep breathing, frequent change of position, adequate hydration, and early ambulation decrease the likelihood of atelectasis developing. Coughing and deep breathing exercises are the most beneficial option to prevent atelectasis. Repositioning needs to occur more frequently (about every 2 hours), ambulating prior to 24 hours postoperatively, and increasing oral fluids will assist with the prevention of atelectasis.

A 32-year-old client has had a positive reaction to a tuberculin skin test and the duration of exposure is unknown. What medication education will the nurse provide to the client? a) The client will need education about the administration of INH and the importance of taking it as prescribed. b) The client will be required to take antibiotics for a minimum of 10 days. c) The client will be instructed to take ethambutol and pyrazinamide (PZA) and be informed of the side effects. d) The client should be instructed that the urine will turn orange, as well as the tears.

The client will need education about the administration of INH and the importance of taking it as prescribed. Explanation: People who are 35 years of age or younger with a positive reaction of unknown duration are considered to harbor a small number of microorganisms and usually are treated with isoniazid (INH). It is important to inform the client that they should take the medication as directed and not skip doses or stop the medication without discussing it with the physician. The client will not be required to take a short course (10 days) of antibiotics. The urine and tears turn orange when taking rifampin. This does not occur when taking INH

Which of the following best describes a secondary disorder of endocrine function?

The disorder occurs when the target organ is normal, but stimulating hormones alter its function.

An infant born prematurely is in the neonatal intensive care unit (NICU) for observation and to ensure hemodynamic stability when the nurse observes the infant beginning to have nasal flaring. What does the nurse understand that this may indicate? a) The infant is using nasal flaring to take in more air. b) The infant is having atelectasis. c) The infant is attempting to decrease oxygen consumption. d) The infant is attempting to rid the lungs of fluid.

The infant is using nasal flaring to take in more air. Explanation: Nasal flaring is a method that infants use to take in more air. This method of breathing increases the size of the nares and decreases the resistance of the small airways.

A newborn is screened for congenital hypothyroidism and is found to have the disorder. When educating the mother about the importance of the infant's taking thyroid hormone supplement, what should be included in the education?

The infant will have dosage levels adjusted as he grows.

A 57-year-old woman has been diagnosed with atrophic vaginitis and has expressed surprise to her care provider, citing a lifetime largely free of gynecological health problems. She has asked what may have contributed to her problem. How can the care provider best respond? The lower levels of estrogen since you've begun menopause make your vagina prone to infection." "Vaginitis is not usually the direct result of any single problem, but rather an inevitability of the vaginal dryness that accompanies menopause." "This type of vaginitis is most commonly a symptom of a latent sexually transmitted infection that you may have contracted in the distant past." "The exact cause of this problem isn't known, but it can usually be resolved with a diet high in probiotic bacteria."

The lower levels of estrogen since you've begun menopause make your vagina prone to infection."

Following a long history of fatigue, weakness and poor appetite, a 39-year-old male has been diagnosed with hypopituitarism. Which of the following clinical findings would most likely cause his care team to suspect that the man has an additional endocrine disorder from a different source?

The man has a chronic platelet deficiency and is occasionally anemic.

Following a long history of fatigue, weakness, and poor appetite, a 39-year-old male has been diagnosed with hypopituitarism. Which of the following clinical findings would most likely cause his care team to suspect that the man has an additional endocrine disorder from a different source? The man has a low sperm count and has been unable to have children. The man has a chronic platelet deficiency and is occasionally anemic. The client is 5.2. tall and was consistently short for his age as a child. The man displays the signs and symptoms of hypothyroidism.

The man has a chronic platelet deficiency and is occasionally anemic.

The nurse is educating a parent of a child with short stature caused by growth hormone (GH) deficiency about the administration of GH. What should the nurse include when educating the parents about administration?

The medication will be given daily during the period of active growth and can be continued into adulthood.

Following a biopsy, a 54-year-old man has been diagnosed as having a benign neoplastic tumor. Which of the following characteristics most likely applies to his tumor?

The well-differentiated, neoplastic cells are clustered together in a single mass.

A nurse is providing care for an older, previously healthy adult male who has been diagnosed today with pneumococcal pneumonia. Which of the following signs and symptoms is the nurse most likely to encounter? The man will be hypotensive and febrile and may manifest cognitive changes. The patient will have a cough producing clear sputum, and he will have faint breath sounds and fine crackles. The patient will have copious bloody sputum and diffuse chest pain and may lose his cough reflex. The patient will lack lung consolidation and will have little, if any, sputum production.

The patient will have a cough producing clear sputum, and he will have faint breath sounds and fine crackles.

A nurse observes that a patient's urine is cola colored and considers which of the following as a possible reason? a) The patient has ingested a dark-colored drink. b) The patient's urine contains material from the degradation of red blood cells. c) The patient has an elevation of urine potassium. d) The patient's urine has a decrease in the specific gravity.

The patient's urine contains material from the degradation of red blood cells. When red blood cells degrade in the urine, urine may appear cola colored.

Which of the following statements best captures an aspect of the function of the hypothalamic-pituitary-adrenal (HPA) system?

The pituitary gland communicates with the adrenal cortex through the release of ACTH.

Which of the following statements best captures an aspect of the function of the hypothalamic-pituitary-adrenal (HPA) system? Adrenocorticotropic hormone (ACTH) released by the hypothalamus controls to release of cortisol. The pituitary gland communicates with the adrenal cortex through the release of ACTH. The adrenal cortex receives corticotrophin-releasing hormone (CRH) and in turn releases cortisol. The pituitary gland causes a release of CRH from the hypothalamus, which promotes hormone release from the adrenal cortex

The pituitary gland communicates with the adrenal cortex through the release of ACTH.

A 76-year-old male client is admitted to the emergency department after a fall. His assessment reveals a bruised left hip, shortness of breath, and shallow respirations with a respiratory rate of 30. What additional physical assessment finding would lead to a possible diagnosis of tension pneumothorax?

The presence of a deviated trachea

The nurse is teaching a group of nursing students about exudative pleural effusion. The nurse determines that teaching was effective when the students' state:

The specific gravity of the pleural exudate is greater than 1.020.

The parents of a child with cystic fibrosis ask the nurse to explain the sweat test performed on their newborn. How should the nurse respond?

The sweat test measures the concentration of salt in the infant's sweat

The parents of a child with cystic fibrosis ask the nurse to explain the sweat test performed on their newborn. How should the nurse respond? a) Abnormal pancreatic enzymes may cause the infant to sweat more. b) Cystic fibrosis decreases the amount of salt in the infant's sweat. c) The sweat test evaluates the infant's ability to sweat in response to heat. d) The sweat test measures the concentration of salt in the infant's sweat.

The sweat test measures the concentration of salt in the infant's sweat. Explanation: Cystic fibrosis is a genetic disorder that produces abnormal functioning in the epithelial cell chloride channels. The chloride is not reabsorbed, leaving high concentrations of sodium chloride in the sweat. The sweat test is the standard approach to diagnosis of CF. A gel is applied to the skin and a patch is applied that has electrodes. Once enough sweat is collected, it is tested in the lab for sodium chloride concentration.

The nurse is caring for an older adult client who is being treated for primary hypothyroidism. The medication therapy includes a "go low and go slow" regimen. What is the importance of this medication regimen for this client?

There is a risk of inducing acute coronary syndromes in the older adult client if a more rapid correction of thyroid levels is used.

The nurse is performing a health history from a client with acromegaly. The client informs the nurse that he is waking up several times a night and has been told he has sleep apnea. What does the nurse inform the client is the rationale for this syndrome?

There is an increase in pharyngeal soft tissue accumulation.

A client with a newborn infant is also the caregiver for her 75-year-old mother, who lives with them and who has diabetes. The client requests pneumonia vaccinations for her entire household. Which vaccine is most likely to be effective for the baby? Since the baby's immune system is mature at birth, regular vaccine is appropriate. There is no effective vaccine for newborn infants. The 23-valent vaccine will be effective. No vaccine is necessary for the baby if the nursing mother is immunized.

There is no effective vaccine for newborn infants.

A young woman presents with signs and symptoms of urinary tract infection (UTI). The nurse notes that this is the fifth UTI in as many months. What would this information lead the nurse to believe? a) The woman takes too many bubble baths. b) The woman has multiple sexual partners. c) There is possible obstruction in the urinary tract. d) The woman does not clean herself properly.

There is possible obstruction in the urinary tract. Urinary tract obstruction encourages the growth of microorganisms and should be suspected in persons with recurrent UTIs. The other answers can cause lower UTIs, but an obstruction would be considered because of the frequency of the infections.

The hallmark manifestations of Cushing syndrome are a moon face, a "buffalo hump" between the shoulder blades, and a protruding abdomen. What other manifestations of Cushing syndrome occur?

Thin extremities and muscle weakness

A client who has just undergone a thyroidectomy is experiencing high fever, tachycardia, and extreme restlessness. The nurse would interpret these manifestations as:

Thyroid crisis

A nurse is collecting a urine specimen prior to measuring the albumin level in a client's urine. A colleague questions the rationale for the test, stating, "I thought albumin was related to liver function, not kidney function." How can the nurse best respond to this statement?

Urine albumin levels are useful for diagnosing diabetic kidney disease."

Because the associated nephropathy is an important cause of end-stage renal failure in adolescents, a toddler who has had an uncomplicated bout of urinary tract infections (UTI) should still be evaluated for 1.Urethrovesical reflux 2.Vesicoureteral reflux 3.Neurogenic bladder 4.Detrusor muscle instability

Vesicoureteral reflux

A 60-year-old man has been diagnosed with renal calculi after repeated episodes of excruciating flank pain in recent weeks. The man states that, "I don't know how this could happen to me, since I'm so careful about eating a healthy diet." What is the most appropriate response to the man's statement?

What you eat can influence your risk of stone formation, but many other factors like hormones and your metabolism are involved."

Which of the following manifestations typically accompanies an asthmatic attack?

Wheezing particularily with exhaling

An endocrinologist is providing care for a 30-year-old male who has lived with the effects of increased levels of growth hormone (GH). Which of the following teaching points about the client's future health risks is most accurate? "It's not unusual for high GH levels to cause damage to your hypothalamus." "GH excess inhibits your pancreas from producing enough insulin." "The high levels of GH that circulate in your body can result in damage to your liver." "When your pituitary gland is enlarged, there's a real risk that you'll develop some sight deficiencies."

When your pituitary gland is enlarged, there's a real risk that you'll develop some sight deficiencies."

Prior to undergoing diagnostic testing with contrast, it is recommended that older adult clients have their creatinine level checked. The rationale for this is to ensure the client: a) Is not allergic to shell fish or iodine b) Does not have a kidney stone obstructing the urethra c) Is in good enough health to withstand a walking on a treadmill d) Will not undergo an acute kidney injury by decreasing renal blood flow

Will not undergo an acute kidney injury by decreasing renal blood flow Some drugs, such as diuretics, high molecular weight radiocontrast media, the immunosuppressive drugs cyclosporine and tacrolimus, and the nonsteroidal anti-inflammatory drugs (NSAIDs), can cause acute kidney injury by decreasing renal blood flow. Checking creatinine levels do not predict the client's allergies, a kidney stone, or tolerance for stress testing.

A patient is admitted with pneumoconiosis. His history most likely reveals which of the following?

Work in a coal mine for 20 years

A care aide who works in a long-term care facility recognizes the high incidence and prevalence of a stage I pressure ulcer in immobile older adults. Which of the following protocols in the facility would the care aide advocate changing?

Wound dressings are applied promptly to all identified or potential pressure ulcers.

The parents of a child diagnosed with cystic fibrosis (CF) ask about the risk of any future children having the condition. How should the nurse respond?

You have a 25% chance that your next child will have CF.

The parents of a child diagnosed with cystic fibrosis (CF) ask about the risk of any future children having the condition. How should the nurse respond? a) Since the male carries the CF gene, you might consider a sperm donor. b) CF is autosomal dominant, so you have a 50% risk in another child. c) You have a 25% chance that your next child will have CF. d) Now that you have one child with CF, the rest will also have it.

You have a 25% chance that your next child will have CF. Explanation: Cystic fibrosis is autosomal recessive, meaning that two defective genes are needed for a child to be born with the disorder. Both parents must either be carriers (having one defective gene, but no symptoms) or have the disease (two defective genes). If both parents are carriers, each child has 1 in 4 chance of receiving two normal genes, a 50 percent chance of inheriting at least one gene, and a 1 in 4 chance of receiving two abnormal genes and having CF.

A patient who is recovering from burn injuries is discussing his prognosis with a physician. Which of the following teaching points about expectations for healing should the physician include?

You may find that the scar is a bit smaller than the area of the wound."

A 34-year-old man has been taking up to 2400 mg of ibuprofen per day following a motor vehicle accident several months ago and consequent chronic pain. He has recently been diagnosed with chronic analgesic nephritis as a result of his high analgesic intake. The man is surprised at the diagnosis stating, "I thought that taking too many drugs hurts your liver if anything, not your kidneys." What is the most appropriate response to the man's statement?

Your kidneys are vulnerable to damage because of how much blood flows through them and the fact that they break down many drugs."

While teaching a class on female cancers, the instructor emphasizes to the nursing students that many patients with ovarian cancer may display abdominal pain, bloating, feeling full quickly after ingesting food. pain after intercourse, bleeding irregularities, perineal tenderness. colicky low abdominal pain, adnexal mass present without palpation. lower abdominal pain localized to one side, referred shoulder pain.

abdominal pain, bloating, feeling full quickly after ingesting food

A child has been brought to an urgent care clinic. The parents state that the child is "not making water." When taking a history, the nurse learns the child had a sore throat about 1 week ago but seems to have gotten over it. "We [parents] only had to give antibiotics for 3 days for the throat to be better." The nurse should suspect the child has developed: a) acute renal failure. b) acute postinfectious glomerulonephritis. c) nephrotic syndrome. d) kidney stones.

acute postinfectious glomerulonephritis. The classic case of poststreptococcal glomerulonephritis follows a streptococcal infection by approximately 7 to 12 days: the time needed for the development of antibodies. The primary infection usually involves the pharynx (pharyngitis), but can also result from a skin infection (impetigo). Oliguria, which develops as the GFR (Glomerular filtration rate ) decreases, is one of the first symptoms.

Upon auscultation of your patient's lung sounds, you notice a high-pitched wheezing sound. This is due to:

airway constriction

Emphysema is characterized by hyperinflation of the lungs resulting from:

alveolar wall destruction from high proteylitic enzymes

Which of the following clients is at risk for developing acute respiratory failure? a) A teenager in a high school that has had an increase in student absences due to an outbreak of strep throat b) A toddler in daycare who has been sharing toys with peers before the staff could sanitize properly c) A middle-aged male diagnosed with amyotrophic lateral sclerosis (ALS) who has pneumonia with low O2 saturation d) An elderly female living in senior housing who has been exposed to a "cold" while her grandchildren visited

c) A middle-aged male diagnosed with amyotrophic lateral sclerosis (ALS) who has pneumonia with low O2 saturation

A patient with ESRD comes into the emergency department in severe acidosis. The nurse notes that the respiratory rate is 36 breaths/minute. The nurse understands the pathophysiology of this response and explains to the student nurse that the patient's

chemoreceptors in the carotid and aortic bodies have noticed the pH change and altered the ventilator rate.

Bronchiectasis is considered a secondary COPD, and, with the advent of antibiotics, it is not a common disease entity. In the past, bronchiectasis often followed specific diseases. Which disease did it not follow?

chicken pox

The term, chronic obstructive pulmonary disease (COPD), can be a combination of two types of obstructive airway diseases. What disease processes have been identified as being part of COPD?

chronic obstructive bronchitis and emphysema

Bronchial circulation differs from the pulmonary circulation by providing blood for the:

conducting airways.

A patient is admitted for a relapse for sarcoidosis. Knowing this is usually caused by an inflammatory process, the nurse can anticipate administering:

corticosteroid

A client reports chest pain to the nurse. Which characteristics of the pain indicate bronchial irritation?

dull pain in mid-chest that is worse when coughing

Your patient describes a sensation of shortness of breath and the inability to get enough air. This is documented as:

dyspnea

The nurse assesses a 65-year-old female client who reported to the emergency department with exacerbation of asthma during her daughter's wedding reception. Select all of the triggers that may have precipitated the attack.

emotional stress aspirin dancing

A client who was hospitalized with bacterial pneumonia has now developed an infection in the pleural cavity. The nurse would recognize this as:

empyema

When trying to discern the extent of a burn, the nurse will note that second-degree full-thickness burns are characterized by

extensive pain along with waxy white areas with blister formation.

A client is being treated in the emergency department after being exposed to caustic vapours in a workplace accident. The client's arterial blood gases indicate severe hypoxemia and hypercapnia, so the nurse knows that the client will:

have elevated lactic acid levels due to anaerobic metabolism.

A pneumonia that occurs 48 hours or more after admission to the hospital is considered

hospital-acquired pneumonia.

When explaining the final stages of the inflammatory response to pathogens, the nurse will educate the patient about

how the body can kill the pathogen by generating toxic oxygen and nitrogen products producing such things as nitric oxide and hydrogen peroxide.

Chronic obstructive pulmonary disease (COPD) is a combination of disease processes. What disease processes have been identified as being part of COPD? a) Chronic bronchitis and emphysema b) Emphysema and asthma c) Chronic obstructive bronchitis and emphysema d) Chronic obstructive bronchitis and asthma

hronic obstructive bronchitis and emphysema Correct Explanation: The term chronic obstructive pulmonary disease encompasses two types of obstructive airway disease: emphysema, with enlargement of air spaces and destruction of lung tissue, and chronic obstructive bronchitis, with increased mucus production, obstruction of small airways, and a chronic productive cough. Persons with COPD often have overlapping features of both disorders. Asthma and chronic bronchitis have not been identified as components of COPD.

The common results of respiratory failure are hypoxemia and:

hypercapnia.

The nurse is teaching new nursing assistants on the unit about the phenomenon of muscle hypertrophy. Which of the following clients on the unit is most likely to experience muscle hypertrophy? A client with

hypertension, obesity, and decreased activity tolerance.

Which of the following clients are displaying known risk factors for the development of pulmonary emboli? Select all that apply. A client who is: immobilized following orthopedic surgery. experiencing impaired Cl- and Na+ regulation. taking amiodarone for the treatment of premature ventricular contractions. a smoker and who takes oral contraceptives. undergoing radiation therapy for the treatment of breast cancer.

immobilized following orthopedic surgery. a smoker and who takes oral contraceptives.

A stroke patient is having difficulty swallowing food and beverages. The patient complains that he feels like "the food is sticking to the back of his throat." Given this information, the priority nursing interventions would be to make the patient "nothing per os" (NPO) and call the physician. feed the patient while he is sitting in an upright position. add a thickening agent to all of the patient's beverages. warrant no action since this is a normal occurrence after a stroke.

make the patient "nothing per os" (NPO) and call the physician.

How would you know if you had hypoxemia or hypercapnia?

measure arterial blood gases

Following a workup that included endocrine studies (FSH/LH, prolactin, testosterone, DHEAS levels), a 22-year-old college student complaining of abnormal bleeding has been diagnosed with dysmenorrhea due to alterations in her hormone levels. The nurse should anticipate that she will likely be prescribed: Select all that apply. oral contraceptives. estrogen only. prostaglandin synthetase inhibitors. anxiolytic medications. androgens.

oral contraceptives. estrogen only. prostaglandin synthetase inhibitors.

A patient who has been on a high-protein diet comes to the emergency department with respiratory symptoms. Upon analysis of arterial blood gases (ABGs), the patient is diagnosed with hypercapnia. The nurse will note the ABG results that confirm this diagnosis include: Select all that apply.

pH 7.31 (normal 7.35 to 7.45) PCO2 of 58 mm Hg (normal 38 to 42). Serum HCO3of -33 mEq/L (normal 22 to 28).

A patient who has been on a high-protein diet comes to the emergency department with respiratory symptoms. Upon analysis of arterial blood gases (ABGs), the patient is diagnosed with hypercapnia. The nurse will note the ABG results that confirm this diagnosis include: Select all that apply. pH 7.31 (normal 7.35 to 7.45). PO2 of 97%. PCO2 of 58 mm Hg (normal 38 to 42). Serum HCO3 of 33 mEq/L (normal 22 to 28). Serum K+ (potassium)of 3.6 mmol/L (normal 3.5 to 5.0).

pH 7.31 (normal 7.35 to 7.45). PCO2 of 58 mm Hg (normal 38 to 42). Serum HCO3 of 33 mEq/L (normal 22 to 28).

Which of the following patients is most likely to have impairments to the wound-healing process? A patient with

poorly controlled blood sugars with small blood vessel disease

Which client will the nurse see first? The client with

refractory asthma, PEF 210 ml out of best PEF 450 ml, diminished wheezing.

The ICU nurse is concerned with her patient's arterial blood gas (ABG) results— the pH 7.30; and PCO2 49 mm Hg HCO3 30. The nurse interprets these ABG results to mean :

respiratory acidosis partially compensated

Which client is showing signs of chronic hypoxemia? A client who is

restless, has clubbed fingers, and frequently drops items.

Which of the following early signs could alert you to the presence of hypoxia, particularly in children?

restlessness

The nurse caring for a male patient with respiratory problems is concerned he may be developing respiratory failure. Upon assessment, the nurse knows that which of the following are clinical manifestations of respiratory failure? select all that apply Swollen glottis nasal flaring grunting on expiration inspiratory wheezes heard severe accessory muscle retraction

severe accessory muscle retraction nasal flaring grunting on expiration

A child is brought to the emergency department struggling to breathe with a prolonged bronchospasm and severe hypoxemia. Assessment revealed the use of accessory muscles, a weak cough, audible wheezing sound, moist skin, and tachycardia. Which of the following is the most likely diagnosis?

severe asthma attack

Which client is exhibiting signs of advanced chronic obstructive pulmonary disease (COPD)? The client who is

sitting in bed resting elbows on overbed table, expiratory wheezes noted.

A 68-year-old African American man who has smoked for at least 50 years reports that lately he feels as though food is getting stuck in his throat. At first, this was a problem just with dry food, but now his morning oatmeal is getting "stuck." On questioning, he reports drinking at least three alcoholic beverages nearly every day. His problem is most likely achalasia. squamous cell carcinoma of the esophagus. dysphagia secondary to scleroderma. gastrointestinal reflux disease.

squamous cell carcinoma of the esophagus.

Because they strengthen the pelvic floor muscles, Kegel exercises are most likely to help overflow incontinence. urge incontinence. stress incontinence. mixed incontinence.

stress incontinence.

When trying to explain to a patient diagnosed with a benign bone tumor, the nurse should emphasize the fact that benign tumors primarily grow rapidly and can spread to the surrounding tissue. cause growth of other tumors as they spread their cancer cells into the bloodstream. tend to grow very slowly without destroying the supporting tissue. occur as a result of tumors in other areas of the body metastasizing into the bone.

tend to grow very slowly without destroying the supporting tissue.

A nursing student asks if a client diagnosed with chronic obstructive pulmonary disease (COPD) is at risk if he receives oxygen at a level that increases the PO2 above 60 mm Hg. The best response would be:

the clients ventilation will be severely depressed

During a myocardial infarction (MI), a patient with a 97% occlusion of his left descending artery develops ventricular arrhythmias due to the amount of ischemia occurring in the myocardium. While educating the patient about MIs, the nurse will base her teaching on the fact that

treatment needs to be sought immediately so that the buildup of lactic acid is limited and cellular changes can be reversed.

A 10-year-old male is experiencing an acute exacerbation of his asthma. The most appropriate treatment for this client would be:

β-adrenergic agonist

A 10-year-old male is experiencing an acute exacerbation of his asthma. The most appropriate treatment for this client would be: a) β-adrenergic agonist b) Anti-inflammatory agents c) Antileukotrienes d) Leukotrienes

β-adrenergic agonist Explanation: The β-adrenergic agonist would be the best option to use in an acute exacerbation of asthma. The quick-relief medications include the short-acting β-adrenergic agonist, anticholinergic agents, and systemic corticosteroids. The short-acting β-adrenergic agonist relax bronchial smooth muscle and provide prompt relief of symptoms, usually within 30 minutes. All of the other options would be used as maintenance treatment

Of the following patient conditions, which patient would be at risk for experiencing a thyroid problem due to a decrease in thyroxine-binding globulin (TBG)? Select all that apply.

• 55-year-old male with cirrhosis due to alcohol abuse • 75-year-old man receiving chronic glucocorticoid therapy to treat his severe chronic obstructive pulmonary disease (COPD) • 18-year-old female anorexia nervosa patient weighing 78 pounds who has consumed no protein for the past three years.

If a client with a kidney stone has the "classic" ureteral colic, the client will describe his pain as: Select all that apply. a) Excruciating b) In the flank and upper outer quadrant of the abdomen c) Diffuse over the entire lower back and legs d) Acute, intermittent

• Acute, intermittent • Excruciating • In the flank and upper outer quadrant of the abdomen The symptoms of renal colic are caused by stones 1 to 5 mm in diameter that can move into the ureter and obstruct flow. Classic ureteral colic is manifested by acute, intermittent, and excruciating pain in the flank and upper outer quadrant of the abdomen on the affected side. The pain may radiate to the lower abdominal quadrant, bladder area, perineum, or scrotum in the man. The pain is usually not described as diffuse and over the entire low back and legs.

Of the following list of nursing interventions, which would be considered priority when managing a patient with life-threatening myxedematous coma? Select all that apply.

• Administer 3% sodium IV solution to increase sodium levels. • Administer 50% dextrose to raise glucose levels. • Place on oxygen therapy to encourage deep breathing.

A client has been diagnosed with dysfunction of the anterior pituitary gland. The nurse is aware that which of the following hormones may be affected? Select all that apply.

• Adrenocorticotropic hormone (ACTH) • Thyroid-stimulating hormone (TSH) • Growth hormone (GH) • Luteinizing hormone (LH)

A physician is providing care for a child who has a diagnosis of cystic fibrosis (CF). Place the following events in the etiology of CF in ascending chronological order. Use all the options.

• Airway obstruction • Increased Na+ absorption • Impaired Cl- transport • Recurrent pulmonary infections • Decreased water content of mucociliary blanket

When CO2 levels in the blood rise, a state of hypercapnia occurs in the body. What factors contribute to hypercapnia? (Select all that apply.)

• Alteration in carbon dioxide production • Abnormalities in respiratory function • Disturbance in gas exchange function • Changes in neural control of respiration

A client has just been diagnosed with Hashimoto thyroiditis and is asking the health care provider for information about the condition. The most appropriate information to provide would be: Select all that apply.

• Autoimmune disorder • Common in females • Prone to goiter development

Which clients are at greatest risk of developing an occupational lung disease? Select all that apply.

• Bricklayer • Coal miner • Farmer • Flour miller

A woman who is exhibiting clinical manifestations of a pituitary adenoma will likely complain of: Select all that apply.

• Cessation of menses • Unusual milk secretion unrelated to pregnancy • Infertility

A client with newly diagnosed squamous cell carcinoma of the lung asks, "So how do we treat this cancer?" Which response from the health care provider is most accurate? Select all that apply. a) Monoclonal antibody b) Stem cell transplant c) Chemotherapy d) Surgery to remove tumor e) Radiation therapy

• Chemotherapy • Surgery to remove tumor • Radiation therapy Explanation: Although all forms of lung cancer are serious and potentially fatal diagnoses, individuals with small cell lung cancer, adenocarcinoma, and large cell lung cancer often face prognoses that are worse than those associated with squamous cell lung cancer. Treatment methods for NSCLC include surgery, radiation therapy, and systemic chemotherapy. These treatments may be used singly or in combination. Stem cell transplant is not utilized in lung cancer. New trials are exploring the use of monoclonal antibodies but have not been approved by the FDA at this time

The nurse is caring for a client with secondary adrenal insufficiency. What subjective assessment data obtained by the nurse would correlate with this client's condition?

• Client states, "I don't feel like eating anything." • Client states, "I feel as though I am going to vomit." • The client states that she feels weak

A client with hypothyroidism has not taken medication for several months, informing the nurse that she lost her insurance and is unable to afford the medication. When assessing the client's temperature tolerance and skin, what does the nurse anticipate finding? Select all that apply.

• Coarse and dry skin and hair • Intolerance to cold • Decreased sweating

A patient is diagnosed with restrictive lung disease caused by fibrosis. Which of the following clinical manifestations is expected? Select all that apply.

• Decreased tidal volume • Increased respiratory rate • Nonproductive cough • Clubbing of fingers

Which are the earliest manifestations that a client is developing hypoxemia? Select all that apply.

• Diaphoresis • Tachycardia

Which client would be considered at a higher risk for developing Goodpasture Syndrome, an aggressive form of glomerulonephritis? Select all that apply. a) Elderly person with a recent influenza infection b) Painter working with a busy construction crew who finishes all the house painting indoor and out c) Electrician who wires homes and businesses for high-definition Internet d) Hair style expert who specializes in coloring [dyes] hair e) Middle-aged over-the-road truck driver

• Elderly person with a recent influenza infection • Painter working with a busy construction crew who finishes all the house painting indoor and out • Hair style expert who specializes in coloring [dyes] hair The cause of the disorder [Goodpasture Syndrome] is unknown, although influenza infection and exposure to hydrocarbon solvent (found in paints and dyes) have been implicated in some persons, as have various drugs and cancers. There is not a greater risk associated with truck drivers or electricians.

The nurse is caring for a client with primary hypothyroidism who is being monitored for the complication of myxedema coma while thyroid level therapy is started. What does the nurse understand are the three major aspects of myxedema coma?

• Elevated carbon dioxide levels and decreased oxygen saturation • Fluid and electrolyte imbalance • Hypothermia

The nurse is caring for a patient who has not received prenatal care during her pregnancy. The patient delivers a newborn who dies an hour after birth because of total agenesis of both kidneys. Which of the following physical features does the newborn exhibit that correlates with this diagnosis? Select all that apply. a) Imperforate anus b) Broad and flat nose c) Low-set ears d) Cleft lip e) Eyes widely separated with epicanthic folds

• Eyes widely separated with epicanthic folds • Low-set ears • Broad and flat nose Total agenesis of both kidneys is incompatible with extrauterine life. Infants are stillborn or die shortly after birth of pulmonary hypoplasia. Newborns with renal agenesis often have characteristic facial features, sometimes called Potter syndrome, resulting from the effects of oligohydramnios. The eyes are widely separated and have epicanthic folds, the ears are low set, the nose is broad and flat, the chin is receding, and limb defects are often present.

Which interventions would be included in the plan of care for a client experiencing myxedematous coma? Select all that apply.

• Frequent monitoring of cardiac functions • Initiation of thyroid replacement therapy • Oxygen therapy to support ineffective respirations • Reversal of hypoglycemia

A client has been recently undergone diagnostic testing for possible Berger disease. The nurse caring for this client would anticipate the primary clinical manifestations include which of the following? Select all that apply. a) Fever, chills, and general body aches b) Gross hematuria c) Recent upper respiratory infection d) Elevated ketone levels in the urine

• Gross hematuria • Recent upper respiratory infection • Fever, chills, and general body aches Early in the disease, many people with the disorder have no obvious symptoms, and the disorder is discovered during screening or examination for another condition. In others, the disorder presents with gross hematuria that is preceded by upper respiratory tract infection, GI tract symptoms, or flulike illness. The hematuria lasts 2 to 6 days. Elevated ketones are usually associated with acidosis, fasting, high-protein diet, or diabetes to name a few.

A nurse is assessing an elderly woman diagnosed with chronic hypothyroidism who has developed myxedematous coma. The nurse is aware that the client may manifest: Select all that apply.

• Hypoventilation • Hyponatremia • Hypoglycemia • Lactic acidosis

A client with cystic fibrosis reports recent weight loss without fever or cough. What interventions will the nurse implement to promote health for the client? Select all that apply.

• Increase protein intake • Increase pancreatic enzymes

A client with cystic fibrosis reports recent weight loss without fever or cough. What interventions will the nurse implement to promote health for the client? Select all that apply. a) Increase protein intake b) Increase chest percussion c) Increase aerobic activity d) Increase pancreatic enzymes e) Increase mucolytic use

• Increase protein intake • Increase pancreatic enzymes Explanation: Cystic fibrosis is a genetic disorder that produces abnormal functioning in the epithelial cell chloride channels. This leads to thick, viscous secretions that block the lung passages and the pancreatic ducts resulting in chronic lung infections, and malabsorption and malnutrition. Treatment for CF is intended to maintain maximum function while minimizing secondary organ damage. Antibiotic therapy, chest percussion and drainage, and adequate hydration are appropriate for respiratory symptoms. Pancreatic enzymes, vitamin and mineral supplements, increased protein intake, and insulin may be needed with pancreatic involvement.

A 2-year-old child is admitted to pediatric unit with bronchiolitis. The nurse calls the physician fearing the child is going into respiratory failure based on which of the following assessment findings? Select all that apply. a) Faint wheezes noted in the posterior lung base b) Productive cough with white secretions c) Increased respiratory rate to 44 breaths/minute d) New-onset expiratory grunting e) Substernal retractions becoming more pronounced

• Increased respiratory rate to 44 breaths/minute • Substernal retractions becoming more pronounced • New-onset expiratory grunting Explanation: Children with impending respiratory failure due to airway or lung disease have rapid breathing; exaggerated use of the accessory muscles; retractions, which are more pronounced in the child than in the adult because of higher chest compliance; nasal flaring; and grunting during expiration.

Growth hormone exerts its effects on the body in many ways. Which of these are effects of GH? (Select all that apply.)

• Increases insulin levels • Enhances fatty acid mobilization • Facilitates the rate of protein synthesis

Testing for short statue growth hormone (GH) problems can be done by pharmacologic means. Which of the following medications can be utilized to test for a rise in GH? Select all that apply.

• Insulin • Levodopa

Which symptoms accompanying shortness of breath indicate a client has cor pulmonale? Select all that apply.

• Jugular vein distention • 2+ pitting edema in feet • Warm moist skin

A nurse is assessing a client for cyanosis. The most appropriate areas for the nurse to assess would be: Select all that apply.

• Lips • Nails • Ears • Cheeks

The nurse is educating a client with hyperthyroidism who has recovered from thyrotoxicosis (thyroid storm) and is preparing for discharge. What can the nurse inform the client that this hypermetabolic state can be caused by in order to decrease exacerbation? Select all that apply.

• Manipulation of the thyroid gland • Physical or emotional trauma • Stress

Which statements are true regarding hypothyroidism? Select all that apply.

• Onset of symptoms is gradual • Weight gain • Anorexia

Which manifestations are most common in clients with COPD that is predominantly chronic bronchitis? Select all that apply.

• Peripheral edema • Increased mucus secretion • Cyanosis

The nurse is receiving physician orders when admitting a client to the inpatient facility with cirrhosis of the liver. Which medications ordered by the physician should the nurse question since they may affect the binding of thyroid hormone to normal concentrations of binding proteins? Select all that apply.

• Phenytoin (Dilantin) • Aspirin • Diazepam (Valium)

A client has a tubulointerstitial disorder and is no able to concentrate urine. The nurse will likely assess which manifestations associated with this disorder? Select all that apply. a) Acidic urine b) Nocturia c) Hematuria d) Polyuria

• Polyuria • Nocturia The tubulointerstitial disorders are distinguished clinically from glomerular diseases by the absence, in the early stages, of such hallmarks of nephritis and nephrosis as hematuria and proteinuria, and by the presence of disorders in tubular function. These disorders, which are often subtle, include the inability to concentrate urine, as evidenced by polyuria and nocturia; interference with acidification of urine, resulting in metabolic acidosis; and diminished tubular reabsorption of sodium and other substances.

Congenital disorders of the kidneys are fairly common, occurring in approximately 1:1000 live births. What is the result to the newborn when bilateral renal dysplasia occurs? (Select all that apply.) a) Oligohydramnios b) Potter facies c) Pulmonary hypoplasia d) Multicystic kidneys e) Renal failure

• Potter facies • Oligohydramnios • Pulmonary hypoplasia • Renal failure Bilateral renal dysplasia causes oligohydramnios and the resultant Potter facies, pulmonary hypoplasia, and renal failure. Multicystic kidneys are a disorder, not the result of a congenital problem.

A nurse is caring for a child with Wilms tumor, stage I. Which of the following can be said regarding this diagnosis? Select all that apply. a) The tumor is limited to the kidney and can be excised with the capsular surface intact. b) The tumor has undergone hematogenous metastasis, most commonly involving the lung. c) Prognosis is good with treatment. d) Prognosis is poor even with treatment.

• Prognosis is good with treatment. • The tumor is limited to the kidney and can be excised with the capsular surface intact. Long-term survival rates have increased to 90% for Wilms tumor stages I through III. The tumors usually are staged using the National Wilms' Tumor Study Group classification: stage I tumors are limited to the kidney and can be excised with the capsular surface intact.

Acute lung injury/acute respiratory distress syndrome (ALI/ARDS) is distinguishable between the two by the extent of hypoxemia involved. What is the clinical presentation of ARDS? (Select all that apply.)

• Rapid onset • Increase in respiratory rate • Hypoxemia refractory to treatment

A nurse is providing care for a client who has been admitted to a medical unit with a diagnosis of bronchiectasis. Which of the following signs and symptoms should the nurse expect to find during physical assessment of the client and the review of the client's history? Select all that apply:

• Recurrent chest infections • Production of purulent sputum • Low hemoglobin levels

A client has just undergone a diagnostic cardiac angiogram. As part of their ordered labs, the physician has ordered a thyroid panel. The physiological principle behind ordering this lab tests includes which of the following correlations? Hyperthyroidism can cause: Select all that apply.

• Rise in oxygen consumption • Increase in cardiac output

The nurse is caring for a client who is in a severely hypothyroid state. Which medications should be avoided related to the inability to metabolize these drugs? Select all that apply.

• Sedatives • Analgesics • Anesthetics

A child has been diagnosed with classic growth hormone deficiency. The child may experience: Select all that apply.

• Short stature • Delayed skeletal muscle maturation • Obesity

A patient with sepsis secondary to pneumonia develops acute respiratory distress syndrome (ARDS) and a low ventilation-perfusion ratio. Which of the following alterations in the lungs occurs as the disease progresses? Select all that apply.

• Surfactant inactivation • Intrapulmonary shunting • lung stiffens • Alveolar collapse

A patient with sepsis secondary to pneumonia develops acute respiratory distress syndrome (ARDS) and a low ventilation-perfusion ratio. Which of the following alterations in the lungs occurs as the disease progresses? Select all that apply. a) Intrapulmonary shunting b) Surfactant inactivation c) lung stiffens d) Bronchoconstriction e) Alveolar collapse

• Surfactant inactivation • Intrapulmonary shunting • lung stiffens • Alveolar collapse Explanation: ARDS causes diffuse alveolar cell damage which leads to accumulation of fluid, surfactant inactivation, and formation of a hyaline membrane that is impervious to gas exchange. As the disease progresses, the work of breathing becomes greatly increased as the lung stiffens and becomes more difficult to inflate. There is increased intrapulmonary shunting of blood, impaired gas exchange, and refractory hypoxemia despite high supplemental oxygen therapy. Gas exchange is further compromised by alveolar collapse resulting from abnormalities in surfactant production

A patient with pulmonary hypertension may display which of the following clinical manifestations? Select all that apply.

• Swell [edema] in their legs and feet. • Shortness of breath. • Decreased exercise tolerance.

Hyperthyroidism that is inadequately treated can cause a life-threatening condition known as a thyroid storm. What are the manifestations of a thyroid storm? (Select all that apply.)

• Tachycardia • Delirium • Very high fever

A client is suspected to have increased growth hormone levels. When performing a health history, what assessment data would be important for the nurse to report to the physician? Select all that apply.

• The client has fainted due to low blood glucose levels on several occasions. • The client has been experiencing a great deal of emotional stress due to family issues. • The client is on a weight loss diet and is exercising excessively.

A COPD patient asks the nurse which of the following medications is prescribed to help their breathing. The nurse, looking at the list of medications, will educate the patient about which of the following medications to help their COPD in the long-term. Select all that apply.

• Tiotropium (Spiriva), anticholinergic. • Salmeterol (Serevent), a bronchodilator.

A client seen in the emergency department after a motor vehicle accident complains of increasing shortness of breath. A left tension pneumothorax is suspected. What manifestations support the diagnosis? Select all that apply.

• Tracheal deviation to the right • Hyperresonance on the left • Diminished breath sounds on the left • Subcutaneous emphysema

A client seen in the emergency department after a motor vehicle accident complains of increasing shortness of breath. A left tension pneumothorax is suspected. What manifestations support the diagnosis? Select all that apply. a) Diminished breath sounds on the left b) Tracheal deviation to the right c) Reduced pulses on the right d) Subcutaneous emphysema e) Hyperresonance on the left

• Tracheal deviation to the right • Hyperresonance on the left • Diminished breath sounds on the left • Subcutaneous emphysema Explanation: Tension pneumothorax allows air to compress the lung on the affected side. This pushes the mediastinal organs and trachea to the opposite side. The affected side has hyperresonance and the breath sounds are decreased. The peripheral pulses are not affected by pneumothorax. Subcutaneous emphysema is air under the skin of the chest and neck.

A student nurse is taking a test on the endocrine system. From the following list of clinical manifestations, she needs to select the ones she would see in hypothyroidism. Which answers should she select? Select all that apply.

• Weight gain despite loss of appetite • Coarse brittle hair • Puffy face with swollen eyelids

Which manifestations are most common in clients with COPD that is predominantly emphysema? Select all that apply. a) peripheral edema b) cyanosis c) hyperresonance d) increased A-P diameter e) increased mucus secretion

• hyperresonance • increased A-P diameter Explanation: Emphysema is noted for its loss of lung elasticity and enlargement of the distal air spaces. This leads to hyperresonance, decreased breath sounds, and an increase in the A-P chest diameter, commonly called barrel chest. Clients with emphysema tend to take short frequent breaths and exhale through pursed lips. In chronic bronchitis, there is an increase in mucus production, cyanosis, and often fluid retention associated with right heart failure.

A client is admitted with acute pyelonephritis. As the nurse enters the room, the client has shaking chills and reports acute pain. Other clinical manifestations the nurse will assess includes: Select all that apply. a) frequency, urgency, and dysuria. b) change in level of consciousness and hallucinations. c) shallow breathing and expiratory wheezes. d) moderate to high fever. e) pale skin and dry mucous membranes.

• moderate to high fever. • frequency, urgency, and dysuria. The onset of acute pyelonephritis is usually abrupt, with shaking chills, moderate to high fever, and a constant ache in the loin area of the back that is unilateral or bilateral. Lower urinary tract symptoms, including dysuria, frequency, and urgency, also are common. It is not associated with expiratory wheezes, hallucinations, or dry mucous membranes. (less)


Set pelajaran terkait

128 Civics Questions & Answers (2020 Version)

View Set

Midterm Exam for Speech Communication

View Set

Medical Terminology - Body Structure

View Set